Download as pdf or txt
Download as pdf or txt
You are on page 1of 228

PHARMACOLOGY SAS

SAS 1

Multiple Choice
1. What is pharmacokinetics?
a. The study of biological and therapeutic effects of drugs
b. The study of absorption, distribution, metabolism and excretion of drugs
c. The study of mechanisms of drug action
d. The study of methods of new drug development
_______________________________________
2. The interaction of one drug increased by the presence of a second drug is known as:
a. Potentiation
b. Addictive effects
c. Antagonism
d. Synergism

3. Considering the absorption of medications during administration, which of the following routes would yield the quickest response?
a. Oral Medications
b. Subcutaneous injections
c. Intravenous medications
d. Topical medications

4. Which of the following best defines the process of pharmacokinetics?


a. The effect of medication on the body
b. The effect of the body on medication
c. The study of medications
d. The preparation of medications for administration

5. When performing an assessment about medication drug history should include which of the following?
a. Complete vital signs
b. Reason for medication
c. Client’s goal therapy
d. Administration of OTC medications

6. When considering the pharmacotherapeutic effects of drugs administered to clients, the nurse considers which property of most
Importance:
a. Efficacy
b. Potency
c. Interaction with other drugs
d. Toxicity

7. Safety of a drug is determined by the degree between which of the following?


a. Therapeutic and toxic doses
b. Potency and efficacy
c. Subtherapeutic and toxic levels
d. Side and adverse effects

8. The nurse is monitoring the therapeutic drug level for a client on vancomycin (Vancocin) and notes that the level is within the
acceptable range. What does this indicate to the nurse? Select all that apply.
a. The drug should cause no toxicities or adverse effects
b. The drug level is appropriate to exert therapeutic effects
c. The dose will not need to be changed for the duration of treatment
d. the nurse will need to continue monitoring because each client response to a drug is unique
9. When a drug is 50% protein bound it means that:
a. 50% of the drug destroys protein
b. A drug not bound to protein is an active drug
c. 50% of the dose is at work
d. Protein must be restricted in the diet

10. When reviewing the patient’s medication regimen, the nurse understands that the interval of drug dosage is related to what?
a. Half-life
b. Biotransformation
c. Metabolism
d. Therapeutic effect

11. It is important for the nurse to be aware of the four sequential processes of the pharmacokinetic phase. What are these
processes?
a. Distribution, metabolism, excretion, absorption
b. Biotransformation, excretion, absorption, metabolism
c. Absorption, distribution, metabolism, excretion
d. Metabolism, distribution, absorption, excretion

12. The nurse is teaching the client about newly prescribed medication. Which statement made by the client would indicated the
need for further medication education?
a. "the liquid form of the drug will be absorbed faster than the tablets."
b. "If I take more, I'll have a better response"
c. "taking this drug with food will decrease how much gets into my system."
d. "I can consult my health care provider if i experience unexpected adverse effects"

13. The nurse is caring for several clients. Which client will the nurse anticipate is most likely to experience an alteration in drug
metabolism?
a. A 3 day old premature infant
b. A 22 year old pregnant female
c. A 32 year old man with kidney stones
d. A 50 year old executive with hypertension

14. A client is being discharged from the hospital with a nebulizer for self-administration of inhalation medication. Which statement
made by the client indicates to the nurse that the client education has been successful?
a. "Inhaled medications should only be taken in the morning."
b. "Doses for inhaled medications are larger than those taken orally"
c. "Medicines taken by inhalation produce a very rapid response."
d. "Inhaled drugs are often rendered inactive by hepatic metabolism reaction”

15. The Nurse is caring for a client with hepatitis and resulting in hepatic impairment. The nurse would expect the duration of action
for most medications to:
a. Decrease
b. Improve
c. Be unaffected
d. Increase
SAS 2

1. The nursery nurse is putting erythromycin ointment in the newborn’s eyes to prevent infection. She places it in the following area
of the eye:
a. Under the eyelid
b. On the cornea.
c. In the lower conjunctival sac
d. By the optic disc.

2. Nursing responsibilities in the assessment phase of the nursing process include which responsibilities?
(Select all that apply.)
a. Identify side effects of drugs that are nonspecific
b. Check peak and trough levels of drugs
c. Advise client to avoid fatty foods prior to ingesting an enteric coated tablet
d. Evaluate client's reaction to drug

3. The nurse is aware of the many factors related to effective health teaching about the medication. The most essential component of
the teaching plan is to do which?
a. Provide written instructions.
b. Establish a trust relationship.
c. Use colorful charts.
d. Review community resources.

4. A medication health teaching plan is tailored to a specific client. Common topics for health teaching include which? (Select all that
apply.)
a. Importance of adherence to the prescribed regimen
b. How to administer medication (s)
c. What side/adverse effects to report to the health care provider
d. Instruction of the client on what foods should be eaten

5. The nurse educator on the unit receives a list of high-alert drugs. Which strategies are recommended to decrease the risk of errors
with these medication?
(Select all that apply.)
a. Store medications alphabetically on their usual shelf.
b. Limit access to these drugs.
c. Use special labels.
d. Provide increased information to staff.

6. Nurse is preparing to administer a medication to a 13-year-old client. The nurse follows the six rights of medication administration
for a pediatric client. After checking for the right client, the right dose, the right drug, the right time and the right route, what is the
final item the nurse must check for this client?
a. Right age
b. Right Label
c. Right Documentation
d. Right strength

7. When performing an assessment about medication, the drug history should include:
a. Complete vital signs
b. Client’s goal of therapy
c. Reason for medication
d. Administration of OTC medications

8. The volume of SC medication must be no more than:


a. 0.5 mL
b. 1.0 mL
c. 1.5 mL
d. 3.0 mL
9. Which of the following muscles is a possible site for IM injections?
a. Outer aspect of the hip
b. Shoulder
c. Vastus gluteus
d. Vastus lateralis

10. When deciding on what time of day to give medications, the nurse pays closest attention to the client’s habits regarding:
a. Eating
b. Sleeping
c. Elimination
d. Activity

11. Which of the following can hinder client’s ability to take oral medications?
a. Age
b. Dysphagia
c. Dental caries
d. Lifestyle

_________________________________________________________________________________________________
12. When performing an assessment to determine which medications can be used, which of the following elements is most
important?
a. Physical examination
b. Age
c. Allergies
d. Weight

13. Central venous access devices are beneficial in pediatric therapy because:
a. They are difficult to see.
b. They cannot be dislodged
c. Use of the arms is not restricted
d. They don’t frighten children

14. A client is nauseated, has been vomiting for several hours, and needs to receive an antiemetic (anti-nausea) medication. Which of
the following is accurate?
a. An enteric-coated medication should be given.
b. Any medication will not be absorbed as easily because of the nausea problem.
c. A parenteral route is the route of choice.
d. A rectal suppository must be administered.

15. The nurse uses a mortar and pedestal to crush a medication before giving it to one of her clients. Which of the five rights is the
nurse ensuring?
a. The right route
b. The right client
c. The right time
d. The right drug
SAS 3

1. Doctor’s Order: Tylenol supp 1 g pr q 6 hr prn temp > 101; Available: Tylenol supp 325 mg (scored). How many supp will you
administer?
a. 2 supp
b. 1 supp
c. 3 supp
d. 5 supp

2. Doctor’s Order: Nafcillin 500 mg po pc; Available: Nafcillin 1 gm tab (scored). How many tab will you administer per day?
a. 2.5 tabs
b. 2 tabs
c. 1.5 tabs
d. 1 tab

3. Doctor’s Order: Synthroid 75 mcg po daily; Available: Synthroid 0.15 mg tab (scored). How many tab will you administer?
a. 1 tab
b. 0.5 tab
c. 2 tabs
d. 1.5 tabs

4. Doctor’s Order: Diuril 1.8 mg/kg po tid; Available: Diuril 12.5 mg caps. How many cap will you administer for each dose to a 31 lb
child?
a. 2 caps
b. 2.5 caps
c. 3 caps
d. 1.5 caps

5. Doctor’s Order: Cleocin Oral Susp 600 mg po qid; Directions for mixing: Add 100 mL of water and shake vigorously. Each 2.5 mL will
contain 100 mg of Cleocin. How many tsp of Cleocin will you administer?
a. 3 tsp
b. 5 tsp
c. 3.5 tsp
d. 1 tsp

6. Doctor’s Order: Sulfasalazine Oral Susp 500 mg q 6 hr; Directions for mixing: Add 125 mL of water and shake well. Each tbsp will
yield 1.5 g of Sulfasalazine. How many mL will you give?
a. 5 ml
b. 3 ml
c. 4 ml
d. 2 ml

7. Doctor’s Order: Kantamycin 7.5 mg/kg IM q 12 hr; Available: Kantamycin 0.35 Gm/mL. How many mL will you administer for each
dose to a 157 lb patient?
a. 2 ml
b. 1 ml
c. 2.5 ml
d. 1.5 ml

8. Doctor’s Order: Heparin 7,855 units Sub Q bid; Available: Heparin 10,000 units per ml. How many mL will you administer?
a. 0.79 ml
b. 1.79 ml
c. 0.17 ml
d. 1.17 ml

9. Doctor’s Order: Demerol 50 mg IVP q 6 hr prn pain; Available: Demerol 75 mg/ 1.3mL. How many mL will you administer?
a. 0.87 ml
b. 1.87 ml
c. 2 ml
d. 2.87 ml

10. Doctor’s Order: Streptomycin 1.75 mg/ lb IM q 12 hr; Available: Streptomycin 0.35 g / 2.3 mL. How many mL will you administer a
day to a 59 Kg patient?
a. 1.5 ml
b. 2 ml
c. 2.5 ml
d. 3 ml

11. Doctor’s Order: Bumex 0.8 mg IV bolus bid; Reconstitution instructions: Constitute to 1000 micrograms/3.1 mL with 4.8mL of 5%
Dextrose Water for Injection. How many mL will you administer?
a. 2 ml
b. 3.5 ml
c. 3 ml
d. 2.5 ml

12. Doctor’s Order: Infuse 50 mg of Amphotericin B in 250 mL NS over 4 hr 15 min; Drop factor: 12gtt/mL. What flow rate (mL/hr) will
you set on the IV infusion pump?
a. 11.8 ml/hr
b. 58.8 ml/hr
c. 14.1 ml/hr
d. 60.2 ml/hr

13. Doctor’s Order: 1 ½ L of NS to be infused over 7 hours; Drop factor: 15 gtt/mL. What flow rate (mL/hr) will you set on the IV
infusion pump?
a. 53.6 ml/hr
b. 214.3 ml/hr
c. 35.7 ml/hr
d. 142.9 ml/hr

14. Doctor’s Order: Mandol 300 mg in 50 mL of D5W to infuse IVPB 15 minutes; Drop factor: 10 gtt/mL. How many mL/hr will you set
on the IV infusion pump?
a. 200 ml/hr
b. 87.5 ml/hr
c. 3.3 ml/hr
d. 50 ml/hr

15. Doctor’s Order: ¼ L of D5W to infuse over 2 hr 45 min; Drop factor: 60 gtt/mL. How many gtt/min will you regulate the IV?
a. 91 gtt/min
b. 96 gtt/min
c. 125 gtt/min
d. 142 gtt/min
SAS 4

1. The nurse is administering an anti-infective that is considered to be a bacteriocidal. Before beginning therapy, the nurse should
assess the client for:
a. Any neurologic abnormalities.
b. Cardiac history.
c. Hypersensitivity.
d. Respiratory conditions.

2. The client asks the nurse why the physician didn't prescribe the same antibiotic that he always takes for an infection. The best
response by the nurse would be:
a. "It doesn't matter which antibiotic is taken."
b. "Try this medicine, and if you're not better in 10 days, return to the office."
c. "You don't want to take the same antibiotic all the time."
d. "Bacteria can become resistant to some antibiotics.

3. A nurse is preparing to administer a broad-spectrum antibiotic medication to a client. An important nursing intervention prior to
administration regarding anti-infectives is:
a. Obtaining the culture report before starting any medication.
b. Performing a culture within 24 hours after starting the medication.
c. Performing the culture for evidence before administering the first dose of the anti-infective.
d. Administering medicine, and omitting performing cultures.

4. A client has been on an antibiotic for two weeks for treatment of an ulcer caused by Helicobacter pylori. The client asks the nurse
why a superinfection is caused by this medication. The nurse responds:
a. "This is a secondary infection due to "Candida."
b. "The infection has developed immunity to the current drug."
c. "The infection has become severe."
d. "The infection has a restricted group of microorganisms."

5. A client has been discharged with a prescription for penicillin. Discharge instructions includes which of the following?
a. Penicillin can be taken while breastfeeding.
b. The entire prescription must be finished.
c. All penicillin can be taken without regards to eating.
d. Some possible side effects include abdominal pain and diarrhea

6. An antimicrobial medication that has selective toxicity has which characteristic?


a. Ability to transfer DNA coding
b. Ability to suppress bacterial resistance
c. Ability to avoid injuring host cells
d. Ability to act against a specific microbe

7. The development of a new infection as a result of the elimination of normal flora by an antibiotic is referred to as what?
a. Resistant infection
b. Superinfection
c. Nosocomial infection
d. Allergic reaction

8. A microbe acquires antibiotic resistance by which means?


a. Development of medication resistance in the host
b. Over-riding of the minimum bactericidal concentration
c. Incorrect dosing, which contributes to ribosome mutations
d. Transfer of DNA coding to other bacteria

9. The nurse identifies which host factor as the most important when choosing an antimicrobial drug?
a. Age
b. Competent immune function
c. Genetic heritage
d. Previous medication reactions

10. A nurse removes a central line access device once the patient no longer requires intravenous (IV) antibiotics. This action is an
example of which strategy to prevent antimicrobial resistance established by the Centers for Disease Control and Prevention (CDC)?
a. Preventing transmission
b. Proper diagnosis
c. Preventing infection
d. Prudent antibiotic use

11. A patient has acquired an infection while in the hospital. The nurse identifies this type of infection as what?
a. Superinfection
b. Suprainfection
c. Nosocomial infection
d. Resistant infection

12. Which of the following is an example of anti- infective used as a means of prophylaxis?
a. Amoxicillin used to treat tonsillitis
b. Penicillin used to treat abcess
c. An antibiotic used before dental surgery
d. Norfloxacin used to treat bladder infection
Answer C AN antibiotic used before dental surgery

13. A nurse is assessing the effects of antimicrobial therapy in a patient with pneumonia. The nurse should establish which outcomes
when planning care? (Select all that apply.)
a. Potassium level of 4 mEq/dL
b. Reduction of fever
c. Sterile sputum culture
d. Oxygen saturation of 99%
e. Elastic skin turgor

14. This anti-infective agent exerts its effect by interfering with steps in protein synthesis.
a. Macrolide
b. Sulfonamides
c. Antimycobacterial
d. Trimethoprim-sulfamethoxazole

15. When an anti-infective agent is said to be no longer effective for a patient, the patient has developed:
a. Immunity
b. Tolerance
c. Non-responsiveness
d. Resistance
SAS 5

1. A client has been prescribed tetracycline. When providing information regarding this drug, the nurse would be correct in stating
that tetracycline:
a. Is classified as a narrow-spectrum antibiotic.
b. Is used to treat a wide variety of disease processes.
c. Has been identified to be safe during pregnancy.
d. Is contraindicated in children under 8 years of age.

2. Important information to include in the client's education regarding taking aminoglycosides is that:
a. The drug can cause discoloration of teeth.
b. Fluid intake should be decreased to prevent retention.
c. This drug primarily is given orally, because it is absorbed in the GI tract.
d. A serious side effect is hearing loss.

3. A client has been prescribed ciprofloxacin (Cipro). Important information that the nurse must know includes:
a. This medicine must be taken on an empty stomach to increase absorption.
b. This medicine is classified as an aminoglycoside and is given for systemic bacterial infections.
c. This medicine should be given with an antacid to increase the absorption and effectiveness of the medicine.
d. This medicine should not be given with the ordered multivitamin.

4. A nursing intervention for administering sulfamethoxazole-trimethoprim (Bactrim) to a client is to:


a. Have the client drink a full glass of water with the medicine.
b. Have the client drink a glass of milk.
c. Have the client take the medicine with solid foods.
d. Have the client take the medicine on an empty stomach.

5. This antibiotic can have an adverse effect of dizziness, vertigo, and loss of hearing.
a. Chloramphenicol
b. Aminoglycoside
c. Vancomycin
d. Fluoroquinolones

6. A 60 year old male patient on fourth day of his antibiotic therapy complains of dizziness and nausea. Which is the most
appropriate nursing action?
a. Institute safety precaution and raise side rails.
b. Collaborate with the doctor about antibiotic therapy’s dosage and duration.
c. Provide comfort measures.
d. Instruct client to inform you if symptoms get worse.

7. A middle-aged woman came to the ER and complains of ringing in the ears, paresthesias of the extremities, and erythema of the
back. She also noticed that she had decreased urine output. What history of drug intake should the nurse ask?
a. Oral contraceptive pills (OCPs)
b. Antifungals
c. Vancomycin
d. Trimethoprim-sulfamethoxazole

8. The physician orders penicillin for a patient with streptococcal pharyngitis. The nurse administers the drug as ordered, and the
patient has an allergic reaction. The nurse checks the medication order sheet and finds that the patient is allergic to penicillin. Legal
responsibility for the error is:
a. Only the nurse’s—she should have checked the allergies before administering the medication.
b. Only the physician’s—she gave the order, the nurse is obligated to follow it.
c. Only the pharmacist’s—he should alert the floor to possible allergic reactions.
d. The pharmacist, physician, and nurse are all liable for the mistake
9. James Perez, a nurse on a geriatric floor, is administering a dose of digoxin to one of his patients. The woman asks why she takes a
different pill than her niece, who also has heart trouble. James replies that as people get older, liver and kidney function decline, and
if the dose is as high as her niece’s, the drug will tend to:
a. Have a shorter half-life.
b. Accumulate.
c. Have decreased distribution.
d. Have increased absorption.

10. The nurse is administering augmentin to her patient with a sinus infection. Which is the best way for her to insure that she is
giving it to the right patient?
a. Call the patient by name
b. Read the name of the patient on the patient’s door
c. Check the patient’s wristband
d. Check the patient’s room number on the unit census list

11. The most important instructions a nurse can give a patient regarding the use of the antibiotic ampicillin prescribed for her are to
a. Call the physician if she has any breathing difficulties.
b. Take it with meals so it doesn’t cause an upset stomach.
c. Take all of the medication prescribed even if the symptoms stop sooner.
d. Not share the pills with anyone else.

12. A woman who had been taking an antibiotic for a UTI calls the nurse practitioner to complain of severe vaginal itching. she also
had noticed a thick, white vaginal discharge. the nurse practitioner suspects that
a. This is an expected response to antibiotic therapy
b. The UTO had become worse instead of better
c. A superinfection has developed
d. The UTI is resistant to the antibiotic

13. What are the potential side effects or adverse effects of tetracycline? Select all that applies.
a. GI discomfort
b. Hepatoxicity
c. Photoensitivity
d. Nausea

14. What class of antibiotic should be avoided in clients allergic to thiazide diuretics, and sulfonylurea-type oral hypoglycemics?
a. Sulfonamides
b. Cephalosporins
c. Flouroquinolones
d. Macrolides

15. The nurse is reviewing the orders for wound care, which include use of an antiseptic. which statement best describes the use of
antiseptics
a. Antiseptics are appropriate for use on living tissue
b. Antiseptic work by sterilizing the surface of the wound
c. The patients allergies must be assessed before using the antiseptic
d. Antiseptics are used to inhibit the growth of microorganisms on the wound surface
SAS 6

1. A patient receiving topical antifungal complains of blisters in her perineum. Which is/are a possible explanation(s) for this?
a. Fungal infection is not healing
b. Patient is allergic to the drug.
2. Which of the following will alert the nurse for possible adverse effect in patients receiving long-term itraconazole therapy?
a. Central obesity
3. An eight (8) month old infant is receiving antifungals. Which should be included in the nurse’s health teaching to the
mother?
b. Make sure area is free from occlusive dressings.
4. Which of these antifungals can be used in pregnant women?
c. Amphotericin B
5. Arvic, a 16-year-old student, has acquired systemic fungal infection, he should be treated with: a. Amphotericin B
6. Which of the following must always be present before beginning antifungal therapy? b. Confirmed diagnosis
7. A 32-year-old woman presents to her gynecologist with a 4 days history of perineal pruritus and a non-malodorous, thick,
cheesy vaginal discharge. The only medication the woman is taking is an oral contraceptive. A wet preparation of vaginal
secretion shows budding yeast cells and pseudohyphae. Which of the following drugs, given locally, would be appropriate
for this patient? a. Mebendazole
d. Miconazole
8. The nurse sees a patient in the clinic who has been taking chloroquine for the treatment of malaria. While the nurse
measures vital signs, the patient repeatedly rubs her eyes. When the nurse questions why, the patient says, "I guess it's time
for a trip to the eye doctor. My glasses don't seem to work very well and I'm having trouble with my vision." What does the
nurse suspect is happening with this patient?
a. Adverse effect of medication
9. The nurse is caring for a patient taking antimalarials for prophylaxis while serving in the Peace Corps in Africa. The patient
has taken the medication for 2 months and is continuing to lose significant weight due to the GI effects of the drug. What
recommendations can the nurse make to reduce GI adverse effects and promote healthy nutrition for this patient? (Select
all that apply.)
d. Eat 5 to 6 small meals a day.
10. An instructor is describing the action of primaquine. What would the instructor include?
e. It disrupts the mitochondria, killing the gametocytes
SAS 7

1. Patient X has genital herpes and is using antiviral creams for her condition. Which of the following is a potential side
effect of the medication?
A. Vulvitis
2. A 15 y/o patient came in for a severe case of respiratory flu and would need drug therapy. Which of the following drugs
should the nurse anticipate to be prescribed?
B. Amantadine (Symmetrel)
3. Which of the following antiviral agents is effective as treatment for H1N1?
C. Oseltamivir
4. A student nurse administering acyclovir was asked by the clinical instructor on how the drug works. The student nurse
would be correct by stating that this drug works in which way?
B. It competes with viral receptors found in the host cells
5. Which of the following prescribed antiviral should the nurse question for a patient with severe hypokalemia?
A. Foscarnet
6. Which enzyme is essential for the maturation of infectious viruses?
C. Protease
7. A senior nurse would be correct to advise her colleague to monitor which parameter in patients receiving cyclosporine
and zidovudine at the same time?
a. Skin lesion, temperature, and color
8. The only protease inhibitor that is not teratogenic.
a. Darunavir
9. Which of the following physical assessment findings will alert the nurse for anthelmintic drug toxicity in elderly patients?
a. Muscle strength +2
10. Which drug is indicated for threadworm infections?
d. mebendazole

SAS 8
1. Chemotherapy dosing is usually based on the total body surface. What should the nurse do before administer
chemotherapy?
c. Weigh and measure the height of the patient on the day of administration.
2. 2. The nurse is caring for of a client who is receiving a chemotherapy. Which of the following would be expected as a
result of the massive cell destruction that occurred from the chemotherapy?
d. Hyperuricemia.
3. Mitomycin (Mutamycin) is prescribed to a client with colorectal cancer. All of which are the routes of administration,
except?
a. Oral
4. The client with a testicular cancer is being treated with Etoposide (Etopophos). Which of the following side effect is
specifically associated with this medication?
d. Orthostatic hypotension
5. The clinic nurse provides instructions to a client receiving an antineoplastic medication. When implementing the plan,
the nurse tells the client to?
d. To consult with health care providers before receiving immunization.
6. The nurse is providing teachings to a client receiving cyclophosphamide (Cytoxan). The nurse tells the client to which of
the following?
b. Increase fluid intake to 2-3 liters per day.
7. The nurse is monitoring a patient receiving an Iv infusion of an antineoplastic medication. During the infusion, the
patient complains of pain and redness at the insertion site. The nurse should take which actions? Select all that apply.

c. Notify the physician.

d. Apply a heat or ice compress to the site.

e. Administer an antidote as prescribed.


8. The nurse is caring for a client receiving chemotherapy when an anaphylactic reaction occurs from the medication. The
nurse should take which actions? Select all that apply.
a. Stop the medication. c. Administer Oxygen. d. Administer epinephrine.
9. The nurse is caring for a client who is receiving Fluorouracil (Adrucil). Which of the following symptoms would
necessitate the nurse to immediate discontinuation of the medication?
b. Ataxia.
10. Nausea and vomiting are frequent adverse effects associated with antineoplastic therapy. What should the nurse advise
clients experiencing these unpleasant adverse effects?
c. Try to maintain hydration and nutrition, which are very important during treatment.

SAS 9
1. As a well-rounded health care provider, you know that corticosteroid therapy is indicated in all of the following conditions
except:
a. Osteoarthritis b. Rheumatoid arthritis c. Systemic lupus erythematosus d. Acute spinal cord injury
2. An appropriate nursing diagnosis for clients who are taking NSAIDs and anticoagulants would be which of the following?
a. Risk for injury related to prolonged bleeding time, inhibition of platelet aggregation, and increased risk of GI bleeding
3. Teaching has been adequate when a client being treated with acetylsalicylic acid states:
a. “I can crush the pills before i swallow them.” b. “I should take the pills with antacids.” c. “Taking the pills on an empty
stomach will help absorption.” d. “If the pills smell like vinegar, I should throw them out.”
4. Which of the following groups of clients are most at risk for GI bleeding from the use of NSAIDs?
a. Clients with dysmenorrhea b. Clients with headaches c. Clients with arthritis d. Clients with renal failure
5. Which of the following NSAIDs is used to prevent thrombosis? a. Motrin b. Toradol c. Aspirin d. Naproxen
6. To minimize the risk of dyspnea and GI bleeding, OTC ibuprofen is given:
a. IV b. With orange juice c. On an empty stomach d. With meals
7. Nurse Kate is taking care of patients taking ibuprofen. Which of the following should be included in her assessment and
monitoring?
a. Blood pressure and bowel sounds b. Weight and appetite c. Muscle strength and range of motion d. Respiratory rate,
depth, and rhythm \
8. Upon checking the medication chart, Nurse Mike found out that his patient is taking both acetaminophen and furosemide.
Which of the following interventions is an appropriate nursing action for these two drugs?
a. Administer medications as they are because they enhance drug actions. b. Measure patient’s intake and output closely. c.
Arrange for SGPT monitoring. d. Assess for signs and symptoms of bleeding.
9. Which of the following patient manifests acetaminophen toxicity?
a. Marie, 26, with new-onset jaundice while on drug therapy b. Eva, 54, with difficulty of breathing while on drug therapy c.
Steven, 37, with joint stiffness while on drug therapy d. Robert, 29, with new-onset glaucoma while on drug therapy
10. Which enzyme is found in active sites of trauma or injury?
a. Streptokinase b. COX-1 c. alpha-ketoglutarate d. COX-2
11. The patient asked the nurse of how soon pain relief will be achieved after a dose of intravenous NSAIDs. The nurse would be
correct by answering:
a. 5-10 minutes depending on the site involved b. at the start of infusion c. 30 minutes since pain is usually extensive d. at
the first hour of infusion
12. Which of the following NSAID pairing is correct?
a. propionic acid: indomethacin b. acetic acid: ibuprofen c. oxicam derivatives: naproxen d. fenamates: mefenamic acid
13. Which of the following analgesic medications should be used with caution in patients with hepatic disease?
a. Naproxen b. Keterolac c. Ibuprofen d. Acetaminofen
14. . Which of the following analgesics is available in “lollipop” form? a. Fentanyl b. Tramadol c. Hydrocodone d. Oxycodone
15. Which of the following is an antidote to opioid overdose?
a. Glucagon b. Naloxone c. Flumazenil d. None of the above
SAS 10

1. The nurse is discussing vaccines with the mother of a 4-year-old child who attends a day care center that requires the DTaP
vaccine. The mother, who is pregnant, tells the nurse that she does not want her child to receive the pertussis vaccine
because she has heard that the disease is "not that serious" in older children. What information will the nurse include when
discussing this with the mother?
b. If the 4-year-old child contracts pertussis, it can be passed on to her newborn

2. A woman who is pregnant tells the nurse she has not had any vaccines but wants to begin so she can protect her unborn child.
Which vaccine(s) may be administered to this patient?

b. Trivalent influenza vaccine b. Trivalent influenza vaccine

3. A 4-year-old child is receiving amoxicillin (Amoxil) to treat otitis media and is in the clinic for a well-child checkup on the last day of
antibiotic therapy. The provider orders varicella (Varivax); mumps, measles, and rubella (MMR); inactivated polio (IPV); and
diphtheria, tetanus, and acellular pertussis (DTaP) vaccines to be given. Which action by the nurse is correct?

a. Administer the vaccines as ordered.

4. A 48-month-old child is scheduled to receive the following vaccines: MMR, Varivax, IPV, and DTaP. The child's parents want the
child to receive two vaccines today and the other two in 1 week. To accommodate the parents' wishes, the nurse will administer

d. MMR and Varivax today and the DTaP and IPV in 1 week.

5. The nurse is preparing to administer rotavirus vaccine to a 4-month-old infant. The nurse notes that the infant received Rotarix
vaccine at 2 months of age. The nurse will plan to administer

a. Rotarix today.

6. Which is an example of acquired passive immunity?

a. Administration of IgG to an unimmunized person exposed to a disease

7. The parent of a 12-month-old child who has received the MMR, Varivax, and hepatitis A vaccines calls the clinic to report redness
and swelling at the vaccine injection sites and a temperature of 100.3° F. The nurse will perform which action?

b. Recommend acetaminophen and cold compresses

8. A mother calls the pediatric clinic to ask when her daughter will receive the Varicella vaccine. Your answer to her question is:

b. at 12 months and 4-6 years

9. When should a child receive the first dose of the Hepatitis B vaccine?

a. Birth

10.A 12-month-old receives a series of vaccinations which includes the Hepatitis A vaccine. When should the child receive the 2nd
dose of this vaccine b. At the 18-month-old visit

11.A 4-year-old is scheduled for routine immunizations. As the nurse you know the physician will most likely order what
vaccinations? Select all that applies

a. DTaP (diphtheria, Tetanus, Pertussis) b. Polio e. MMR (Measles, Mumps, Rubella) g. Varicella

12.A parent has a question about the Rotavirus vaccine and when it is administered. As the nurse you know that ________ doses are
given, and the last dose is given at ________?

d. 3; 6 months

13.Bacterial meningitis strikes babies more often than any other age group. Which vaccine will help prevent one previously common
type of meningitis?
b. Hib

14.You’re providing a free educational clinic to new moms about immunizations. You inform the attendees that the Measles, Mumps,
and Rubella (MMR) vaccine is given?

b. 12 months and 4-6 years

15.A pregnant woman passes antibodies to her unborn baby through the placenta to protect against certain diseases. About how
long does this natural immunity last after birth?

d. None of the above


SAS 11

1. A client diagnosed with schizophrenia is slow to respond and appears to be listening to unseen others. Which medication
should a nurse expect a physician to order to address this type of symptom?
A. Haloperidol (Haldol) to address the negative symptom
B. Clonazepam (Klonopin) to address the positive symptom
C. Risperidone (Risperdal) to address the positive symptom
D. Clozapine (Clozaril) to address the negative symptom
2. A client is diagnosed with schizophrenia. A physician orders haloperidol (Haldol) 50 mg bid, benztropine (Cogentin) 1 mg prn,
and zolpidem (Ambien) 10 mg HS. Which client behavior would warrant the nurse to administer benztropine?
A. Tactile hallucinations
B. Tardive dyskinesia
C. Restlessness and muscle rigidity
D. Reports of hearing disturbing voices
3. A client diagnosed with chronic schizophrenia presents in an emergency department (ED) with uncontrollable tongue
movements, stiff neck, and difficulty swallowing. The nurse would expect the physician to recognize which condition and
implement which treatment?
A. Neuroleptic malignant syndrome and treat by discontinuing antipsychotic medications
B. Agranulocytosis and treat by administration of clozapine (Clozaril)
C. Extrapyramidal symptoms and treat by administration of benztropine (Cogentin)
D. Tardive dyskinesia and treat by discontinuing antipsychotic medications
4. After taking chlorpromazine (Thorazine) for 1 month, a client presents to an emergency department (ED) with severe muscle
rigidity, tachycardia, and a temperature of 105oF (40.5C). The nurse expects the physician to recognize which condition and
implement which treatment?
A. Neuroleptic malignant syndrome and treat by discontinuing Thorazine and administering dantrolene (Dantrium)
B. Neuroleptic malignant syndrome and treat by increasing Thorazine dosage and administering an antianxiety medication
C. Dystonia and treat by administering trihexyphenidyl (Artane)
D. Dystonia and treat by administering bromocriptine (Parlodel)
5. A client diagnosed with schizophrenia takes an antipsychotic agent daily. Which assessment finding should a nurse immediately
report to the client's attending psychiatrist?
A. Respirations of 22 beats/minute
B. Weight gain of 8 pounds in 2 months
C. Temperature of 104F (40C)
D. Excessive salivation
6. An aging client diagnosed with chronic schizophrenia takes an antipsychotic and a beta-adrenergic blocking agent (propranolol)
for hypertension. Understanding the combined side effects of these drugs, which statement by a nurse is most appropriate?
A. "Make sure you concentrate on taking slow, deep, cleansing breaths."
B. "Watch your diet and try to engage in some regular physical activity."
C. "Rise slowly when you change position from lying to sitting or sitting to standing."
D. "Wear sunscreen and try to avoid midday sun exposure."
7. A client diagnosed with schizophrenia is prescribed clozapine (Clozaril). Which client symptoms related to the side effects of this
medication should prompt a nurse to intervene immediately?
A. Sore throat, fever, and malaise
B. Akathisia and hypersalivation
C. Akinesia and insomnia
D. Dry mouth and urinary retention
8. If clozapine (Clozaril) therapy is being considered, the nurse should evaluate which laboratory test to establish a baseline for
comparison in order to recognize a potentially life-threatening side effect?
A. White blood cell count
B. Liver function studies
C. Creatinine clearance
D.
SAS 12
1. A secondary amine tricyclic antidepressant is prescribed for a patient. The medication will significantly increase the availability of
which neurotransmitter?
a. GABA
b. Glutamate
c. Serotonin
d. Norepinephrine
ANSWER: ___D_____
RATIO: Norepinephrine, also called noradrenaline or noradrenalin, is an organic chemical in the catecholamine
family that functions in the brain and body as a hormone and neurotransmitter

2. Nortriptyline (Aventyl) was prescribed for a 68-year-old patient diagnosed with depression and insomnia. Benefits specific to use
of nortriptyline would include:
a. Regular bowel movements.
b. Improved sleep pattern.
c. Weight loss.
d. Anhedonia.
ANSWER: _____B___
RATIO:This medication is used to treat mental/mood problems such as depression. It may help improve mood
and feelings of well-being, relieve anxiety and tension, and increase your energy level. This medication
belongs to a class of medications called tricyclic antidepressants.

3. Priority assessments for a patient beginning amitriptyline (Elavil) include:


a. Nausea, diarrhea, and irritability.
b. Seizures, agranulocytosis, and insomnia.
c. Headache, sexual dysfunction, and weight loss.
d. History of falls, arrhythmias, and blurred vision.
ANSWER: ___D_____
RATIO:Tricyclic antidepressants (TCAs) produce anticholinergic (blurred vision) and antiadrenergic (orthostatic hypotension leading to
falls and arrhythmias) side effects. Gastrointestinal symptoms, sexual dysfunction, and weight variations are related to selective
serotonin reuptake inhibitor (SSRI) therapy. Seizures are seen with bupropion therapy. Agranulocytosis is not considered a major
problem with any of the groups of antidepressants.

4. A 75-year-old patient with a long history of depression begins amitriptyline (Elavil) 100 mg/day. The patient also takes a diuretic
daily for hypertension. The highest priority nursing diagnosis is risk for:
a. Falls related to dizziness and orthostatic hypotension.
b. Ineffective thermoregulation related to anhidrosis.
c. Infection related to suppressed white blood cell count.
d. Constipation related to slowed peristalsis.
ANSWER: ___A_____
RATIO:Amitriptyline is a TCA that has a high risk of producing orthostatic hypotension. The patient is placed at
even greater risk because of older age and diuretic therapy, which reduces fluid volume. The other options
are either unassociated or are remote possibilities.

5. A depressed patient prepares for discharge. The patient is prescribed desipramine (Norpramin) and will have outpatient visits. The
patient reports, "They gave me only a 1-week supply of my medicine." Select the nurse's best reply.
a. "Federal law limits the amount you may be given at any one time."
b. "It will save you money if the drug doesn't work well for your symptoms."
c. "This is a way of ensuring that you will come in for your follow-up appointment."
d. "Prescribing a small amount of drug addresses our concerns for your continuing safety."
ANSWER: ___D_____
RATIO:Desipramine is an activating antidepressant, and it might provide a patient who has suicidal ideation with
the energy to make an attempt. Because the therapeutic dose and lethal dose are not widely separated,
TCA overdose is an often-used suicide plan. Because desipramine appears to be the most toxic TCA,
prescribing only a 7-day supply limits the possibility of using the drug in a suicide attempt. The other
options are either less relevant or incorrect.

6. Evaluation of a patient's response after 1 week of tricyclic antidepressant therapy would be expected to show:
a. No change in objective or subjective symptoms.
b. Increased appetite and weight gain.
c. Decreased suicidal ideation.
d. Improved mood and affect.
ANSWER: ___B_____
RATIO:Improved appetite might be caused by the antihistaminic effect of the drug. Improvement in mood
might not occur for 2 to 4 weeks. Suicidal ideation might not improve for 2 to 4 weeks.

7. A patient diagnosed with depression who has reported vague suicidal ideation will stay at home, have close family supervision,
and make weekly visits to the health care provider. Bupropion (Wellbutrin) is prescribed. What is the benefit of bupropion in this
scenario?
a. It has antianxiety properties as well as antidepressant effects.
b. It lowers the seizure threshold to a lesser extent than TCAs.
c. There is reduced potential for lethal overdose.
d. It stimulates appetite and weight gain.
ANSWER: __C______
RATIO:Bupropion has no lethal-overdose potential, making it well suited for use in outpatient treatment of
depression. Bupropion has a narrow therapeutic index but is far less lethal than TCAs or monoamine
oxidase inhibitors (MAOIs). However, it might cause agitation, anxiety, seizures, anorexia, and weight loss.

8. A patient with depression has taken an SSRI for 1 month. The nurse should use direct questions to evaluate which potential side
effect?
a. Aggressive impulses
b. Sexual dysfunction
c. Paranoid delusions
d. Weight gain
ANSWER: __B______
RATIO: SSRIs often produce sexual dysfunction, such as decreased libido. Patients readily tell nurses about
anticholinergic, gastrointestinal, and other side effects, but are not as forthcoming in reporting sexual
problems. The nurse might need to ask directly to elicit this information. Patients are more willing to
discuss the other options.

9. A patient prescribed fluoxetine (Prozac) is being changed to tranylcypromine (Parnate). The nurse schedules a time lapse between
the last dose of fluoxetine and the first dose of tranylcypromine of at least:
a. 7 days.
b. 14 days.
c. 3 weeks.
d. 6 weeks.
ANSWER: ___D_____
RATIO:Drugs that have a high probability for serious interactions (e.g., MAOIs) will need to be withheld for up to
6 weeks or more as fluoxetine is washing out of the system. The remaining options are too short an Interval.

10. A patient tells the nurse that he stopped taking sertraline (Zoloft) because the drug made him impotent. The nurse can be most
helpful by saying:
a. "Your doctor wants you to continue taking your medication."
b. "Have you talked with your therapist regarding your feelings about sex?"
c. "Let's talk with your doctor. Changing your medication might be a possibility."
d. "Our priority is to treat your depression. Impotence can be addressed in a few weeks."
ANSWER: ___C_____
RATIO:SSRIs commonly cause sexual dysfunction. Sertraline is a widely marketed SSRI and was the second drug
of this class to be used in the United States. Sertraline can also be given once daily (morning or evening)
with or without food. Sertraline causes sexual dysfunction in men and women. Sexual function typically
returns to normal 2 to 3 days after drug cessation. Changing to another type of antidepressant or adding
bupropion in small doses can be helpful. The other options are not compassionate or therapeutic.

11. A client is to receive a dose of fluphenazine hydrochloride (Prolixin) by intramuscular injection. What is the most important
nursing intervention related to the injection?
a. Massage the site vigorously after injection.
b. Administer the drug using Z-tracking.
c. Avoid rotating the injection sites.
d. Select a 22- to 23-gauge needle.
ANSWER: ___B_____
RATIO:The Z-track method for all I.M. injections in adults. By leaving a zigzag path that seals the needle track,
this technique prevents drug leakage into the subcutaneous tissue, helps seal the drug in the muscle,
and minimizes skin irritation.

12. The client has been diagnosed with schizophrenia and is exhibiting a loss of function and motivation. The nurse recognizes that
these symptoms are categorized as:
a. Positive.
b. Paranoiac
c. Negative
d. Incoherent.
ANSWER: ___C_____
RATIO: Negative symptoms include blunting of affect, poverty of speech and thought, apathy, anhedonia, reduced social drive, loss of
motivation, lack of social interest, and inattention to social or cognitive input.

13. What is a common side effect for which the nurse must monitor during administration of both phenothiazine and non-
phenothiazine medications?
a. Hypertension
b. Renal failure
c. Increase in number of white blood cells
d. Extrapyramidal symptoms
ANSWER: ____D____
RATIO: Extrapyramidal symptoms, also called drug-induced movement disorders, describe the side effects caused
by certain antipsychotic and other drugs. These side effects include: involuntary or uncontrollable
movements. tremors.

14. The client with a major depressive disorder taking the selective serotonin reuptake inhibitor (SSRI) fluoxetine (Prozac) calls the
psychiatric clinic and reports feeling confused and restless and having an elevated temperature. Which action should the psychiatric
nurse take?
a. Determine if the client has flulike symptoms
b. Instruct the client to stop taking the SSRI
c. Recommend the client take the medication at night.
d. Explain that these are expected side effects
ANSWER: ___B_____
RATIO: Serotonin syndrome is a serious complication of SSRIs that produces mental changes (confusion, anxiety, and restlessness),
hypertension, tremors, sweating, hyperpyrexia (elevated temperature), and ataxia. Conservation treatment includes stopping the
SSRI and supportive treatment. If untreated, ESE can lead to death

15. The client diagnosed with depression is prescribed phenelzine (Nardil), a monoamine oxidase (MAO) inhibitor. Which statement
by the client indicates to the nurse the medication teaching is effective?
a. "I am taking the herb ginseng to help my attention span"
b. "I drink extra fluids, especially coffee and iced tea"
c. "I am eating three well-balanced meals a day"
d. "At a family cookout I had chicken instead of a hotdog"

SAS 13
1. A nurse providing instructions to a client is taking Doxapram (Dopram). Which of the following statements made by the client
needs further instructions?
A. “I need to take the medication before meals”.
B. “I need to take the medication at bedtime”.
C. “I need to avoid drinking coffee“.
D. “I will not chew or crush long acting form of the medications”.
ANSWER: ____C____
RATIO: Administered intravenously, doxapram stimulates an increase in tidal volume, and respiratory rate. The patient must really
avoid drinking coffee to prevent having palpitations while the medication is being administered.

2. Narcolepsy can be best explained as:


A. A sudden muscle weakness during exercise
B. Stopping breathing for short intervals during sleep
C. Frequent awakenings during the night
D. An overwhelming wave of sleepiness and falling asleep
ANSWER: ___B_____
RATIO: Narcolepsy is best associated with sleep apnea, that, is a potentially serious sleep disorder in which breathing repeatedly
stops and starts.

3. Older adults who take long-acting sedatives or hypnotics are likely to experience:
A. Hallucinations
B. Ataxia
C. Alertness
D. Dyspnea
ANSWER: __b______
RATIO: Many hypnotics with long half lives have serious hangover effects, including drowsiness, unsteady gait, slurred speech and
confusion. Ataxia describes a lack of muscle control or coordination of voluntary movements, such as walking or picking up objects
which fits the unsteady gait.

4. Which nursing diagnosis is appropriate for a patient who has received a sedative-hypnotic agent?
A. Alteration in tissue perfusion
B. Fluid volume excess
C. Risk for injury
D. Risk for infection
ANSWER: ____C____
RATIO: The Food and Drug Administration (FDA) is advising that rare but serious injuries have happened with certain common
prescription insomnia medicines because of sleep behaviors, including sleepwalking, sleep driving, and engaging in other activities
while not fully awake.

5. A patient is admitted to the emergency department with an overdose of a barbiturate. The nurse immediately prepares to
administer which of the following from the emergency drug cart?
A. Naloxone HCl (Narcan)
B. Activated charcoal
C. Flumazenil (Romazicon)
D. Ipecac syrup
ANSWER: ___A_____
RATIO: A medicine called naloxone (Narcan) may be given if an opiate was part of the mix. This medicine often rapidly restores
consciousness and breathing, but its action is short-lived, and may need to be given repeatedly. There is no direct antidote for
barbiturates

6. During patient teaching, the nurse explains the difference between a sedative and hypnotic by stating:
A. “Sedatives are much stronger than hypnotic drugs and should only be used for short periods of time.”
B. “Sedative drugs induce sleep, whereas hypnotic drugs induce a state of hypnosis.”
C. “Most drugs produce sedation at low doses and sleep (the hypnotic effect) at higher doses.”
D. “There really is no difference; the terms are used interchangeably.”
ANSWER: ___B_____
RATIO:A sedative drug decreases activity, moderates excitement, and calms the recipient, whereas a hypnotic drug produces
drowsiness and facilitates the onset and maintenance of a state of sleep that resembles natural sleep in its electroencephalographic
characteristics and from which the recipient can be aroused easily
7. The patient’s chart notes the administration of dantrolene (Dantrium) immediately postoperatively. The nurse suspects that the
patient experienced:
A. Delirium tremens
B. Malignant hyperthermia
C. A tonic-clonic seizure
D. Respiratory arrest
ANSWER: _____C___
RATIO: Tonic-clonic seizures involve both tonic (stiffening) and clonic (twitching or jerking) phases of muscle activity. Tonic-clonic
seizures may start with a simple partial seizure or aura. The person may experience changes in sensation, mood or emotion leading
up to the tonic-clonic seizure. Dantrolene is a muscle relaxer that is used to treat muscle spasticity (stiffness and spasms) caused by
conditions such as a spinal cord injury, stroke, cerebral palsy, or multiple sclerosis.
8. Which of the following is an important nursing action for the administration of a benzodiazepine as a sedative-hypnotic agent?
A. Use IM dosage forms for longer duration
B. Administer safely with other CNS depressants for insomnia
C. Monitor geriatric patients for the common occurrence of paradoxical reactions.
D. Evaluate for physical dependence that occurs within 48 hours of beginning the drug.
ANSWER: ____A____
RATIO: Benzodiazepines are commonly administered via the intravenous route. They may also be administered rectally, intranasally,
and intramuscularly, as protocol/patient presentation dictates

9. Pediatric and geriatric patients often react with more sensitivity to CNS depressants. This type of sensitivity manifests itself in the
development of which type of reaction?
A. Idiopathic
B. Teratogenic
C. Paradoxical
D. Psychogenic
ANSWER: ____C____
RATIO: A paradoxical reaction or paradoxical effect is an effect of a chemical substance, mostly a medical drug, opposite to the
effect which would normally be expected

10. Which of the following is an appropriate nursing intervention for patients who are receiving CNS depressants?
A. Prevent any activity within the hospital setting while on oral muscle relaxants
B. Make sure that the patient knows that sedation should be minimal with these agents.
C. Cardiovascular stimulation, a common side effect, would lead to hypertension
D. Make sure the patient’s call light is close by in case of the need for assistance with activities.
ANSWER: ___B_____
RATIO: Sedation is a drug-induced state during which patients respond normally to verbal commands. The nurse should educate the
patient about the side effects of this drug.

11. Which of the following conditions characterizes rapid eye movement (REM) sleep?
A. Disorientation and disorganized thinking
B. Jerky limb movements and position changes
C. Pulse rate slowed by 5 to 10 beats/minute
D. Highly active brain and physiological activity levels.
ANSWER: ___D_____
RATIO: During REM sleep, our brain is almost as active as it is when we are awake. In this phase of sleep, breathing can become fast
and irregular. REM sleep is thought to help consolidate memories

12. Abrupt withdrawal of barbiturates can precipitate what medical condition?


A. Ascites
B. Hypertensive crisis
C. Status epilepticus
D. Coma
ANSWER: ____B____
RATIO: Side effects of barbiturates include arrhythmias, cough, headache.
13. What drug refers to the ability to help patient feel calm and unaware of his environment?
A. Anesthesia
B. Sedatives
C. Hypnotics
D. Anxiolytics
ANSWER: _____B___
RATIO: Anxiolytic-hypnotics agents are used to alter an individual's responses to environmental stimuli. These agents are referred to
as anxiolytics (prevent feelings of tension or fear), sedatives (help patient feel calm and unaware of their environment), and
hypnotics (help patients sleep)

14. What is the most commonly used anxiolytics.


A. Benzodiazepines
B. Diphenhydramine
C. Barbiturates
D. Buspirone
ANSWER: ________
RATIO:Benzodiazepines are a class of drugs primarily used for treating anxiety, but they also are effective in treating several other
conditions.

15. Which of the following medical condition(s) can be considered as contraindication(s) to use of anxiolytics and hypnotics?
A. Psychosis
B. Alcoholic intoxication
C. Acute gastroenteritis
D. Both A and B

SAS 14
1. A client with chronic pain reports to you, the charge nurse, that the nurse have not been responding to requests for pain
medication. What is your initial action?
A. Check the MARs and nurses’ notes for the past several days.
B. Ask the nurse educator to give an in-service about pain management.
C. Perform a complete pain assessment and history on the client.
D. Have a conference with the nurses responsible for the care of this client.
ANSWER: ___D_____
RATIO: As charge nurse, you must assess for the performance and attitude of the staff in relation to this client. After gathering data
from the nurses, additional information from the records and the client can be obtained as necessary. The educator may be of
assistance if knowledge deficit or need for performance improvement is the problem.

2. Family members are encouraging your client to “tough it out” rather than run the risk of becoming addicted to narcotics. The
client is stoically abiding by the family’s wishes. Priority nursing interventions for this client should target which dimension of
pain?
A. Sensory
B. Affective
C. Sociocultural
D. Behavioral
E. Cognitive
ANSWER: ____C.____
RATIO:The family is part of the sociocultural dimension of pain. They are influencing the client and should be included in the teaching
sessions about the appropriate use of narcotics and about the adverse effects of pain on the healing process. The other dimensions
should be included to help the client/family understand overall treatment plan and pain mechanism

3. A client with diabetic neuropathy reports a burning, electrical-type in the lower extremities that is not responding to NSAIDs.
You anticipate that the physician will order which adjuvant medication for this type of pain?
A. Amitriptyline (Elavil)
B. Corticosteroids
C. Methylphenidate (Ritalin)
D. Lorazepam (Ativan)
ANSWER: ____A.____
RATIO:Antidepressants such as amitriptyline can be given for diabetic neuropathy. Corticosteroids are for pain associated with
inflammation. Methylphenidate is given to counteract sedation if the client is on opioids. Lorazepam is an anxiolytic.

4. Which client is most likely to receive opioids for extended periods of time?
A. A client with fibromyalgia
B. A client with phantom limb pain
C. A client with progressive pancreatic cancer
D. A client with trigeminal neuralgia
ANSWER: ___c._____
RATIO:Cancer pain generally worsens with disease progression and the use of opioids is more generous. Fibromyalgia is more likely to
be treated with non-opioid and adjuvant medications. Trigeminal neuralgia is treated with anti-seizure medications such as
carbamazepine (Tegretol). Phantom limb pain usually subsides after ambulation begins.

5. In caring for a young child with pain, which assessment tool is the most useful?
A. Simple description pain intensity scale
B. 0-10 numeric pain scale
C. Faces pain-rating scale
D. McGill-Melzack pain questionnaire
ANSWER: ___C_____
RATIO: The Faces pain rating scale (depicting smiling, neutral, frowning, crying, etc.) is appropriate for young children who may have
difficulty describing pain or understanding the correlation of pain to numerical or verbal descriptors. The other tools require abstract
reasoning abilities to make analogies and use of advanced vocabulary.

6. In applying the principles of pain treatment, what is the first consideration?


A. Treatment is based on client goals.
B. A multidisciplinary approach is needed.
C. The client must be believed about perceptions of own pain.
D. Drug side effects must be prevented and managed.
ANSWER: ___C._____
RATIO:The client must be believed and his or her experience of pain must be acknowledged as valid. The data gathered via client
reports can then be applied to other options in developing the treatment plan.

7. A client appears upset and tearful, but denies pain and refuses pain medication, because “my sibling is a drug addict and has
ruined out lives.” What is the priority intervention for this client?
A. Encourage expression of fears on past experiences.
B. Provide accurate information about use of pain medication.
C. Explain that addiction is unlikely among acute care clients.
D. Seek family assistance in resolving this problem.
ANSWER: ___A_____
RATIO:This client has strong beliefs and emotions related to the issue of sibling addiction. First, encourage expression. This indicated
to the client that the feelings are real and valid. It is also an opportunity to assess beliefs and fears. Giving facts and information is
appropriate at the right time. Family involvement is important, bearing in mind that their beliefs about drug addiction may be similar
to those of the client.

8. A client is being tapered off opioids and the nurse is watchful for signs of withdrawal. What is one of the first signs of
withdrawal?
A. Fever
B. Nausea
C. Diaphoresis
D. Abdominal cramps
ANSWER: ___C_____
RATIO:Diaphoresis is one of the early signs that occur between 6 and 12 hours. Fever, nausea, and abdominal cramps are late signs
that occur between 48 and 72 hours.

9. The physician has ordered a placebo for a chronic pain client. You are newly hired nurse and you feel very uncomfortable
administering the medication. What is the first action that you should take?
A. Prepare the medication and hand it to the physician
B. Check the hospital policy regarding use of the placebo.
C. Follow a personal code of ethics and refuse to give it.
D. Contact the charge nurse for advice.
ANSWER: ____D.____
RATIO: The charge nurse is a resource person who can help locate and review the policy. If the physician is insistent, he or she could
give the placebo personally, but delaying the administration does not endanger the health or safety of the client. While following
one’s own ethical code is correct, you must ensure that the client is not abandoned and that care continues.

10. For a cognitively impaired client who cannot accurately report pain, what is the first action that you should take?
A. Closely assess for nonverbal signs such as grimacing or rocking.
B. Obtain baseline behavioral indicators from family members.
C. Look at the MAR and chart, to note the time of the last dose and response.
D. Give the maximum PRS dose within the minimum time frame for relief.
ANSWER: ___B_____
RATIO: Complete information from the family should be obtained during the initial comprehensive history and assessment. If this
information is not obtained, the nursing staff will have to rely on observation of nonverbal behavior and careful documentation to
determine pain and relief patterns.

11. Which route of administration is preferable for administration of daily analgesics (if all body systems are functional)?
A. Intravenous
B. IM or subcutaneous
C. Oral
D. Transdermal
E. PCA
ANSWER: ___C_____
RATIO:If the gastrointestinal system is functional, the oral route is preferred for routine analgesics because of lower cost and ease of
administration. Oral route is also less painful and less invasive than the IV, IM, subcutaneous, or PCA routes. Transdermal route is
slower and medication availability is limited compared to oral forms.
12. A first day postoperative client on a PCA pump reports that the pain control is inadequate. What is the first action you should
take?
A. Deliver the bolus dose per standing order.
B. Contact the physician to increase the dose.
C. Try non-pharmacological comfort measures.
D. Assess the pain for location, quality, and intensity.
ANSWER: ____D____
RATIO:Assess the pain for changes in location, quality, and intensity, as well as changes in response to medication. This assessment
will guide the next steps.

13. What is the best way to schedule medication for a client with constant pain?
A. PRN at the client’s request
B. Prior to painful procedures
C. IV bolus after pain assessment
D. Around-the-clock
ANSWER: ___D._____
RATIO: IF the pain is constant, the best schedule is around-the-clock, to provide steady analgesia and pain control. The other options
may actually require higher doses to achieve control

14. Which of the following medications is contraindicated with acetaminophen?


A. Percocet
B. Furosemide
C. Phenytoin
D. Ampicillin
ANSWER: ___A_____
RATIO: Percocet contains acetaminophen, and the two medications
should not be administered together

15. What is the antidote for narcotic overdose.


A. Naloxone
B. Nubain
C. Morphine
D. Codeine

SAS 15

1. It is a rare type of seizure that appears sporadically.


A. Febrile seizure
B. Myoclonic seizure
C. Jacksonian seizure
D. Absence seizure
ANSWER: ___B_____
RATIO: It is characterized by short, sporadic periods of muscle contractions that last for several minutes. It is relatively rare.

2. Which antiseizure agents is used to treat absence seizures?


A. Hydantoins
B. Benzodiazepines
C. Barbiturates
D. Succinimides
ANSWER: ____D____
RATIO: Most frequently used to treat absence seizures and reduction of frequency of attacks

3. A patient on Dilantin began showing bruises and signs of active infection. The nurse would be right to anticipate that these
manifestations give clue to ________________.
A. Drug underdosage
B. Hypersensitivity reactions
C. Cellular toxicity
D. None of the above
ANSWER: ___c_____
RATIO:_______It is characterized by severe liver toxicity, bone marrow suppression, gingival hyperplasia, and serious dermatological
reaction (e.g. hirsutism, Steven-Johnson
syndrome).___________________________________________________________________________________________________
____________________________________________________________________________________________________________
_______________________________________________________________________
4. Which barbiturate is also indicated for management of anxiety?
A. mephobarbital
B. primidone
C. phenobarbital
D. mysoline
ANSWER: ____a____
RATIO:Aside for being used as treatment for tonic-clonic and absence seizures, mephobarbital is also used as an anxiolytic or
hypnotic
agent._______________________________________________________________________________________________________
____________________________________________________________________________________________________________
__________________________________________________________________________
5. Benzodiazepines exert their effect primarily on which neurotransmitter?
A. GABA
B. Serotonin
C. Norepinephrine
D. Both A and B
ANSWER: ___A_____
RATIO:

Potentiate effects of GABA (inhibitory neurotransmitter that stabilizes nerve cell membranes).
Act primarily in the limbic system and RAS so it can also cause muscle relaxation and relief of anxiety without substantially affecting
the functions of the cortex.

6. The client is prescribed phenytoin (Dilantin), an anticonvulsant, for a seizure disorder. Which statement indicates the client
understands the discharge teaching concerning this medication?
A. "I will brush my teeth after every meal."
B. "I will check my Dilantin level daily."
C. "My urine will turn orange while on Dilantin."
D. "I won't have any seizures while on this medication."
ANSWER: ____A____
RATIO:1. Thorough oral hygiene after each
meal, gum massage, daily flossing,
and regular dental care are essential
to prevent or control gingival
hyperplasia, which is a common
occurrence in clients taking
Dilantin.
2. A serum (venipuncture) Dilantin level is
checked monthly at first and then, after a
therapeutic level is attained, every six
(6) months.
3. Dilantin does not turn the urine orange.
4. The use of Dilantin does not ensure that
the client will not have any seizures, and
in some instances, the dosage may need to
be adjusted or another medication may
need to be used.

7. The client is admitted to the intensive care department (ICD) experiencing status
epilepticus. Which collaborative intervention should the nurse anticipate?
A. Assess the client's neurological status every hour.
B. Monitor the client's heart rhythm via telemetry.
C. Administer an anticonvulsant medication by intravenous push.
D. Prepare to administer a glucocorticosteroid orally.
ANSWER: __c______
RATIO:__ Administering an anticonvulsant
medication by intravenous push
requires the nurse to have an order or
confer with another member of the
health-care team.
____________________________________________________________________________________________________________
____________________________________________________________________________________________________________
___________________________________________________________________
8. The client has been newly diagnosed with epilepsy. Which discharge instructions should be taught to the client? Select all that
apply.
A. Keep a record of seizure activity.
B. Take tub baths only; do not take showers.
C. Avoid over-the-counter medications.
D. Have anticonvulsant medication serum levels checked regularly.
E. Do not drive alone; have someone in the car.
ANSWER: a c d________
RATIO:1. Keeping a seizure and medication chart
will be helpful when keeping follow-up
appointments with the health-care
provider and in identifying activities
that may trigger a seizure.
3. Over-the-counter medications may
contain ingredients that will interact
with antiseizure medications or, in
some cases, as with use of stimulants,
possibly cause a seizure.
4. Most of the anticonvulsant medications
have therapeutic serum levels that
should be maintained, and regular
checks of the serum levels help to
ensure the correct
level.________________________________________________________________________________________________________
____________________________________________________________________________________________________________
_________________________________________________________________________
9. The nurse asks the male client with epilepsy if he has auras with his seizures. The
client says, "I don't know what you mean. What are auras?" Which statement by the
nurse would be the best response?
A. "Some people have a warning that the seizure is about to start."
B. "Auras occur when you are physically and psychologically exhausted."
C. "You're concerned that you do not have auras before your seizures?"
D. "Auras usually cause you to be sleepy after you have a seizure."
ANSWER: __A______
RATIO:_. An aura is a visual, auditory, or
olfactory occurrence that takes place
prior to a seizure and warns the client
a seizure is about to occur. The aura
often allows time for the client to lie
down on the floor or find a safe place
to have the seizure.
____________________________________________________________________________________________________________
____________________________________________________________________________________________________________
____________________________________________________________________
10. The male client is sitting in the chair and his entire body is rigid with his arms and legs contracting and relaxing. The client is not
aware of what is going on and is making guttural sounds. Which action should the nurse implement first?
A. Push aside any furniture.
B. Place the client on his side.
C. Assess the client's vital signs.
D. Ease the client to the floor.
ANSWER: __D______
RATIO:_4. The client should not remain in the
chair during a seizure. He should be
brought safely to the floor so that he
will have room to move the extremities.
____________________________________________________________________________________________________________
____________________________________________________________________________________________________________
____________________________________________________________________

11. The nurse is giving instructions to a client receiving Phenytoin (Dilantin). The nurse concludes that the client has a sufficient
knowledge if the client states that:
A. “Wearing a medical alert tag is not required”.
B. “Alcohol is permitted in while taking this medication”.
C. “I can take the medicine with milk”.
D. “Have the serum phenytoin level taken before giving the medication”.
ANSWER: _D_______
RATIO:Taking the prescribed daily dosage to keep the blood level of the drug constant and having a sample drawn for serum drug
level before taking the morning dose. Option A: Wearing a medical tag allows any medical care provider to know that the client is on
seizure medication. Option B: Alcohol use can increase the blood levels of phenytoin and may increase side effects. Option C: Taking
it with milk will impair the
absorption.___________________________________________________________________________________________________
____________________________________________________________________________________________________________
______________________________________________________________________________
12. A 17-year-old client is taking Phenytoin (Dilantin) for the treatment of seizures. Phenytoin blood level reveals to be 25 mcg/ml.
Which of the following symptoms would be expected as a result of the laboratory result?
A. No symptoms, because the value is within the normal range.
B. Hypoglycemia.
C. Tachycardia.
D. Nystagmus.
ANSWER: ___D_____
RATIO:The therapeutic level of phenytoin is 10-20mcg/ml. Higher than 20mcg/ml results in nystagmus (Involuntary eye
movement).__________________________________________________________________________________________________
____________________________________________________________________________________________________________
_______________________________________________________________________________
13. A client with tonic-clonic seizure is receiving Phenobarbital (Luminal) and Valproic acid (Depakene). The nurse tells the client that:
A. Valproic acid decreases phenobarbital metabolism.
B. Valproic acid increases phenobarbital metabolism.
C. There is no interaction between the two.
D. Increase the dosage of the two medications.
ANSWER: ________
RATIO: Valproic acid is not enzyme inducer, but it may cause clinically relevant drug interactions by inhibiting the metabolism of
selected substrates, most notably phenobarbital and lamotrigine

14. The patient is receiving IV antiepileptic therapy. Which parenteral antiepileptic drug is effective against generalized seizures?
A. Phenytoin (Dilantin)
B. Fosphenytoin (Cerebyx)
C. Phenobarbital (Luminal)
D. Valproic acid (Depakene)
ANSWER: ________
RATIO: In 1940, phenytoin (PHT) was found to be an effective drug for the treatment of epilepsy, and since then it has become a
major first-line antiepileptic drug (AED) in the treatment of partial and secondarily generalized seizures____

15. What information should the nurse provide to a patient who will self-administer an antiepileptic agent for the first time at home?
A. Report any vision or hearing problems with levetiracetam (Keppra).
B. Stir phenobarbital (Luminal) elixir into a cold carbonated beverage.
C. Have a complete blood count monthly while taking carbamazepine (Tegretol).
D. Administer pregabalin (Lyrica) every morning on an empty stomach.

SAS 16

1. A client with myasthenia gravis has become frequently complaining of weakness and fatigue. The physician plans to identify
whether the client is responding to an overdose of the medication or a worsening of the disease. A tensilon test is performed. Which
of the following would indicate that the client is experiencing an overdose of the medication?
A. Temporarily worsening of the condition.
B. Improvement of the weakness and fatigue.
C. No change in the condition.
D. Complaints of muscle spasms.
ANSWER: ____A____
RATIO:Tensilon test/ injection of edrophonium is performed to diagnose cholinergic crisis (overdose with anticholinesterase) or
myasthenic crisis (under medication). A tensilon injection makes the client in cholinergic crisis temporarily worse (negative tensilon
test). An improvement in the weakness indicates myasthenia crisis.

2. The diagnostic work-up of a client hospitalized with complaints of progressive weakness and fatigue confirms a diagnosis of
myasthenia gravis. The medication used to treat myasthenia gravis is:
A. Prostigmine (neostigmine)
B. Atropine (atropine sulfate)
C. Didronel (etidronate)
D. Tensilon (edrophonium)
ANSWER: __A______
RATIO:Neostigmine is a medication used to treat myasthenia gravis, Ogilvie syndrome, and urinary retention without the presence of
a blockage. It is also used together with atropine to end the effects of neuromuscular blocking medication of the non-depolarizing
type.

3. Karina a client with myasthenia gravis is to receive immunosuppressive therapy. The nurse understands that this therapy is
effective because it:
A. Promotes the removal of antibodies that impair the transmission of impulses
B. Stimulates the production of acetylcholine at the neuromuscular junction.
C. Decreases the production of autoantibodies that attack the acetylcholine receptors.
D. Inhibits the breakdown of acetylcholine at the neuromuscular junction.
ANSWER: ________
RATIO:
Steroids decrease the body's immune response thus decreasing the production of antibodies that attack the acetylcholine receptors
at the neuromuscular junction
4. Myasthenia gravis is due to ____ receptors being blocked and destroyed by antibodies.
A. Epinephrine
B. Nicotinic
C. Acetylcholine
D. Transient
ANSWER: ________
RATIO:Acetylcholine receptor antibody is a protein found in the blood of many people with myasthenia gravis. The antibody affects
a chemical that sends signals from nerves to muscles and between nerves in the brain. This article discusses the blood test for
acetylcholine receptor antibody.

5. A client with myasthenia gravis has been receiving Neostigmine (Prostigmin). This drug acts by:
A. Stimulating the cerebral cortex
B. Blocking the action of cholinesterase
C. Replacing deficient neurotransmitters
D. Accelerating transmission along neural swaths
ANSWER: ________
RATIO:Neostigmine inhibits the hydrolysis of acetylcholine by competing with acetylcholine for attachment to acetylcholinesterase at
sites of cholinergic transmission. It enhances cholinergic action by facilitating the transmission of impulses across neuromuscular
junctions

6. The most significant initial nursing observations that need to be made about a client with myasthenia include:
A. Ability to chew and speak distinctly
B. Degree of anxiety about her diagnosis
C. Ability to smile an to close her eyelids
D. Respiratory exchange and ability to swallow
ANSWER: ________
RATIO:Muscle weakness can lead to respiratory failure that will require emergency intervention and inability to swallow may lead to
aspiration.

7. In making a diagnosis of myasthenia gravis Edrophonium HCI (Tensilon) is used. The nurse knows that this drug will cause a
temporary increase in:
A. Muscle strength
B. Symptoms
C. Blood pressure
D. Consciousness
ANSWER: ____A____
RATIO: Tensilon, an anticholinesterase drug, causes temporary relief of symptoms of myasthenia gravis in client who have the disease
and is therefore an effective diagnostic aid.

8. The nurse is teaching the female client with myasthenia gravis about the prevention of myasthenic and cholinergic crises. The
nurse tells the client that this is most effectively done by:
A. Eating large, well-balanced meals
B. Doing muscle-strengthening exercises
C. Doing all chores early in the day while less fatigued
D. Taking medications on time to maintain therapeutic blood levels
ANSWER: ___D_____
RATIO:Clients with myasthenia gravis are taught to space out activities over the day to conserve energy and restore muscle strength.
Taking medications correctly to maintain blood levels that are not too low or too high is important. Muscle-strengthening exercises
are not helpful and can fatigue the client. Overeating is a cause of exacerbation of symptoms, as is exposure to heat, crowds, erratic
sleep habits, and emotional stress

9. Helen is diagnosed with myasthenia gravis and pyridostigmine bromide (Mestinon) therapy is started. The Mestinon dosage is
frequently changed during the first week. While the dosage is being adjusted, the nurse’s priority intervention is to:
A. Administer the medication exactly on time
B. Administer the medication with food or mild
C. Evaluate the client’s muscle strength hourly after medication
D. Evaluate the client’s emotional side effects between doses
ANSWER: ____C____
RATIO:Peak response occurs 1 hour after administration and lasts up to 8 hours; the response will influence dosage levels.

10. A female client has experienced an episode of myasthenic crisis. The nurse would assess whether the client has precipitating
factors such as:
A. Getting too little exercise
B. Taking excess medication
C. Omitting doses of medication
D. Increasing intake of fatty foods
ANSWER: ________
RATIO:Myasthenic crisis often is caused by undermedication and responds to the administration of cholinergic medications, such as
neostigmine (Prostigmin) and pyridostigmine (Mestinon). Cholinergic crisis (the opposite problem) is caused by excess medication
and responds to withholding of medications. Too little exercise and fatty food intake are incorrect. Overexertion and overeating
possibly could trigger myasthenic crisis.

11. Bryan with a tentative diagnosis of myasthenia gravis is admitted for diagnostic make up. Myasthenia gravis can confirmed by:
A. Kernigs sign
B. Brudzinski’s sign
C. A positive sweat chloride test
D. A positive edrophonium (Tensilon) test
ANSWER: _____D___
RATIO:Antibodies attack their acetylcholine receptors. This prevents muscles from being stimulated and makes muscles easy to tire. A
person tests positive for myasthenia gravis if their muscles get stronger after being injected with Tensilon.

12. All of the following are direct-acting skeletal muscle relaxants, except:
A. Botulinum toxin type A
B. Myobloc
C. Dantrium
D. None of the above
ANSWER: ____a____
RATIOBotulinum toxin type A (Botox Cosmetic), botulinum toxin type B (Myobloc), and dantrolene (Dantrium) are all direct-acting
skeletal muscle relaxants.
13. Which of the following is the priority nursing intervention for children receiving muscle relaxants?
A. Assess bowel sounds and activity.
B. Provide skin care.
C. Administer drug with food.
D. Both A and B.
ANSWER: ________
RATIO:Children are especially at an increased risk of CNS and GI toxicity.

14. What is the centrally acting muscle relaxant of choice for children with tetanus?
A. Botulinum toxin type B
B. Dantrium
C. Baclofen
D. Methocarbamol
ANSWER: ________
RATIO;Methocarbamol is a centrally acting muscle relaxant. It is used in the treatment of muscle spasms resulting from injury,
musculoskeletal disorders, tetanus and other disorders.

15. How long should Baclofen be tapered to prevent development of psychoses and hallucinations?
A. 12-21 days
B. 1-10 days
C. 7-14 days
D. 7-21 days
ANSWER: ________
RATIO:Baclofen is tapered over 1-2 weeks to prevent development of psychoses and hallucinations.
SAS 17

1. A nurse is instructing a client regarding Carbidopa-levodopa (Sinemet) for the treatment of Parkinson’s disease. The nurse tells the
client that which of the following is a side effect of the medication?
A. Difficulty performing a voluntary movement.
B. Increased blood pressure.
C. Increased heart rate.
D. Itchiness of the skin.
ANSWER: ____D____
RATIO: Itchiness of the skin may indicate an allergy to dopaminergics. This is considered to be one of the contraindications and
cautions when taking Carbidopa-levodopa (Sinemet)

2. A nurse is giving dietary instructions to a client receiving levodopa. Which of the following food items should be avoided by the
client?
A. Goat yogurt.
B. Whole grain cereal.
C. Asparagus.
D. Apples
ANSWER: ___B____
RATIO: Protein and levodopa use the same transporter to cross the small intestine wall. Therefore it's possible that dietary protein
can interfere with absorption of levodopa including whole grain cereal which contains 13g of its protein

3. Biperiden hydrochloride (Akineton) is added to a list of antiparkinsonian medications that an elderly client is taking. Which of the
following instructions made by the nurse that needs further learning?
A. To avoid smoking, alcohol and caffeine.
B. To use ice chips, candy or gum for dry mouth.
C. Walking in the morning to have a daily source of direct sunlight.
D. Eating foods rich in fiber and increase fluid intake.
ANSWER: _____B___
RATIO: Dry mouth is considered to be one of the side effects when taking Biperiden hydrochloride (Akineton)

4. What is the goal of therapy for Parkinson’s disease? B.


A. To decrease dopamine and to increase cholinergic neurons
B. To balance dopamine and cholinergic neurons
C. To excite neurons more
D. To inhibit neurons more
ANSWER: _____A____
RATIO: • The goal of the therapy is to restore the balance between decreasing dopamine levels (has inhibitory effect on the neurons
of the basal ganglia) and increasing cholinergic neurons (excitatory).

5. What is the mainstay of treatment for Parkinson’s?


A. Symmetrel
B. Levodopa
C. Cogentin
D. Parlodel
ANSWER: ____B____
RATIO: Levodopa, the precursor of dopamine is the mainstay of treatment for Parkinson’s. It crosses the blood-brain barrier and is
converted into dopamine. When combined with carbidopa, the enzyme dopa decarboxylase is inhibited from metabolizing levodopa,
leading to higher levels that can cross the barrier.

6. The drug of choice in children with parkinsonian symptom.


A. Levodopa
B. Artane
C. Benadryl
D. Benztropine
ANSWER: ____A____
RATIO: Levodopa, the most effective Parkinson's disease medication, is a natural chemical that passes into your brain and is
converted to dopamine. Levodopa is combined with carbidopa (Lodosyn), which protects levodopa from early conversion to
dopamine outside your brain.
7. Which can decrease efficacy of levodopa?
A. Phenytoin
B. Pyridoxine
C. Niacin
D. Both A and B
ANSWER: ____D___
RATIO: Vitamin B6,phenytoin: decreased levodopa efficacy

8. A construction worker for 10 years is about to receive anticholinergics. What should the nurse consider in handling this patient?
A. Do not give the drug to the patient.
B. Administer it with caution.
C. Discuss it with the doctor and have the order changed.
D. Arrange for a possible increase in the dose.
ANSWER: ___B_____
RATIO: Administer drug with caution for patients exposed in hot weather or environments because patients are at increased risk for
heat prostration due to decreased ability to sweat.

9. Which of the following is a contraindication for the use of anticholinergic agents?


A. Heart rate of 120 beats per minute
B. Myasthenia gravis
C. Active hepatitis B
D. All of the above.
ANSWER: ___D_____
RATIO: Myasthenia gravis. Exacerbated by acetylcholine receptor sites blocking effect of the drug • Tachycardia, dysrhythmia, hyper
or hypotension. Blocking the parasympathetic system may cause a dominance of sympathetic stimulatory activity • Hepatic
dysfunction. Interfere with the metabolism of drugs increasing risk of toxicity

10. Edward, a 66-year-old client with slight memory impairment and poor concentration, is diagnosed with primary degenerative
dementia of the Alzheimer’s type. Early signs of this dementia include subtle personality changes and withdrawal from social
interactions. To assess for progression to the middle stage of Alzheimer’s disease, the nurse should observe the client for:
A. Occasional irritable outbursts.
B. Impaired communication.
C. Lack of spontaneity.
D. Inability to perform self-care activities.
ANSWER: ___B_____
RATIO: The nurse should explain simply. Use simple explanations and face-to-face interaction when communicating with client. Do
not shout message into client’s ear. Speaking slowly and in a face-to-face position is most effective when communicating with an
elderly individual experiencing a hearing loss.

11. Nurse Pauline is aware that Dementia, unlike delirium, is characterized by:
A. Slurred speech.
B. Insidious onset.
C. Clouding of consciousness.
D. Sensory perceptual change.
ANSWER: ___C____
RATIO: Delirium is an abrupt change in the brain that causes mental confusion and emotional disruption. It makes it difficult to think,
remember, sleep, pay attention, and more.

12. The nurse is aware that the following ways in vascular dementia different from Alzheimer’s disease is:
A. Vascular dementia has more abrupt onset.
B. The duration of vascular dementia is usually brief.
C. Personality change is common in vascular dementia.
D. The inability to perform motor activities occurs in vascular dementia.
ANSWER: __D______
RATIO: Vascular dementia is a decline in thinking skills caused by conditions that block or
reduce blood flow to various regions of the brain, depriving them of oxygen and
nutrients.
13. A 65 years old client is in the first stage of Alzheimer’s disease. Nurse Patricia should plan to focus this client’s care on:
A. Offering nourishing finger foods to help maintain the client’s nutritional status.
B. Providing emotional support and individual counseling.
C. Monitoring the client to prevent minor illnesses from turning into major problems.
D. Suggesting new activities for the client and family to do together.
ANSWER: __C______
RATIO: Observe client closely. Close observation of client’s behavior is indicated if delusional thinking reveals an intention for
violence. Client safety is a nursing priority.

14. Nurse Kate would expect that a client with vascular dementia would experience:
A. Loss of remote memory related to anoxia.
B. Loss of abstract thinking related to emotional state.
C. Inability to concentrate related to decreased stimuli.
D. Disturbance in recalling recent events related to cerebral hypoxia.
ANSWER: ___A_____
RATIO: The impact of vascular conditions on thinking skills varies widely, depending on the
severity of the blood vessel damage and the part of the brain it affects. Memory loss
may or may not be a significant symptom depending on the specific brain areas where
blood flow is reduced. Vascular damage that starts in the brain areas that play a key
role in storing and retrieving information may cause memory loss that is very similar
to Alzheimer’s disease.

15. The client with Alzheimer’s disease is being assisted with activities of daily living when the nurse notes that the client uses her
toothbrush to brush her hair.
A. Agnosia
B. Apraxia
C. Anomia
D. Aphasia
ANSWER: ____A____

SAS 18

Multiple Choice
1. All of the following are true about dopamine and dobutamine, except:
A. They bind to both alpha- and beta-receptors
B. They can increase heart rate and cause bronchoconstriction
C. Dopamine is the sympathomimetic of choice for shock
D. Administration of these drugs can put patients with peripheral cardiovascular disease into exacerbation
ANSWER: ___B_____
RATIO:Dopamine and dobutamine are alpha- and beta-receptors. They can increase the heart rate and cause bronchodilation as well
as increasing the rate and depth of respirations. They can put patients with peripheral CV disease into exacerbation because of their
vasoconstrictive effects. Lastly, dopamine is the sympathomimetic of choice for shock.

2. The sympathomimetic found in many OTC cold products.


A. Salmeterol
B. Albuterol
C. Phenylephrine
D. Terbutaline
ANSWER: __C______
RATIO:Phenylephrine is often found in OTC allergy and cold preparations so primary caregivers should be instructed to check labels
for ingredients and not combine drugs with similar ingredients.

3. Nurse Jake was doing his usual morning routine care for his patient receiving parenteral isoproterenol when he noticed a
developing extravasation in the site. Which is the best nursing action for this situation?
A. Provide skin care and warm compress because it will subside in 1-2 hours.
B. Explain to the client that it is an expected side effect of the drug.
C. Document and endorse to next nurse on duty because extravasation need long hours of observation before an appropriate course
of nursing actions can be determined.
D. Inject 10 mL of saline with 5 mg of phentolamine to save the area from necrosis.
ANSWER: __D______
RATIO:The vasoconstrictive effects of isoproterenol can cause necrosis. Management for this is infiltrating the site with 10 mL saline
containing 5-10 mg of phentolamine within 12 hours to save the area from necrosis.

4. This drug is used to control hypertension and is used as an epidural infusion for patients suffering from cancer pain.
A. Midodrine
B. Clonidine
C. Albuterol
D. Isoproterenol
ANSWER: ___B_____
RATIOClonidine specifically stimulates alpha2-receptors of the CNS leading to decreased CNS outflow of norepinephrine. Orally and
transdermally, it is used to control hypertension and as an injection, it is for epidural infusion for controlling cancer pain.

5. A pregnant patient on 32 weeks age of gestation came rushing to the emergency complex because of increased watery discharge
and frequent uterine contractions. Upon physical examination, patient was confirmed to have ruptured of membranes and is in
preterm labor. Which of the additional PE findings will render beta-agonists contraindicated as part of her management for uterine
relaxation?
A. HbA1c of 6
B. Blood pressure of 150/70 mmHg and +3 proteinuria
C. BUN: 10 mg/dL
D. Few to many pus cells in urinalysis
ANSWER: __B______
RATIO:This could signify a possibility of eclampsia and this can be complicated by drug effects of beta-agonists (e.g. uterine relaxation
and increased blood pressure).

6. Student Nurse Aimee is reviewing for her licensure exam. She would be right to pick which drug pairing (classification and
corresponding example)?
A. Alpha1-selective adrenergic blocking agent: phentolamine
B. Nonselective adrenergic blocking agent: labetalol
C. Nonselective alpha-adrenergic blocking agent: doxazosin
D. Nonselective beta-adrenergic blocking agent: metoprolol
ANSWER: ____B____
RATIO:Other examples of nonselective adrenergic blocking agents aside from labetalol are amiodarone and carvedilol. Phentolamine
is a nonselective alpha blocker. Doxazosin is an alpha1-selective blocker and metoprolol is a selective beta-blocker.

7. Which of the following drugs can improve urine flow in males?


A. Prazosin
B. Nebivolol
C. Labetalol
D. Amiodarone
ANSWER: __A______
RATIO:Prazosin, a selective alpha1-receptor adrenergic antagonist, can improve urine flow in male patients and are used as
treatment for benign prostatic hypertrophy (BPH). This is because they can block smooth muscle receptors in the genitourinary tract
which leads to relaxation of prostate and bladder.
8. Nurse Sam is taking care of a patient on carvedilol therapy. Which parameter should he monitor regularly to assess for possible
dose adjustment?
A. Serum sodium level
B. Hemoglobin and hematocrit
C. SGPT
D. BUN
ANSWER: ___C_____
RATIO:Carvedilol has been associated with hepatic failure related to its effects on the liver.

9. Mrs. Smith, a 40-year-old patient, came in due to acute hypertensive episode. Her chart says she is asthmatic. Which of the
following drugs would be safest to give?
A. Nebivolol
B. Timolol
C. Propranolol
D. Atenolol
ANSWER: ____D____
RATIO:Atenolol is a selective beta-blocker and it does not block the receptors responsible for bronchodilation. All other options are
nonselective beta-blockers and may cause bronchoconstriction and spasms.

10. Which of the following drug is used for diagnosis of pheochromocytoma?


A. Amiodarone
B. Phentolamine
C. Tamsulosin
D. Acebutolol
ANSWER: ___B_____
RATIO: Phentolamine is a long-acting, adrenergic, alpha-receptor blocking agent which can produce and maintain "chemical
sympathectomy" by oral administration.

11. Your patient with asthma is taking Theophylline. Which product below should the patient avoid consuming?
A. Caffeine
B. Dairy
C. Wheat
D. Shellfish
ANSWER: ____A____
RATIO:The answer is A. Caffeine has the same properties as Theophylline and can increase the effects the drug.

12. Which medication below blocks the function of Leukotriene for the treatment of asthma?*
A. Salmeterol
B. Theophylline
C. Tiotropim
D. Montelukast
ANSWER: __D______
RATIO: Montelukast is used to prevent wheezing, difficulty breathing, chest tightness, and coughing caused by asthma in adults and
children 12 months of age and older. Montelukast is also used to prevent bronchospasm (breathing difficulties) during exercise in
adults and children 6 years of age and older.

13. A patient with asthma is prescribed to take inhaled Salmeterol and Fluticasone for long-term management of asthma. You
observe the patient taking these medications. Which option below best describes the correct order in how to take these
medications?
A. The patient inhales the Salmeterol first and then waits 5 minutes before inhaling the Fluticasone.
B. The patient inhales the Fluticasone first and then waits 5 minutes before inhaling the Salmeterol.
C. The patient inhales the Salmeterol first and then waits 1 minute before inhaling the Fluticasone.
D. The patient inhales the Fluticasone and immediately inhales the Salmeterol.
ANSWER: ___A_____
RATIO: The answer is A. The bronchodilator inhaler (Salmeterol) is administered first to open up the airways. Then the patient is to
wait five minutes and then administer the corticosteroid (Fluticasone). The bronchodilator will open the airways so the corticosteroid
can easily enter the airways to decrease inflammation.

14. Epinephrine is used to treat cardiac arrest and status asthmaticus because of which of the following actions?
A. Increased speed of conduction and gluconeogenesis
B. Bronchodilation and increased heart rate, contractility, and conduction
C. Increased vasodilation and enhanced myocardial contractility
D. Bronchoconstriction and increased heart rate
ANSWER: ____B____
RATIO: Bronchodilation results from stimulated beta receptors, and cardiac effects result from the stimulation of ß1 receptors.

15. Your patient is having a sudden and severe anaphylactic reaction to a medication. You immediately stop the medication and call a
rapid response. The patient’s blood pressure is 80/52, heart rate 120, and oxygen saturation 87%. Audible wheezing is noted along
with facial redness and swelling. As the nurse you know that the first initial treatment for this patient’s condition is?
A. IV Diphenhydramine
B. IV Normal Saline Bolus
C. IM Epinephrine
D. Nebulized Albutero

SAS 19

1. The following are true about cholinergic agonists, except:

A. These drugs exert their effect by mobilizing the acetylcholine into their receptors found throughout the body.

Ratio-Cholinergic agonists act at the same site as the neurotransmitter acetylcholine (ACh) and increase the activity of the ACh
receptor sites throughout the body. They do not mobilize acetylcholine into their receptors.

2. All of the following are direct-acting cholinergic agonists, except:

B. Neostigmine

Ratio- It is an agent for myasthenia gravis and is therefore an indirect-acting cholinergic agonist.

3. This drug is usually indicated for postoperative and postpartum urinary retention as well as bladder atony

b. bethanechol

Ratio-This drug has specific affinity for receptors in the urinary bladder.

4. Which of the following is an indication for withholding indirect-acting cholinergic agonists?

D. Adult heart rate of 55 beats per minute

Ratio-It can be exacerbated by the parasympathetic effects of the drug

5. A patient on neostigmine complained of frequent diarrhea and salivation. Which is the best nursing action for this situation?

B. Withhold the dose of the drug and refer to physician

Ratio- Discontinue drug if excessive salivation, diarrhea, emesis, or frequent urination becomes a problem to decrease the
risk of severe adverse reactions. The nurse should refer this to the physician.

6. Anticholinergic drugs are used


A. To allow the sympathetic system to dominate.

Ratio- It decreases GI activity and secretions in the treatment of ulcers and to decrease other parasympathetic activities to allow the
sympathetic system to dominate. More specific and less systematically toxic drugs are available for treatment of ulcers.

7. Atropine and scopolamine work by blocking what receptor(s) in the parasympathetic nervous system?

C. Muscarinic receptors only

Ratio- Atropine and scopolamine work by blocking only the muscarinic effectors in the parasympathetic nervous system
and the few cholinergic receptors in the SNS.

8. Which of the following suggestions would the nurse make to help a patient who is receiving an anticholinergic agent reduce the risks
associated with decreased sweating?

B. Ensuring hydration and temperature control

Ratio- This drug blocks sweating, which is the body’s way of cooling off. This places the patient at an increased risk for heat stroke.
Extremes of temperature should be avoided.

9.Which of the following would the nurse be least likely to include when developing a teaching plan for a patient who is

receiving an anticholinergic agent?

D. Performing exercises to increase the heart rate

Ratio- Exercises should be avoided because this drug blocks sweating. This placed the patient at increased risk for heat stroke.

10. Remembering that anticholinergics block the effects of PNS, the nurse would question an order for an anticholinergic drug for
patients with which of the following conditions?

C. Glaucoma

Ratio- All other options are indications for atropine.

11. Remembering that anticholinergics block the effects of PNS, the nurse would question an order for an anticholinergic drug for
patients with which of the following conditions?

B . Heart failure

Ratio- Cholinergic drugs decrease heart rate and force of contraction, which can result in increased symptoms of HF.

12. A client diagnosed with Alzheimer's disease is prescribed donzepil (Aricept) for treatment. The client has a medical history of
osteoarthritis and GERD. For what possible complications of this drug therapy should the nurse assess in the client?

c. Aspiration

Rationale: Aricept is an indirect sympathomimetic drug. These drugs exacerbate conditions of decreased sphincter muscle tone and
gastric motility, placing the client at risk for esophageal injury and aspiration.

13. Pyridostigmine (Mestinon) is prescribed for a client diagnosed with myasthenia gravis. Priority nursing assessment includes
assessing the:

a) Ability to chew and swallow.


Rationale: Mestinon stimulates skeletal muscle contraction, and helps to reverse the muscle weakness seen with multiple
sclerosis. An improvement in chewing and swallowing should occur, but the client must be assessed for this improved
function.

14. The nurse provides preoperative teaching for a client on the side effects of atropine given as a preoperative medication. The
nurse informs the client to expect:

B. Dry mouth.

Rationale: Atropine blocks the parasympathetic action of ACh, and causes symptoms such as increased heart rate, decreased GI
motility, and decreased secretions from glands.

15. Anticholinergic drugs like atropine are given preoperatively to:

C. Decrease the risk of bradycardia during surgery


Ratio: Atropine is the first line medication for the treatment of bradycardia. The administration of atropine typically causes an
increase in heart rate.

SAS 20

1. Treatment with ADH preparations is associated with adverse effects, including


A. “hangover symptoms” like headache and sweating
B. constipation and paralytic ileus
C. cholecystitis and bile obstruction
D. nocturia and bedwetting
ANSWER: _____A___
RATIO: These are symptoms of water intoxication. Other adverse effects include cramps, dry mouth, vomiting, and local reaction at
injection site.

2. Which of the following is true regarding growth hormone (GH) deficiencies?


A. It occurs only in children.
B. It always results in dwarfism.
C. It is treated only in children because GH is usually produced only until puberty.
D. It can occur in adults as well as children.
ANSWER: _____D___
RATIO:GH deficiencies can be caused either by congenital defects (children) and pituitary tumors and trauma (usually adults), among
others.

3. Which of the following drugs can increase the toxicity of bromocriptine?


A. Phenothiazine
B. Opioids
Erythromycin
C. Statins
ANSWER: __Erythromycin (WALA SIYAY LETTER SA SAS)______
RATIO:Combining erythromycin and bromocriptine will increase the risk of toxicity. Phenothiazines decrease the effectiveness of
bromocriptine. Lastly, opioids will warrant an increase in dosage of GH antagonists.

4. This is the most common type of thyroid dysfunction.


A. Cretinism
B. Myxedema coma
C. Hyperthyroidism
D. Hypothyroidism
ANSWER: ____D____
RATIO: Hypothyroidism is a lack of sufficient levels of thyroid hormones to maintain a normal metabolism. This is the most common
type of thyroid dysfunction and is common among older women and men. Symptoms include obesity and fatigue, among others.

5. What does thyroid hormones do?


A. stimulate overproduction of TSH
B. increase oxygen consumption
C. increase the heart rate without affecting the respirations
D. retard growth and maturation
ANSWER: ___A_____
RATIO: Thyroid hormones increase the metabolic rate of body tissues, increasing oxygen consumption, respiration, heart rate, growth
and maturation, and the metabolism of fats, carbohydrates, and proteins.

6. All of the following should be included in the nursing interventions for patients taking thyroid hormones, except:
A. consistent administration before breakfast each day
B. monitor cardiac response
C. advise patient to frequently exercise to develop the core
D. provide for temperature control and safety precautions
ANSWER: ___C_____
RATIO:As much as possible, patients should be provided for rest periods when on this therapy.

7. Which of the following is the antithyroid agent of choice for pregnant women.
A. Methimazole
B. PTU
C. Levothyroxine
D. iodine solution
ANSWER: __B______
RATIO:Other antithyroid agents can lead to development of cretinism.

8. Which of the following history data will possibly render strong iodine products contraindicated for the patient?
A. 35 years old
B. history of blood-tinged sputum for the past three weeks
C. intake of oral anticoagulants
D. both B and C
ANSWER: ____D____
RATIO:A blood-tinged sputum for the past three weeks may signify tuberculosis. If this is the case, strong iodine products are
contraindicated. Also, strong iodine products can change the metabolism and level of anticoagulants and may place the patient at
greater risk for bleeding. Lastly, I131 is only for patients over 30 years old because of adverse effects associated with radioactivity.

9. A patient taking calcitriol is also prescribed with a magnesium-containing antacid. Which of the following should be included in the
nurse’s monitoring?
A. Color of stool
B. Signs of bleeding
C. Liver function test
D. Serum electrolyte level
ANSWER: ____D____
RATIO: Antihypocalcemic agents combined with magnesium-containing antacids increase the risk for hypermagnesemia. Therefore, it
is important to monitor serum magnesium level.

10. Which of the following should be included in patient teaching of patients taking bisphosphonates?
A. Stay upright 30 minutes after taking the drug.
B. Take drug on empty stomach.
C. Chew drug instead of swallowing.
D. Both B and C
ANSWER: ___A_____
RATIO:Drug should be taken with a full glass of water and the patient should stay upright for 30 minutes because serious esophageal
erosion can occur.

11. Common adverse effects of calcitonin include:


A. Headache
B. Flushing of the face and hands
C. Abdominal tenderness
D. Diarrhea
ANSWER: __B______
RATIO:Calcitonin is available in IM, subcutaneous, and nasal forms. Other adverse effects include skin rash, nausea and vomiting,
urinary frequency, and local inflammation at the site of injection. Headache and diarrhea are common in bisphosphonates.
12. This is a genetically-linked condition with parathyroid dysfunction characterized by deep bone pain, headaches, and hearing loss.
A. Tay-Sachs disease
B. Paget’s disease
C. Gaucher disorder
D. Marfan Syndrome
ANSWER: ___B_____
RATIO: Paget’s disease is a genetically linked disorder characterized by overactive osteoclasts that are eventually replaced by
enlarged and softened bony structures. Manifestations include deep bone pain, headaches, and hearing loss.

13. Glucocorticoids can reduce inflammation and suppress the immune system by:
A. forming complex reactions needed to reduce inflammation
B. activating more lymphocytes to reduce inflammation
C. inhibiting the localization of phagocytes so immune system can rest
D. all of the above
ANSWER: __A______
RATIO:They bind to cytoplasmic receptors of target cells to form complex reactions needed to reduce inflammation and suppress the
immune system. They also limit the activity of lymphocytes to act within the immune system. Lastly, they inhibit the spread of
phagocytes to the bloodstream and injured tissues.

14. A patient is started on a regimen of prednisone because of a crisis in her ulcerative colitis. Nursing care of this patient would need
to include:
A. immunizations to prevent infections
B. increased calories to deal with metabolic changes
C. fluid restriction to decrease water retention
D. administration of the drug around 8 or 9 AM to mimic normal diurnal rhythm.
ANSWER: ___D_____
RATIO:Administer drug daily at 8 to 9 AM to mimic normal peak diurnal concentration levels and thereby minimize suppression of the
hypothalamic-pituitary axis (HPA).

15. Which of the following patient complaint(s) will alert the nurse for possible aldosterone toxicity?
A. muscle weakness and abdominal fullness
B. BP of 80/60 mmHg
C. parched skin and dry mucous membranes
D. all of the above

SAS 21

1. Marlisa has been diagnosed with diabetes mellitus type 1. She asks Nurse Errol what this means. What is the best response by the
nurse? Select all that apply.
A. “Your alpha cells should be able to secrete insulin, but cannot.”
B. “The exocrine function of your pancreas is to secrete insulin.”
C. “Without insulin, you will develop ketoacidosis (DKA).”
D. “The endocrine function of your pancreas is to secrete insulin.”
E. “It means your pancreas cannot secrete insulin.”
ANSWER: ________
RATIO:One function of your pancreas is to secrete insulin. The endocrine function of the pancreas is to secrete insulin. The
endocrine, not the exocrine, function of the pancreas is to secrete insulin. Insulin is secreted by the beta, not the alpha, cells of the
pancreas. A consequence of diabetes mellitus type 1 is that without insulin, severe metabolic disturbances, such as ketoacidosis
(DKA) will result

2. Dr. Shrunk orders intravenous (IV) insulin for Rita, a client with a blood sugar of 563. Nurse AJ administers insulin lispro (Humalog)
intravenously (IV). What does the best evaluation of the nurse reveal? Select all that apply.
A. The nurse could have given the insulin subcutaneously.
B. The nurse should have contacted the physician.
C. The nurse should have used regular insulin (Humulin R).
D. The nurse used the correct insulin.
ANSWER: ________
RATIO:Regular insulin is the only insulin that can be given intravenously (IV). The nurse did not use correct insulin as it was not
regular insulin. Contact the provider to clarify the order, regular insulin is the only insulin that can be given intravenously (IV). The
nurse cannot give the insulin subcutaneously when it is ordered to be given intravenously (IV).

3. Ben injects his insulin as prescribed, but then gets busy and forgets to eat. What will the best assessment of the nurse reveal?
A. The client will be very thirsty.
B. The client will complain of nausea.
C. The client will need to urinate.
D. The client will have moist skin.
ANSWER: ________
RATIO: Moist skin is the sign of hypoglycemia, which the client would experience if he injected himself with insulin and did not eat.
Thirst, nausea, and increased urination are signs of hyperglycemia.

4. A clinical instructor teaches a class for the public about diabetes mellitus. Which individual does the nurse assess as being at
highest risk for developing diabetes?
A. The 50-year-old client who does not get any physical exercise
B. The 56-year-old client who drinks three glasses of wine each evening
C. The 42-year-old client who is 50 pounds overweight
D. The 38-year-old client who smokes one pack of cigarettes per day
ANSWER: ________
RATIO:Obesity increases the likelihood of developing diabetes mellitus due to over stimulation of the endocrine system. Exercise is
important, but lack of exercise is not as big a risk factor as obesity. Smoking is a serious health concern but is not a specific risk factor
for diabetes. Consuming alcohol is associated with liver disease but is not as high a risk factor for diabetes as obesity.

5. Steven John has type 1 diabetes mellitus and receives insulin. Which laboratory test will the nurse assess?
A. Potassium
B. AST (aspartate aminotransferase)
C. Serum amylase
D. Sodium
ANSWER: ________
RATIO: Insulin causes potassium to move into the cell and may cause hypokalemia. There is no need to monitor the sodium, serum
amylase, and AST levels.

6. Jansen is receiving metformin (Glucophage). What will be the best plan of the nurse with regard to patient education with this
drug? Select all that apply.
A. It stimulates the pancreas to produce more insulin.
B. It must be taken with meals.
C. It decreases sugar production in the liver.
D. It inhibits absorption of carbohydrates.
E. It reduces insulin resistance.
ANSWER: ________
RATIO: Metformin (Glucophage) reduces insulin resistance, decreases sugar production in the liver, and should be taken with meals
for the best absorption and effect. It does not stimulate the pancreas to produce more insulin and does not inhibit the absorption of
carbohydrates.

7. Nurse Andy has finished teaching a client with diabetes mellitus how to administer insulin. He evaluates the learning has occurred
when the client makes which statement?
A. “I should check my blood sugar immediately prior to the administration.”
B. “I should provide direct pressure over the site following the injection.”
C. “I should use the abdominal area only for insulin injections.”
D. “I should only use calibrated insulin syringe for the injections.”
ANSWER: ________
RATIO:To ensure the correct insulin dose, a calibrated insulin syringe must be used. Insulin injections should be rotated to the arm
and thigh, not just the abdominal area. There is no need to apply direct pressure over the site following an insulin injection. There is
no need to check blood glucose immediately prior to the injection.

8. Genevieve has diabetes type 1 and receives insulin for glycemic control. She tells the nurse that she likes to have a glass of wine
with dinner. What will the best plan of the nurse for client education include?
A. The alcohol could cause pancreatic disease.
B. The alcohol could cause serious liver disease.
C. The alcohol could predispose you to hypoglycemia.
D. The alcohol could predispose you to hyperglycemia.
ANSWER: ________
RATIO: Alcohol can potentiate hypoglycemic, not hyperglycemic, effects in the client. Alcohol can cause pancreatic disease, but the
client's pancreas is not producing any insulin currently. Alcohol can cause liver disease, but the more immediate concern is
hypoglycemia.

9. Dr. Wijangco orders insulin lispro (Humalog) 10 units for Alicia, a client with diabetes mellitus. When will the nurse administer this
medication?
A. When the client is eating
B. Thirty minutes before meals
C. fifteen minutes before meals
D. When the meal trays arrive on the floor
ANSWER: ________
RATIO:The onset action for the insulin lispro (Humalog) is 10 to 15 minutes so it must be given when the client is eating to prevent
hypoglycemia. It must be given when the client is eating, not when the meal trays arrive on the floor and not thirty minutes before
meals.
10. Nurse Matt makes a home visit to the client with diabetes mellitus. During the visit, Nurse Matt notes the client’s additional
insulin vials are not refrigerated. What is the best action by the nurse at this time?
A. Instruct the client to label each vial with the date when opened.
B. Tell the client there is no need to keep additional vials.
C. Have the client place the insulin vials in the refrigerator.
D. Have the client discard the vials.
ANSWER: ________
RATIO:Correct answer: C. Have the client place the insulin vials in the refrigerator.

Option C: Vials not in use should be refrigerated to preserve drug potency.


Option A: Writing the date of opening on the vial is good practice, but does not address the need to refrigerate additional vials.
Option B: The client should always have additional vials of insulin available.
Option D: There is no need to discard the vials.

11. During the morning rounds, Nurse AJ accompanied the physician in every patient’s room. The physician writes orders for the
client with diabetes mellitus. Which order would the nurse validate with the physician?
A. Use Humalog insulin for sliding scale coverage.
B. Metformin (Glucophage) 1000 mg per day in divided doses.
C. Administer regular insulin 30 minutes prior to meals.
D. Lantus insulin 20U BID.
ANSWER: ________
RATIO:Correct answer: D. Lantus insulin 20U BID.

Option D: Lantus insulin is usually prescribed once-a-day so an order for BID dosing should be validated with the physician.
Option A: Humalog insulin can be prescribed for sliding scale coverage.
Option B: Metformin (Glucophage) is often prescribed in divided doses of 1000 mg per day.
Option C: Regular insulin is administered 30 minutes before meals.

12. An older woman with diabetes mellitus visits the clinic concerning her condition. Of which of the following symptoms might an
older woman with diabetes mellitus complain?
A. Anorexia
B. Pain intolerance
C. Weight loss
D. Perineal itching
ANSWER: ________
RATIO: Older women might complain of perineal itching due to vaginal candidiasis.

13. Which of the following exerts its effect by inhibiting the enzyme that breaks down glucose for absorption?
A. Miglitol
B. Metformin
C. Pramlintide
D. Linagliptin
ANSWER: ________
RATIO:The enzyme that breaks down glucose for absorption is called alpha glucosidase. Drugs under alpha glucosidase inhibitors
include acarbose and miglitol.

14. This antidiabetic agent works best in patients with insulin resistance.
A. Biguanides
B. Dipeptidyl peptidase-4 inhibitors
C. Thiazolidinediones
D. Meglitinides
ANSWER: ________
RATIO:Thiazolidinediones pioglitazone and rosiglitazone work by decreasing insulin resistance.

15. Which antidiabetic agent works by modulating gastric emptying after a meal to cause a feeling of fullness?
A. Incretin mimetics
B. Meglitinides
C. Human amylin
D. Biguanides
ANSWER: ________
RATIO:Synthetic human amylin pramlintide works to modulate gastric emptying after a meal to cause a feeling of fullness or satiety. It
also prevents the postmeal rise in glucagon that usually elevates glucose levels. Human amylin is a hormone produced by beta cells
in the pancreas that is important in regulating postmeal glucose levels. It should not be used when patient is unable to eat.

SAS 22

1. The three primary elements that determine the pressure in the cardiovascular system are heart rate, stroke volume, and:
A. Baroreceptors
B. Peripheral resistance
C. Renin-Angiotensin-Aldosterone System (RAAS)
D. All of the above
ANSWER: ___B_____
RATIO: Small arterioles help regulate blood pressure in a constant basis and are the most important factor affecting peripheral
resistance. When small arterioles constrict, they are almost able to stop the flow of the blood. Their high responsiveness to
sympathetic nervous stimulation cause them to constrict and increase blood pressure
2. Untreated hypertension can result to which of the following:
A. Stroke
B. Renal failure
C. Loss of vision
D. All of the above
ANSWER: ___D_____
RATIO:The force of the blood propelled against blood vessels can damage their linings, making them susceptible to atherosclerosis
and narrowing. When these happen, tiny vessels in the retina, kidneys, and brain can be destroyed leading to unresponsiveness of
the system.
3. A student nurse was asked to take the blood pressure of a patient in the emergency room. The BP reads 140/90 mmHg. She would
be correct to say that this BP belongs to ____________ classification.
A. Normal
B. Elevated
C. Stage 1 Hypertension
D. Stage 2 Hypertension
ANSWER: ___D_____
RATIO:A category rating the severity of hypertension has been devised and the classifications of blood pressure are as follows:
Normal - systolic: <120 mmHg; diastolic: <80 mmHg; Elevated - systolic: 120-129 mmHg; diastolic: <80 mmHg; Stage 1 Hypertension -
systolic: 130-139 mmHg; diastolic: 80-89 mmHg; Stage 2 Hypertension - systolic: less than or equal to 140 mmHg; diastolic: less than
or equal to 90 mmHg.
4. Which of the following medications will decrease the therapeutic effects of quinapril (Accupril)?
A. NSAIDs
B. Allopurinol
C. Antacids
D. All of the above
ANSWER: ___A_____
RATIOQuinapril belongs to the class ACE inhibitors. NSAIDs are linked to decreased antihypertensive effects of ACE inhibitors.
Allopurinol increases risk of hypersensitivity when combined with ACE inhibitors.
5. A patient receiving nitroprusside begun manifesting headache, distant heart sounds, imperceptible pulses, and shallow breathing.
The nurse knows that these are signs of?
A. Reflex tachycardia
B. Hypothyroidism
C. Cyanide toxicity
D. Severe alteration of blood pressure
ANSWER: ___C_____
RATIONitroprusside is metabolized into cyanide. Other manifestations of cyanide toxicity include pink color, loss of consciousness,
and absence of reflexes.

6. the nurse is administering antihypertensive drugs to older adults patients. the nurse knows that which adverse effect is of most
concern for these patients?
A. dry mouth
B. hypotension
C. restlessness
D. constipation
ANSWER: ___B_____
RATIO:Low blood pressure might seem desirable, and for some people, it causes no problems. However, for many people, abnormally
low blood pressure (hypotension) can cause dizziness and fainting. In severe cases, low blood pressure can be life-threatening.

7. Which of the following objective data should the nurse consider when determining the effectiveness of the drug therapy?
A. Blood pressure log notes blood pressure 120/70-134/88 since discharge.
B. Weight loss of six pounds in the past month
C. Frequency of voiding of at least six times per day
D. Absence of edema in lower extremities
ANSWER: ___A_____
RATIO:Maintenance of blood pressure within normal limits indicates that treatment goals are achieved. Absence of edema, weight
loss, and urinating all indicate that the diuretic has promoted fluid loss, but are not the best measure of the drug's effectiveness for
hypertension.

8. A client receiving HCTZ 25 mg q.d. and digoxin 0.125 mg q.d. complains of nausea and vomiting, and of seeing halos around lights.
The client's serum digoxin level is 2.5 ng. The appropriate nursing intervention is to:
A. Administer both drugs as ordered.
B. Document the findings; the lab results are within normal limits
. C. Hold the digoxin and HCTZ.
D. Hold the digoxin, and give HCTZ as ordered.
ANSWER: ___C_____
RATIO: Thiazide diuretics increase serum digitalis levels by promoting potassium loss, which increases the risk of digoxin toxicity. A
digitalis level above 2.0 ng is toxic.

9. Nitroprusside (Nitropress) is prescribed for a client admitted with a blood pressure of 220/110. What action by a new nurse would
require intervention by the charge nurse? A. The nurse inserts a Foley catheter. B. The nurse documents the IV rate and status of site
every 15 minutes. C. The nurse uses electronic monitoring of blood pressure every hour. D. The nurse cautions the client to call for
assistance before getting out of bed.
ANSWER: ___C_____
RATIO:Nitroprusside decreases blood pressure instantaneously. Vital signs must be monitored very closely—e.g., every 5-15 minutes.

10. The nurse reviews the teaching plan with a client receiving nifedipine (Procardia). Which of the following client behaviors
indicates understanding?
A. The client avoids taking the drug with grapefruit juice.
B. The client consumes three servings of alcohol daily.
C. The client breaks an enteric-coated tablet for ease of swallowing.
D. The client monitors blood pressure every week.
ANSWER: ____A____
RATIO:_Grapefruit juice increases absorption of nifedipine, resulting in increased serum level. Blood pressure ideally should be
monitored more frequently. Alcohol intake of three times a day is excessive. Breaking enteric tablets will interfere with time release
of the medication.

11. A diuretic is added to the treatment regimen for a client with hypertension. The nurse explains that diuretics help reduce blood
pressure by:
A. Removing serum potassium.
B. Dilating peripheral blood vessels.
C. Reducing sympathetic outflow.
D. Constricting blood vessels.
ANSWER: ___C_____
RATIO:Diuretics decrease blood volume, which in turn decreases the workload of the heart and reduces blood pressure. They do not
dilate blood vessels. Some diuretics promote potassium loss, but this does not reduce the blood pressure. Central-acting
antihypertensives work by blocking sympathetic outflow.

12. Atenolol (Tenormin) is prescribed for a client with hypertension. The nurse recognizes that a safe dose for this drug is:
A. Atenolol (Tenormin) 100 mg b.i.d.
B. Atenolol (Tenormin) 150 mg q.d.
C. Atenolol (Tenormin) 50 mg b.i.d.
D. Atenolol (Tenormin) 75 mg b.i.d.
ANSWER: ___C_____
RATIO:The maximum dosage range for Atenolol (Tenormin) is 100 mg/day.

13. Nifedipine (Procardia) 30 mg p.o. is prescribed for a client. The nurse teaches the side effects and instructs the client to
immediately report:
A. Blood pressure 110/70-114/78 for two successive readings.
B. Dizziness when changing positions.
C. Increased shortness of breath and orthopnea.
D. Weight loss of two pounds per week.
ANSWER: ____C____
RATIO: Nifedipine (Procardia) is a calcium channel blocker. Calcium channel blockers decrease myocardial contractility, increasing the
risk of heart failure. Dizziness can occur, especially when the medication is started. The BP is a desired reading

14. The client is taking atenolol (Tenormin) and doxazosin (Cardura). what is the rationale for combining 2 anti-HT drugs? A. BP will
decrease faster B. Lower doses of both drugs may be given with fewer adverse effects. C. There is less daily medication dosing. D.
Combination therapy will treat the patient's other medical conditions.
ANSWER: ___B_____
RATIO: The advantage of using a combination of two drugs such as atenolol (Tenormin; a beta blocker) and doxazosin (Cardura; an
alpha-1 antagonist) is that lower doses of each may be used, resulting in fewer side effects.

15. What health teaching should the nurse provide for the client receiving nadolol (Corgard)? A. Increase fluids & fiber to prevent
constipation B. Report weight gain of 1kg / month or more C. Immediately stop taking meds if sexual dysfunction occurs. D. Rise
slowly after prolonged periods of sitting or lying down.
ANSWER: ___D_____
RATIO:Nadolol (Corgard) may increase the risk of orthostatic hypotension and the client should be taught to rise slowly to standing
from a sitting or lying position.

SAS 23

Multiple Choice
1. Nursing student Annie was assigned in the cardiovascular unit A wherein all the patients have heart failure. Ms. Annie is aware
that the primary treatment for heart failure (HF) is which of the following?.
A. Increasing the heart rate so the heart can pump more blood
B. Decreasing the heart rate so the heart can rest
C. Increasing contractility so the heart will be able to pump more blood
D. Decreasing contractility to prevent muscle fatigue
ANSWER: ________
RATIO:______________________________________________________________________________________________________
____________________________________________________________________________________________________________
_________________________________________________________________________.
2. Which of the following is the most common cause of heart failure?
A. Hypertension
B. Valvular heart diseases
C. Cardiomyopathy
D. Coronary artery disease (CAD)
ANSWER: ___D_____
RATIO: Coronary artery disease, if leads to insufficient blood supply for the myocardium and is also the most common cause of HF.

3. Digoxin was prescribed to a patient with ventricular tachycardia. What should the nurse do?
A. Administer the drug as ordered.
B. Discuss the order with the doctor.
C. Discontinue other intravenous medications before administering digoxin.
D. Count apical pulse for one full minute before administering.
ANSWER: ________
RATIO: The nurse should discuss the order with the doctor because the patient has a ventricular tachycardia which is a
contraindication and its potentially fatal arrhythmias and should be treated with another drug

4. What is the antidote for digoxin intoxication?


A. Diphenhydramine
B. Atropine sulfate
C. Digoxin immune fab (Digibind, DigiFab).
D. Phosphodiesterase inhibitors
ANSWER: ________
RATIO: these antibodies bind molecules of digoxin, making them unavailable at site of action. Used when serum digoxin is >10 ng/mL
and serum potassium is >5 mEq/L.

5. An infant who is receiving cardiac glycosides has an apical pulse of 80 beats per minute. Which is the best nursing intervention for
this assessment finding?
A. Administer drug as ordered.
B. Withdraw the drug and notify doctor.
C. Assess apical pulse every hour for the next five hours.
D. Decrease drug dose and administer.
ANSWER: ________
RATIO: Drug is withheld if pulse is less than 90 beats per minute in infants. Apical pulse is taken after one hour and if it remains low,
nurse must document it, withhold the dose, and inform doctor

6. What signals the novice nurse that intravenous milrinone was combined to furosemide in management of patients with heart
failure?
A. Presence of bubbles
B. Pink discoloration of the solution
C. Formation of precipitates
D. No obvious sign. Solution is clear.
ANSWER: ________
RATIO: Phosphodiesterase inhibitor-furosemide combination should be avoided. Alternate lines should be used if both of these drugs
are given intravenously
7. What is the therapeutic level for digoxin?
A. 0.5-2 ng/Ml
B. 1.5-2 ng/mL
C. 0.5-1.5 mg/mL
D. 0.5-2 mg/Ml
ANSWER: __A______
RATIO: Monitor serum digoxin level as ordered (normal: 0.5-2 ng/mL) to evaluate therapeutic dosing and development of adverse
effects

8. In severe cardiac glycoside toxicity, all of the following should be in the bedside, except _________.
A. Lidocaine
B. Phenytoin
C. Calcium channel blocker
D. A and B only
ANSWER: ________
RATIO:Ensure maintenance of emergency drugs and equipment at bedside (e.g. potassium salts and lidocaine for arrhythmias,
phenytoin for seizures, atropine in case of clinically significant low heart rate, and cardiac monitor) to promote prompt treatment in
cases of severe toxicity

9. Type of angina which involves spasm of the blood vessels


A. Stable angina
B. Pre-infarction angina
C. Unstable angina
D. Prinzmetal angina
ANSWER: ________
RATIO: vasospastic/Prinzmetal’s/variant angina, which is caused by reversible vasospasm even at rest. Both types decrease oxygen
supply of the heart.

10. Which instruction should be included in the discharge teaching for a patient with a transdermal nitroglycerin (Nitro-Dur) patch?
A. "If you get chest pain, apply a second patch next to the first patch."
B. "If you get a headache, remove the patch for 4 hours and then reapply."
C. "Make sure to rub a lotion or cream on the skin before putting on a new patch."
D. "Apply the patch to a hairless, nonirritated area of the chest, upper arm, back or shoulder."
ANSWER: ________
RATIO:__if the patch is placed on a hairy chest, onced it is time to remove the patch,hair on the chest will also be removed and can
cause skin sensitivity and irritation.

11. Contraction and relaxation in each cardiac cycle is controlled by:


A. autonomic nervous system
B. the heart
C. cranial nerves
D. central nervous system
ANSWER: ________
RATIO:______________________________________________________________________________________________________
____________________________________________________________________________________________________________
_________________________________________________________________________.

12. All of the following can cause arrhythmia, except:


A. Acidosis
B. respiratory depression
C. hyperkalemia
D. none of the above
ANSWER: ________
RATIO: Acidosis has complex electrophysiological effects, which are associated with a high recurrence of ventricular arrhythmias.
Hyperkalemia is a higher than normal level of potassium in the blood. Although mild cases may not produce symptoms and may
be easy to treat, severe cases of hyperkalemia that are left untreated can lead to fatal cardiac arrhythmias, which are abnormal
heart rhythms.

13. Verapamil and diltiazem belong to which class of antiarrhythmics?


A. Class IV
B. Class III
C. Class Ia
D. Class II
ANSWER: ________
RATIO:Class IV ant arrhythmic drugs are the calcium channel blockers, verapamil and diltiazem. These are normally monitored by
haemodynamic effects rather than using TDM

14. The conduction system of the heart include the following:


A. Sinoatrial node
B. Bundle of Purkinje
C. His Fibers
D. Atriomyocardial node
ANSWER: ________
RATIO:______________________________________________________________________________________________________
____________________________________________________________________________________________________________
_________________________________________________________________________.

15. The patient states to the nurse, "My friend said nitroglycerin relieves angina pain by reducing preload. What is preload?" Which
statement by the nurse explains preload to this patient?
A. "It is the blood return to the heart."
B. "It is the oxygen demand of the heart."
C. "It is the pressure against which the heart must pump."
D. "It is dilation of arteries and veins “
SAS 24

Multiple Choice
1. Lipid levels of individuals with coronary artery disease (CAD) is usually high. All of the following are factors of CAD, except:
A. Men
B. Gout
C. Untreated Chlamydia infections
D. None of the above
ANSWER: ________
RATIO: THE INCIDENCE OF CAD IN MEN ARE HIGHER THAN PREMENOPAUSAL WOMEN.

2. Which assessment finding in a patient taking an HMG-CoA reductase inhibitor will the nurse act on immediately?
A. Decreased haemoglobin
B. Elevated liver function tests
C. Elevated HDL
D. Elevated LDL
ANSWER: ________
RATIO:HMG-CoA reductase inhibitors (statins) can cause hepatic toxicity; thus, it is necessary to monitor liver function tests. The
nurse should act on this finding immediately. Decreased hemoglobin should be addressed but not immediately. It is most likely not
related to the administration of the HMG-CoA reductase inhibitor. Also, while an elevated LDL level must be addressed, it is not as
high a priority as the elevated liver function test results. An elevated HDL is a positive finding and an encouraging result.

3. A patient is taking pravastatin sodium. Which assessment finding requires immediate action by the nurse?
A. Headache
B. Slight nausea
C. Muscle pain
D. Fatigue
ANSWER: ________
RATIO: Patients who experience severe muscle pain while taking pravastatin sodium need to report the findings right away, as this
may be indicative of rhabdomyolysis, a muscle disintegration that can become fatal.
4. Antihyperlipidemic agent that is used to decrease plasma cholesterol levels.
A. HMG-CoA reductase inhibitors
B. Phosphodiesterase inhibitors
C. Bile acid sequestrants
D. Cholesterol absorption inhibitor
ANSWER: ________
RATIO:Cholesterol absorption inhibitors are one of the new class of drugs approved (2003) to lower serum cholesterol levels.

5. A pregnant woman needs a lipid-lowering agent. What would be the best class of lipid-lowering agent for pregnant women?
E. HMG-CoA reductase inhibitors
F. Bile acid sequestrants
G. Cholesterol absorption inhibitors
H. Phosphodiesterase inhibitors
ANSWER: ________
RATIO:A statin must be used during pregnancy. Atorvastatin, also called Lipitor, is one of the more common medications prescribed
to help manage high cholesterol. Atorvastatin is in a class of medications called HMG-CoA reductase inhibitors (statins).

6. A patient begins taking cholestyramine (Questran) to treat hyperlipidemia. The patient reports abdominal discomfort and
constipation. The nurse will provide which instruction to the patient? A. Increase fluid and slowly increase fiber intake. B. Stop the
medication. C. Decrease oral fluids D. Give stool softeners
ANSWER: ________
RATIO:Cholestyramine can cause gastrointestinal upset and constipation, and these symptoms can be reduced with increased fluids
and foods high in fiber. Stopping the medication is not indicated. Over-the-counter laxatives are not recommended until other
methods have been tried. Giving the medication on an empty stomach will not relieve the discomfort.

7. The only statin with outcome data to show effectiveness in decreasing CAD and incidence of myocardial infarction.
A. atorvastatin (Lipitor)
B. simvastatin (Zocor)
C. pravastatin (Pravachol)
D. fluvastatin (Lescol)
ANSWER: ________
RATIO:Simvastatin can decrease the risk of cardiac and all-cause death and the recurrence of myocardial infarction (MI) in patients
with CHD.

8. Which of the following medical conditions will render antihyperlipidemics ineffective?


A. biliary obstruction
B. diabetes mellitus
C. both A and B
D. none of the above
ANSWER: ________
RATIO:Complete biliary obstruction. Prevent bile from being secreted into the intestines.

9. This drug works on the brush border of the intestines.


A. Ezetimibe
B. Pitavastatin
C. Gemfibrozil
D. Colestipol
ANSWER: ________
RATIO:Ezetimibe localizes to the brush border of the small intestine and blocks the uptake of cholesterol into the enterocytes. The
mechanism of action is still unknown, but it appears that ezetimibe blocks a putative transporter that controls the uptake of
cholesterol and related sterols into the enterocyte.

10. A patient who is taking a bile acid sequestrant complains of abdominal distention and nausea. What should the nurse do?
A. Document, withdraw drug, and notify doctor.
B. Provide comfort measures.
C. Prepare emergency equipment at bedside.
D. Dismiss the complaint.
ANSWER: ________
RATIO:GI discomforts as evidenced by abdominal distention, nausea, and vomiting are common complaints by patients taking bile
acid sequestrants. The nurse should provide comfort measures to help patients tolerate drug effects.

11. What vitamin plays a role in lowering cholesterol concentration?


A. Vitamin C
B. Vitamin E
C. Vitamin B3
D. Vitamin B2
ANSWER: ________
RATIO:Niacin, a B vitamin, has long been used to increase high-density lipoprotein (HDL) cholesterol — the "good" cholesterol that
helps remove low-density lipoprotein (LDL), the "bad" cholesterol from your bloodstream.

12. When should statins be taken to maximize its therapeutic effects?


A. After meals
B. During meals
C. At night
D. In the mornings
ANSWER: ________
RATIO:Most manufacturers of statins recommend that they are taken at night, on the basis of physiological studies which show that
most cholesterol is synthesized when dietary intake is at its lowest.

13. The doctor ordered atorvastatin for an obese client’s hyperlipidemia. This patient is receiving erythromycin for a toe infection at
the same time. The nurse knows that this combination should be avoided because?
A. It increases risk of atorvastatin toxicity.
B. It can result to potentially fatal lipolysis.
C. It can result to breakdown of muscles.
D. It decreases the effectiveness of erythromycin.
ANSWER: ________
RATIO:Combining these medications may significantly increase the blood levels of atorvastatin. This can increase the risk of side
effects such as liver damage and a rare but serious condition called rhabdomyolysis that involves the breakdown of skeletal muscle
tissue.

14. A patient has been taking atorvastatin (Lipitor) for several months to treat hyperlipidemia. The patient reports muscle weakness
and tenderness. The nurse will counsel the patient to A. Contact the provider to report these symptoms. B. Stop the medication C.
Change the medication with the same generic name. D. None of the above
ANSWER: ________ RATIO:Patients taking statins should report immediately any muscle aches or weakness, which can lead to
rhabdomyolysis, a muscle disintegration that can become fatal. All statins carry this risk, so changing to another statin is not
indicated. Ibuprofen may be useful, but notifying the provider is essential. Patients should not abruptly discontinue statins without
discussing this with the provider.

15. The nurse provides teaching to a patient who will begin taking simvastatin (Zocor) to treat hyperlipidemia. Which statement by
the patient indicates understanding of the teaching? A. "I will need an annual eye examination while taking this medication." B. "I will
need an annual ear examination while taking this medication." C. "I will need an annual breast examination while taking this
medication." D. "I will need an annual dental examination while taking this medication.

SAS 25

Multiple Choice
1. Which organ(s) produce(s) clotting factors?
A. Bone marrow
B. Spleen and other lymphoid tissues
C. Liver
D. Both B and C
ANSWER: __c______
RATIO:The coagulation factors (proteins) are manufactured by the liver. Ionized calcium ( Ca++ ) is available in the blood and from
intracellular sources.
2. What is the first reaction to a blood vessel injury?
A. Vasodilation and swelling
B. Vasoconstriction
C. Bleeding
D. Blood coagulation
ANSWER: __b______
RATIO:The first step in controlling blood loss is for the blood vessel to narrow (called constriction) so blood flow lessens. Within
seconds, the tiny platelets rush to the site of the injury and bunch together around the wound. They attract other platelets and help
form a plug to close up the break at the site.

3. Heparin is an anticoagulant. What family of anticoagulant medications does this drug belong to?
A. Direct thrombin inhibitors
B. Indirect thrombin inhibitors
C. Vitamin K antagonists
D. Factor Xa inhibitors
ANSWER: ________
RATIO:Heparin is an anticoagulant that belongs to the Indirect Thrombin Inhibitors family.

4. Which statement below BEST describes how Heparin works as an anticoagulant?


A. “It inhibits clotting factors from synthesizing Vitamin K.”
B. “It inactivates the extrinsic pathways of coagulation.”
C. “It prevents Factor Xa from activating prothrombin to fibrinogen.”
D. “It enhances the activation of antihrombin III, which prevents the activation of thrombin and the conversion of fibrinogen to
fibrin.”
ANSWER: ________
RATIO:Heparin enhances the activation of antihrombin III, which prevents the activation of thrombin and the conversion of
fibrinogen to fibrin. Therefore, this medication INDIRECTLY inhibits thrombin via antithrombin III.

5. Which patients below would be at a HIGH risk for developing adverse effects of Heparin drug therapy? Select all that apply:
A. A 55-year-old male patient who is post-op day 1 from brain surgery.
B. A 45-year-old female patient with a pulmonary embolism.
C. A 36-year-old male patient with active peptic ulcer disease.
D. A 43-year-old female with uncontrolled atrial fibrillation.
ANSWER: __A C______
RATIO:The answers are A and C. These patients are both at risk for major bleeding if placed on an anticoagulant due to their
condition (one patient is post-op from brain surgery and the other patient has ulcers that could bleed). Option B and D are
candidates from Heparin therapy because the patient in option B has a blood clot (Heparin can prevent it from getting bigger and
developing new blood clots), and the patient in option D is at risk for developing a blood clot.

6. A patient is ordered to start an IV continuous Heparin drip. Prior to starting the medication, the nurse would ensure what
information is gathered correctly before initiating the drip?
A. Vital signs
B. Weight
C. PT/INR level
D. EKG
ANSWER: ___B_____
RATIO:The answer is B. The nurse would want to make sure the documented weight of the patient is current and accurate. This
medication is weight-based. Therefore, for proper dosing to be administered, a correct weight should be used.

7. What is the approximate NORMAL level range for an activated partial thromboplastin time (aPTT)?
A. 20-25 seconds
B. 2-3 seconds
C. 30-40 seconds
D. 60-80 seconds
ANSWER: ___c_____
RATIO:The answer is C. This is considered a (approximate…varies in labs) normal aPTT level in someone who is NOT on Heparin.

8. A patient, who is receiving continuous IV Heparin, has an aPTT of 105 seconds. What is your next nursing action per protocol?
A. Continue with the infusion because no change is needed based on this aPTT.
B. Increase the drip rate per protocol because the aPTT is too low.
C. Re-draw the aPTT STAT.
D. Hold the infusion for 1 hour and decrease the rate per protocol because the aPTT is too high.
ANSWER: ____d____

RATIO:The answer is D. The aPTT is 105 seconds, which is too high. Any aPTT value greater than 80 seconds places the patient at risk
for bleeding. Most Heparin protocols dictate that the nurse would hold the infusion for 1 hour and to decrease the rate of infusion. If
the aPTT is less than 60 seconds, the dose would need to be increased and a bolus may be needed. aPTT values should be around 60-
80 seconds to achieve a therapeutic response for Heparin.

9. Select all the TRUE statements about the medication Heparin:


A. Heparin can be used during pregnancy.
B. Heparin has a short half-life.
C. Heparin works to affect the intrinsic pathways of clotting.
D. Heparin can be administered orally, intravenously, or subcutaneously.
ANSWER: ___ABC_____

RATIO:The answers are A, B, and C. The option that is wrong is D. Heparin can NOT be administered orally….only subq or IV.

10. Your patient is being discharged home and will be required to self-administer injectable Heparin. You are observing the patient
administer their scheduled dose of Heparin to confirm that the patient knows how to do it correctly. What action by the patient
requires you to re-educate them about how to administer Heparin?
A. The patient injects the needle into the fatty tissue of the abdomen.
B. The patient injects the needle 1 inch away from the umbilicus.
C. The patient rotated the injection site from the previous dose of Heparin.
D. The patient does not massage the injection site after administering the medication.
ANSWER: __b______
RATIO:The answer is B. The patient should inject the needle 2 inches (NOT 1 inch) away from the umbilicus. All the other options are
correct.

11. A patient is on a continuous IV Heparin drip. As the nurse you are monitoring for any adverse reactions. Select all the signs and
symptoms that would indicate this patient is having an adverse reaction to this medication: A. Hematuria B. Decreasing platelets C.
Increased blood glucose D. Low hemoglobin and hematocrit
E. Positive stool guaiac test
ANSWER: __abde______
RATIO:The answers are A, B, D and E. Hematuria, low hbg/hct and positive stool guaiac test all indicate the patient is bleeding. A
decrease in platelet level could indicate the patient is developing Heparin-induced thrombocytopenia, which is also an adverse
reaction to Heparin.

12. Your patient on Heparin develops Heparin-Induced Thrombocytopenia (HIT). What signs and symptoms in the patient confirm
this diagnosis? Select all that apply:
A. Decrease in platelet level
B. Increase in platelet level
C. Development of a new thrombus
D. Increase in hemoglobin level
ANSWER: ___AC_____
RATIO:The answers are A and C. HIT is where the body makes antibodies against Heparin because it’s binding to platelet factor 4 (a
blood protein). This creates antibodies that will bind to the heparin and PF4 complex, which activate the platelets. Small clots will
form (hence new clots or worsening of clots) and the platelet count falls...hence thrombocytopenia.

13. A patient develops Heparin-Induced Thrombocytopenia (HIT). As the nurse, you would expect the Heparin to be discontinued and
the patient to be started on what other type of anticoagulant?
A. Direct thrombin inhibitor
B. Protamine sulfate
C. Switched to subcutaneous Heparin injections
D. Vitamin-K agonist
ANSWER: ___A_____
RATIO:The Heparin is discontinued and direct thrombin inhibitors can be started like: Argatroban, Bivalirudin etc.

14. What is the antidote for Heparin? A. Protamine sulfate B. Vitamin K C. Flumazenil D. Narcan
ANSWER: ____A____
RATIO: Protamine sulfate is the antidote for Heparin.

15. You’re providing care to a patient who has been receiving long-term doses of Heparin. What finding in this patient demonstrates
the patient may be experiencing a complication that can occur due to long-term use of this drug? A. Uncontrolled hypertension B.
Bone fractures C. Hyperkalemia D. Raynaud’s Syndrome
ANSWER: __B______
RATIO:The answer is B. Osteoporosis can occur due to long-term, high doses of Heparin. Bone fractures would indicate this patient is
experiencing this complication. Heparin can stimulate osteoclasts and inhibits osteoblast, which affects the strength of the bones.

SAS 26

SAS# 26

1. As a nurse you know that Ferrous should not be given with:\


A. Dairy products
B. Antacids
C. Tetracyclines
D. All of the above.
ANSWER: D. ALL OF THE ABOVE
RATIO: it decreases the absorption

2. What do you give with Ferrous sulfate, ferrous fumarate, or ferrous gluconate to help promote absorption?
A. Vitamin C
B. Vitamin D
C. Vitamin E
D. Vitamin B12
ANSWER: A. VITAMIN C
RATIO : Helps promote absorption of iron

3. The client receives epoetin alfa (Epogen) subcutaneously, and says to the nurse, "My doctor said I have anemia. Are there little red
blood cells in that shot?" What are the best responses by the nurse?
A. "No, we do not give blood for anemia anymore."
B. "No, this medication stimulates your body to make red blood cells."
C. " Yes, this small amount of red blood cells will stimulate your bone marrow to produce more cells in the kidney."
D. "No, this medication promotes clotting so you will not lose even more red blood cells."
ANSWER: B .
RATIO: This medication is used to treat anemia (low red blood cell count) in people with long-term serious kidney disease (chronic
kidney failure), people receiving zidovudine to treat HIV, and people receiving chemotherapy for some types of cancer (cancer that
does not involve the bone marrow or blood cells).

4. The nurse is teaching a class on how red blood cell formation is regulated by the body to a group of clients who have AIDS. The
nurse evaluates that learning has occurred when the clients make which statements?
A. "Red blood cell formation is regulated through chemicals called colony-stimulating factors that come from white blood cells."
B. "Red blood cell formation is regulated through messages from the hormone, secretin, which is located in the kidney."
C. "Red blood cell formation is regulated through specific liver enzymes and a process called hemochromatosis."
D. "Red blood cell formation is regulated through messages from the hormone erythropoietin."
ANSWER: __D______
RATIO: Regulation of hematopoiesis occurs through messages from hormones such as erythropoietin.

5. 5. The client receives chemotherapy as therapy for cancer. The physician orders epoetin alfa (Procrit) subcutaneously. The client
asks the nurse if this drug is also chemotherapy. What is the best response by the nurse?
A. "No, but it works with your chemotherapy to make it more effective."
B. "No, this drug helps to counteract the nausea and vomiting caused by your chemotherapy." C. "No, it will stimulate your immune
system to help you battle the cancer."
D. "No, this drug will help prevent anemia that can be caused by your chemotherapy."
A. All of the above.
ANSWER: ___D_____
RATIO:_ Epoetin alfa (Procrit) is given to clients undergoing cancer chemotherapy to counteract the anemia caused by antineoplastic
agents.

6. The client is receiving medication for the treatment of anemia. The nurse has taught the client about this drug and about anemia.
The nurse evaluates that learning has occurred when the client makes which statement?
A. "My anemia could be caused by blood loss somewhere, but there are other causes too."
B. "My anemia was caused by drinking too many carbonated beverages with caffeine."
C. "There are many causes for anemia; mine was caused by heart failure and fluid overload."
D. "I think my anemia occurred when I started that vegetarian diet."
ANSWER: ____A____
RATIO: The three categories of blood loss are hemorrhage, increased erythrocyte destruction, and impaired erythrocyte production.

7. The client had stomach cancer and a surgical removal of his stomach several years ago. The physician prescribed cyanocobalamin
(Nascobal). The client stopped this drug several months ago. What will the nurse most likely assess in this client?
A. Memory loss, numbness in the limbs, and depression
B. A gradual decrease in red blood cell count
C. Jaundice and tarry stools
D. Low hemoglobin and hematocrit counts
ANSWER: ____A____
RATIO:The most common cause of vitamin B12 deficiency (pernicious anemia) is absence of intrinsic factor, a protein secreted by
stomach cells. This protein is required for vitamin B12 to be absorbed from the intestine. Symptoms of pernicious anemia involve the
nervous system and include memory loss, confusion, tingling or numbness in the limbs, and mood disturbances.

8. The client has chronic alcoholism. He asks the nurse why his doctor put him on folic acid (Folvite) since he promised the doctor
that he would stop drinking. What is the best response by the nurse?
A. "You should ask your doctor since you promised him that you would not drink anymore."
B. "You have been drinking instead of eating, and alcohol interferes with folate metabolism in your liver."
C. "You need folic acid to make up for the vitamin B12 deficiency that was caused by your alcoholism."
D. "You need folic acid because you have not been compliant with taking your vitamins and attending Alcoholics Anonymous (AA)
meetings."
ANSWER: ___B_____
RATIO: Insufficient folic acid can manifest itself as anemia. This is often observed in chronic alcoholism, since alcoholics consume
alcohol instead of eating nutritious foods. Alcohol interferes with folate metabolism in the liver.

9. The nurse teaches a class on iron-deficiency anemia to a group of pregnant clients. The nurse evaluates that additional learning is
needed when the clients make which statement?
A. "Most iron in our bodies is stored on hemoglobin in the red blood cell."
B. Transferrin is a protein that transports iron to places in our bodies where it is needed."
C. "We need extra iron because when our red blood cells die, all their iron is excreted from the body."
D. "The most common cause of nutritional anemia is iron deficiency."
ANSWER: ____C____
RATIO: After erythrocytes die, nearly all of the iron in their hemoglobin is incorporated into transferrin and recycled for later use.

10. The physician has prescribed epoetin alfa (Epogen) for the client. What is the priority assessment by the nurse?
A. The client's blood pressure
B. The client's report of a headache, indicating a stroke
C. The client's ability to use the proper injection techniques for self-administration
D. The client's hemoglobin and hematocrit levels
ANSWER: _____B___
RATIO: Clients are at risk for a stroke, but this is a complication of uncontrolled hypertension; blood pressure assessment is a priority.

11. The client is pregnant and has been told by her physician that she needs cyanocobalamin (Nascobal). She asks the nurse, "Will
this hurt my baby?" What is the best response by the nurse?
A. "No, this medication will not hurt your baby as long as you take it with ascorbic acid."
B. "No, this is safe as long as long as you take it in pill form; it is a Pregnancy Category A drug, which means it is safe for your baby."
C. "No, this medication will not hurt your baby as long as you take the pills only in the third trimester."
D. "No, this is safe in either pill or injectable form; it is a Pregnancy Category A drug which means it is safe for your baby."
ANSWER: ___B_____
RATIO: Cyanocobalamin (Nascobal), oral formulation, is a Pregnancy Category A drug, but it is a Pregnancy Category C when used
parenterally.

12. The client complains of constipation while receiving ferrous sulfate (Feosol). What is the best plan by the nurse to assist the client
in resolving this common side effect?
A. Plan to teach the client about which laxatives are the safest to use.
B. Plan to teach the client to increase fluids and high-fiber foods in the diet.
C. Plan to teach the client to self-administer Fleets enemas.
D. Plan to teach the client to increase exercise
ANSWER: ___B_____
RATIO: Constipation is a common side effect of ferrous sulfate; therefore, an increase in dietary fiber may be indicated. The nurse
should encourage a healthy diet that is packed with essential nutrients.

13. The client is receiving chemotherapy for cancer. The physician has prescribed oprelvekin (Neumega). The nurse has completed
medication education and evaluates it as effective when the client makes which statement?
A. "This medication will help my chemotherapy work better."
B. "This medication will help increase my platelet count."
C. "This medication will help me regain the weight I have lost."
D. "This medication will help increase my red blood cell count."
ANSWER: ________
RATIO: Oprelvekin (Neumega) is used to stimulate the production of platelets in clients who are at risk for thrombocytopenia caused
by cancer chemotherapy.

14. The client calls the nurse and is very frantic. "I think something is wrong! My stools are black, and they have never been this color
before!" The client is receiving ferrous sulfate (Feosol). What is the best response by the nurse?
A. "This is an expected side effect of ferrous sulfate (Feosol); it is okay."
B. "This sounds serious; you may have started bleeding again."
C. "Do you have hemorrhoids? That could be the problem."
D. "I will speak with your doctor and call you right back."
ANSWER: ____A____
RATIO: Ferrous sulfate (Feosol) will turn stools a harmless, dark green or black color; this is an expected side effect of the medication.

15. A nurse is preparing to administer ferrous sulfate IM to a client with anemia. What should the nurse consider when giving this
injection? Select all that apply.
A. Give the injection in the deltoid muscle.
B. Iron is best absorbed if given subcutaneously.
C. Iron is irritating to the tissues.
D. The z-track method should be used.
ANSWER: ____C and D____
RATIO: 1: The injection should be given deep IM in a larger muscle.
Rationale 2: Iron should be given deep IM.
Rationale 3: Iron is irritating to tissues.
Rationale 4: Z-track injection reduces leakage into the tissues and is the preferred method of IM injection of iron.
Rationale 5: There is no indication of need to use a large diameter needle for injection.

SAS 27

1. What is the only potassium-sparing diuretics that can be used in children?


A. Furosemide
B. Spironolactone
C. Amiloride
D. Triamterene
ANSWER: ___B_____
RATIO:Spironolactone is an inhibitor of the action of aldosterone, which enhances potassium secretion and sodium reabsorption by
the distal nephron

2. A patient receiving diuretics should alert the nurse if she feels the following:
A. insomnia
B. low-grade fever
C. muscle weakness
D. all of the above
ANSWER: __C______
RATIO:This is a sign of hypokalemia and would require prompt intervention. Other signs and symptoms of hypokalemia include
arrhythmia and muscle cramps.

3. A patient admitted for cerebral swelling complained of feeling light-headed and nauseous while receiving mannitol. What should
the nurse do?
A. Document, withdraw, and notify doctor.
B. Decrease mannitol flow.
C. Provide comfort measures.
D. Increase mannitol flow.
ANSWER: ____A____
RATIOA sudden drop in fluid level is one of the most potentially fatal adverse effects of mannitol. Manifestations include
hypotension, lightheadedness, nausea, and confusion.

4. The physician prescribes the patient a loop diuretic. As the nurse you know that this type of diuretic causes diuresis by mainly
affecting what structure in the nephron?
A. Distal convoluted tubule
B. Descending limb of the loop of Henle
C. Proximal convoluted tubule
D. Ascending limb of the loop of Henle
ANSWER: ___D_____
RATIO:_Loop diuretics affect the loop of Henle, specifically the ASCENDING limb (the thick part of this
limb)._____________________________________________________________________
5. Loop diuretics are effective with inhibiting sodium reabsorption within the nephron because it inhibits?
A. the sodium-chloride transporter
B. the effects of aldosterone on the distal convoluted tubule
C. the sodium-potassium-chloride cotransporter
D. the transport of bicarbonate by the proximal convoluted tubule
ANSWER: ____C____
RATIOThis inhibits the amount of sodium that is reabsorbed by the kidneys, which will cause the nephron to decrease the amount of
water it reabsorbs...hence leading to more water leaving the kidneys via the urine (leading to its diuretics affects).

6. Your patient is ordered a loop diuretic at 1000. Which finding below would require you to hold the dose and notify the physician
for further orders?
A. Calcium level 9 mg/L
B. Potassium level 1.5 mEq/L
C. Blood pressure 102/78
D. Sodium level 144
ANSWER: ___b_____
RATIO:Loop diuretics are known to decrease potassium levels due to the way they affect how the nephron wastes potassium (this
happens in the distal convoluted tubule due to the high concentrated amount of sodium in the filtrate

7. You’re providing discharge instructions to a patient who will be taking a loop diuretic at home. Which statement by the patient
demonstrates they did NOT understand the teaching material and requires that you reinforce some of the teaching points?
A. “I will eat a diet rich in potatoes, bananas, avocadoes, strawberries, and spinach.”
B. “I will weight myself daily and report to my physician if I gain more than 3 lbs in 1 day.”
C. “I will change position slowly because I can become dizzy easily while taking this medication.”
D. “This medication can cause dehydration, so I will stay hydrated by consuming at least 2 L or more of fluid per day.”
ANSWER: ___D_____
RATIO:The patient should be educated on the signs and symptoms of dehydration (excessive thirst, fatigue, hypotension etc.).
However, they should not be taught to consume 2L or more of fluid per day.
8. What electrolyte imbalances would the nurse monitor for in the patient who is taking a loop diuretic? Select all that apply:
A. Hyperkalemia
B. Hypocalcemia
C. Hypernatremia
D. Hypokalemia
ANSWER: ___D____
RATIO:_Loop diuretics can cause hypokalemia, hyponatremia, and hypomagnesemia.

____________________________________________________________________________________________________________
__________________________________
9. Which class of diuretics is indicated for the treatment of glaucoma?
A. Loop diuretics
B. Thiazide diuretics
C. Potassium-sparing diuretics
D. Carbonic anhydrase inhibitors
ANSWER: ________
RATIO:Diuretics (specifically the carbonic anhydrase inhibitors) are used as adjunctive treatment of chronic simple (open-angle)
glaucoma and secondary glaucoma Thiazide diuretics have off–label (non- FDA -approved) uses for osteoporosis in postmenopausal
women. They can be given alone or in combination with calcium or estrogen.

10. Which physiologic change will be observed in a patient after administering diuretics? Select all that apply.
A. Increased plasma volume
B. Decreased cardiac output
C. Decreased blood pressure
D. Increased extracellular fluid volume
E. Increased peripheral vascular resistance
ANSWER: __B and C______
RATIO:Anaphylactic shock is an allergic reaction, but massive in response. It is characterized by loss of plasma from capillaries as an
effect of histamine and a sudden drop in the intravascular blood volume and blood pressure.

11. A 10-year-old child who has congestive heart failure has been prescribed furosemide. Which instruction does the nurse give to
the child's parents to prevent adverse effects of the medication?
A. "Administer the medication on an empty stomach."
B. "Do not give oral rehydration drinks to your child."
C. "Give the medication to your child before bedtime."
D. "Protect your child from excessive sunlight exposure."
ANSWER: ________
RATIO:exposure to excessive sunlight may cause loss of fluids and electrolytes in the child and result in dehydration. Furosemide
should be given with meals or with milk to prevent gastrointestinal irritation.
.
12. A patient is prescribed chlorthalidone. What information will the nurse teach the patient?
A. "Take this medication on an empty stomach."
B. "Take this medication before bed each night."
C. "Wear protective clothing and sunscreen while taking this medication."
D. "Do not drink more than 10 ounces of fluid a day while on this medication."
ANSWER: __C______
RATIO: adverse effects include photosensitivity

13. Which drug may cause melena as a side effect?


A. Mannitol
B. Furosemide
C. Metolazone
D. Acetazolamide
ANSWER: ____D____
RATIO:Acetazolamide is a prescription medicine used to treat the symptoms of Epilepsy with occasional adverse reactions include
urticaria, melena, hematuria,

14. Which diuretic is the drug of choice for the prevention of tissue damage after a closed head injury?
A. Mannitol
B. Metolazone
C. Spironolactone
D. Hydrochlorothiazide
ANSWER: ____A____
RATIO:Mannitol (Osmitrol) is an effective hyperosmotic drug that is currently the agent ... Loss of sodium in the oral group was
corrected by the intravenous infusion Mannitol will slowly permeate the neural tissue, preventing the formation of an osmotic
gradient.

15. What acts to increase the reabsorption of water?


A. Aldosterone
B. Hypernatremia
C. Glomerular filtration
D. Antidiuretic hormone
ANSWER: __D___RATIO:Antidiuretic hormone binds to receptors on cells in the collecting ducts of the kidney and promotes
reabsorption of water back into the circulation.

SAS 28

1. A client is prescribed with Guaifenesin (Mucinex). The nurse determines that the client understands the proper administration of
this medication if the client states that he or she will:
A. Drink extra fluids while taking this medication.
B. Take the medication with meals only.
C. Take an additional dose once fever and cough persist.
D. Limit oral fluid intake.
ANSWER: ___A_____
RATIO: Increase fluid intake. Increase humidity and push fluids to decrease the problem of thickened secretions and dry nasal
mucosa.

2. A nurse is about to administer Albuterol (Ventolin HFA) 2 puff and Budesonide (Pulmicort Turbohaler) 2 puff by metered dose
inhaler. The nurse plans to administer by?
A. Alternating with a single puff each, starting with albuterol.
B. Alternating with a single puff each, starting with budesonide.
C. Budesonide inhaler first then the albuterol.
D. Albuterol inhaler first then the budesonide
ANSWER: ___D_____
RATIO: If two different inhaled medications are prescribed and one of the medications contains a corticosteroid, administer the
bronchodilator (Albuterol) first and the corticosteroid (Budesonide) second. This will allow for the widening of the air passages by the
bronchodilator, making the corticosteroids more effective.

3. A nurse is about to administer Naloxone hydrochloride (Narcan) to a client with known opioid overdose. Which of the following
equipment should be readily available at the bedside?
A. Suction machine.
B. Resuscitative equipment.
C. Nasogastric tube.
D. Dressing tray.
ANSWER: ___B_____
RATIO: While administering Narcan, resuscitation equipment, oxygen, mechanical ventilator should be readily available. Options A, C,
and D are not used during the medication therapy.

4. A client with a chronic obstructive pulmonary disease is prescribed with Ipratropium (Combivent). Upon reviewing the medical
history of the patient, the nurse questions the prescription if which of the following is noted?
A. History of smoking.
B. History of allergy to egg.
C. History of allergy to peanut.
D. History of a previous infection.
ANSWER: ____C____
RATIO: The client with a peanut allergy should not take ipratropium because the product contains soy lecithin, which is in the same
plant family as peanuts.

5. Montelukast (Singulair) is prescribed to a client with asthma. During the medication therapy, which of the following laboratories
should be monitored?
A. Complete blood count (CBC).
B. Sodium and Potassium.
C. Calcium and Platelet count.
D. ALT and AST.
ANSWER: ___D_____
RATIO: Montelukast (Singulair) is a leukotriene receptor and is used to relieve allergy symptoms and is also used to prevent asthma
attacks. Alanine aminotransferase (ALT) and Aspartate aminotransferase (AST) should be monitored while taking this medication.

6. The nurse is giving medication teachings to a client receiving theophylline. The nurse instruct the client to limit the intake of which
of the following?
A. Apple and banana.
B. Yogurt and cheese.
C. Tuna and oysters.
D. Cola and chocolate.
ANSWER: ___D_____
RATIO: Theophylline is a methylxanthine bronchodilator. The nurse instructs the client to limit the intake of xanthine-containing foods
such as chocolate, cola, and coffee.

7. A nurse is giving teachings to a client receiving Desloratadine (Clarinex). Which of the following statements made by the client will
need further instructions?
A. “I can eat gum after I drink the medicine”.
B. “I can take the medicine on an empty stomach”.
C. “I should avoid using alcohol”.
D. “I will avoid driving while using this medication”.
ANSWER: ___B_____
RATIO: This medicine should be taken with food or milk to minimize gastrointestinal upset. Option A: Use gum or hard candy to
minimize dry mouth. Options C and D: The medication causes drowsiness so avoid taking alcohol or engaging in activities which
require mental alertness such as driving a car.

8. Antitussives are useful in blocking the cough reflex and preserving the energy associated with prolonged, nonproductive coughing.
Antitussives are best used with the following except:
A. Postoperative patients
B. COPD patients who tire easily
C. Patients with a dry, irritating cough
D. Asthma patients
ANSWER: ___D_____
RATIO: Patients with asthma and emphysema are contraindicated because cough suppression could lead to accumulation of
secretions and a loss of respiratory reserve.

9. Which of the following pathophysiological mechanisms that occur in the lung parenchyma allows pneumonia to develop?
A. Atelectasis
B. Bronchiectasis
C. Effusion
D. Inflammation
ANSWER: ___D_____
RATIO: The common feature of all types of pneumonia is an inflammatory pulmonary response to the offending organism or agent.
Atelectasis & bronchiectasis indicate a collapse of a portion of the airway that doesn't occur in pneumonia.
An effusion is an accumulation of excess pleural fluid in the pleural space, which may be a secondary response to pneumonia.

10. A nurse is giving teachings to a client receiving Desloratadine (Clarinex). Which of the following statements made by the client
will need further instructions?
A. “I can eat gum after I drink the medicine”
B. “I can take the medicine on an empty stomach“
C. “I should avoid using alcohol”
D. “I will avoid driving while using this medication”
ANSWER: ___B_____
RATIO: This medicine should be taken with food or milk to minimize gastrointestinal upset. Option A: Use gum or hard candy to
minimize dry mouth. Options C and D: The medication causes drowsiness so avoid taking alcohol or engaging in activities which
require mental alertness such as driving a car.

11. Nurse Zeke is giving instructions to her client who is taking antihistamine. Which of the following nurse teachings is appropriate
for the client?
A. Expect a relief in 24 hours
B. Be aware that you may have increased saliva
C. Be aware that you may need to take a decongestant
D. Avoid ingesting alcohol
ANSWER: __D______
RATIO: Avoid alcohol. Caution the patient to avoid alcohol while taking these drugs because serious sedation can occur.

12. Andrew has vertigo, which antihistamine is best for his condition?
A. Terfenadine
B. Guaifenesin
C. Meclizine
D. Hydrocodone
ANSWER: ___C_____
RATIO: Doctors recommend antihistamines, such as Antivert (meclizine), Benadryl (diphenhydramine), or Dramamine
(dimenhydrinate) to help vertigo episodes. Meclizine hydrochloride was shown to be significantly more effective than placebo in
treating patients with positional and continuous vertigo of vestibular origin. Meclizine reduced the severity and frequency of attacks,
as well as signs and symptoms associated with the vertigo.

13. Raul, a 20-year-old student, used to buy OTC drugs whenever he feels sick. Which of the following statements best describes the
danger of self-medication with over-the-counter drugs?
A. Clients are not aware of the action of over-the-counter drugs.
B. Clients are not aware of the side effects of over-the-counter drugs.
C. Clients minimize the effects of over-the-counter drugs because they are available without a prescription.
D. Clients do not realize the effects of over-the-counter drugs.
ANSWER: ____B____
RATIO: Avoid OTC drugs. Caution the patient to avoid excessive dose and to check OTC drugs for the presence of antihistamines,
which are found in many OTC preparations and could cause toxicity.

14. Which histamine-2 antagonist is associated with the most drug interactions?
A. Prilosec
B. Nizatidine
C. Ranitidine
D. Cimetidine
ANSWER: ___C_____
RATIO: H2 Blockers or H2 Antagonists are used to reduce gastric acid in PUD Examples: cimetidine (Tagamet), ranitidine (Zantac), or
famotidine (Pepcid)

15. Stephanie will be having her exam in pharmacology tomorrow. She should be aware that antitussive is indicated to:
A. Encourage removal of secretions through coughing
B. Relieve rhinitis
C. Relieve a dry cough
D. Control a productive cough
ANSWER: __A______
RATIO: An antitussive is a cough suppressant. Choices A and C describe the action of an expectorant. Choice D describes the action of
a decongestant.

SAS 29

1. A client has been taking Ibuprofen for some quite time and was given Misoprostol (Cytotec). Which of the following is exhibiting
the therapeutic effect of Cytotec?
A. Relief of gastric ulcer.
B. Relief of diarrhea.
C. Relief of vomiting.
D. Relief of constipation.
ANSWER: ____A____
RATIO:Misoprostol (Cytotec) is a synthetic (man-made) prostaglandin that is used to reduce the risk of stomach ulcers in patients
treated with nonsteroidal antiinflammatory drugs (NSAIDs, for example, aspirin, ibuprofen, etc.).

2. A geriatric patient is prescribed with Cimetidine (Tagamet) for the treatment of heartburn. Which of the following is the most
frequent CNS side effect?
A. Agitation.
B. Drowsiness.
C. Headache.
D. Somnolence
ANSWER: ____A____
RATIO:Cimetidine an H2-receptor antagonist passes the blood brain barrier, and central nervous system side effects can happen.
Most common serious side effects are confusion, agitation, depression, and disorientation. Options B, C, and D: They are considered
as the less common side effect.

3. A nurse is administering an IV bolus of Cimetidine (Tagamet). Which of the following should the nurse monitor closely follow the
administration?
A. Respiratory rate.
B. Skin turgor.
C. Blood pressure.
D. Temperature.
ANSWER: ___C_____
RATIO:Intravenous administration of Cimetidine causes hypotension. Options A, B, and D are not related to this medication.

4. A client has been given Loperamide hydrochloride (Imodium). Which of the following situation is the medication indicated to?
A. Abdominal pain.
B. Patients with an ileostomy.
C. Bloody Diarrhea.
D. Acute dysentery.
ANSWER: ___B_____
RATIO:Loperamide hydrochloride is an antidiarrheal agent. It can also be used to reduce the volume of drainage from an ileostomy.
Options A, C, and D: It is contraindicated in patients with abdominal pain in the absence of diarrhea, and in patients with acute
dysentery, which is characterized by blood in stools and high fever.

5. A client has been given Ondansetron (Zofran). For which condition should the nurse administer this medication to the
postoperative patient?
A. Vomiting.
B. Incisional pain.
C. Abdominal infection.
D. Atelectasis.
ANSWER: ___A_____
RATIO:Ondansetron is used to prevent nausea and vomiting that may be caused by surgery or by medicine to treat cancer
(chemotherapy or radiation). Options B, C, and D, are not related to this medication.

6. A client with a duodenal ulcer is diagnosed with H. pylori infection. The physician prescribed Amoxicillin (Wymox), Pantoprazole
(Prevacid), and Clarithromycin (Biaxin). Which statement made by the nurse correctly explains the purpose of these medications?
A. “These medicines will minimize acid production and will coat the ulcer”.
B. “These medicines will stop the acid production and will kill the bacteria”.
C. “The ulcer will heal because the medications will kill the bacteria”.
D. “These medicines will control the ulcer and motion sickness”.
ANSWER: ___B_____
RATIO:The triple therapy treatment of H-pylori infection is the includes 2 antibiotics (Clarithromycin and Amoxicillin) and one proton
pump inhibitor such as omeprazole, lansoprazole, pantoprazole, or esomeprazole.
7. A client is prescribed with Omeprazole (Prilosec). The nurse determines that the client is receiving its therapeutic effect if which of
the following is stated by the client:
A. Relief of nausea and vomiting.
B. Decrease diarrheal episodes.
C. The absence of constipation.
D. Relief from GERD.
ANSWER: ___D_____
RATIO:Omeprazole is used to treat symptoms of gastroesophageal reflux disease (GERD) and other conditions caused by excess
stomach acid. It is also used to promote healing of erosive esophagitis (damage to your esophagus caused by stomach acid). Options
B, C, and D are not related to this medication.

8. A client has been prescribed with Pancrelipase (Pancrease). Which of the following symptoms would prompt the nurse that the
medication is having its therapeutic effect if which of the following is noted?
A. Negative abdominal pain.
B. An absence of constipation.
C. An absence of diarrhea.
D. Reduction of excess fat in feces.
ANSWER: ___D_____
RATIO:Pancrelipase is used to help improve food digestion in certain conditions (cystic fibrosis, pancreatitis) where the pancreas is
not working properly. This medicine minimizes the amount of steatorrhea (fatty stools). Options A, B, and C are not related to this
medication.

9. A nurse is giving a nothing per orem instructions to a malnourished client with diarrhea and frequent abdominal pain episodes
which is about to receive a Total Parenteral Nutrition. Which statement made by the nurse is the most appropriate?
A. “It will help in your weight loss”.
B. “It can assure you that you feel better after receiving TPN”.
C. “It will decrease your diarrhea and your bowel can rest”.
D. “It will give you less time in the hospital”.
ANSWER: ___C_____
RATIO:The priority in this kind of situation is to stop diarrhea and to provide fluids and electrolyte thru the use of TPN. The bowel is
rested so that the abdominal cramping will also stop.

10. A client is receiving Sulfasalazine (Azulfidine) for the treatment of ulcerative colitis. Which of the following assessment finding will
concern the nurse most?
A. Drowsiness.
B. Decreased urine output.
C. Urine discoloration.
D. Vomiting.
ANSWER: __B______
RATIO:Sulfasalazine is used to treat bowel inflammation, diarrhea (stool frequency), rectal bleeding, and abdominal pain in patients
with ulcerative colitis. It is nephrotoxic, so a decrease in urine output is the most serious concern.

11. A client has been prescribed with Sucralfate (Carafate) for the treatment of gastric ulcer. The nurse instruct the client that this
medication is taken?
A. 1 hour before meals.
B. 1 hour after meals.
C. At the same time with an antacid.
D. Lunch time.
ANSWER: __A______
RATIO:Sucralfate is a gastric protective agent. It works by forming a protective layer on the ulcer to serve as a barrier against acid, bile
salts, and enzymes in the stomach. Taken by mouth on an empty stomach at least 1 hour before or 2 hours after eating. Options B
and D are incorrect. Option C: Do not take an antacid for at least 30 minutes before or after taking sucralfate.

12. Tincture of opium is given to a patient who is having diarrheal episodes. Which of the following is true regarding this medication?
A. Opium tincture is not a controlled substance.
B. Opium tincture can be used with medications like naltrexone and buprenorphine.
C. Has an unpleasant taste and it can be diluted with 15-30 ml water.
D. It increases intestinal motility and peristalsis.
ANSWER: ___C_____
RATIO:Opium tincture is an oral liquid medication used to control diarrhea. It has an unpleasant taste so it should be diluted with 15-
30 ml of water.

13. A nurse is giving medicine instructions to a client with hemorrhoids who is receiving a Mineral oil. Which of the following
statements made by the client indicates a further teaching?
A. “I can take it at least 2 hours before bedtime”.
B. “It can interfere with the absorption of the vitamin E that I am taking”.
C. “If I miss a dose of mineral oil liquid, I’ll take it as soon as I remember”.
D. “I can use mineral oil liquid for an extended period to prevent further damage”.
ANSWER: ____D____
RATIO:Mineral oil liquid is a lubricant laxative that works by slowing the absorption of water from the bowel, which softens the stool.
The use of mineral oil liquid for a long time may result in loss of normal bowel function. Options A, B, and C are true regarding
mineral oil.

14. A nurse is giving discharge instructions to a client who is receiving a bulk-forming laxative as part of the home medications. All of
which are examples of bulk-forming laxative, except?
A. Docusate Sodium (Colace).
B. Methylcellulose (Citrucel).
C. Polycarbophil (Fibercon).
D. Psyllium (Metamucil).
ANSWER: ___A_____
RATIO:Option A is an example of surfactant laxative.

15. What is the priority nursing intervention for a client receiving an antiemetic?
A. Monitor intake and output.
B. Keep items far away from the bed.
C. Give the client privacy by letting him walking around the room.
D. Keep bed in low position with side rails up.
ANSWER: ____D____
RATIO:Antiemetics can cause drowsiness; hence the priority nursing intervention is to protect the client from injury such as raising
the side rails and keeping the bed in low position. Option A is not the most priority intervention. Options B and C will risk the client
for fall.

SAS 30

1. Nurse Ejay is assigned to a telephone triage. A client called who was stung by a honeybee and is asking for help. The client reports
of pain and localized swelling but has no respiratory distress or other symptoms of anaphylactic shock. What is the appropriate initial
action that the nurse should direct the client to perform?
A. Removing the stinger by scraping it.
B. Applying a cold compress.
C. Taking an oral antihistamine.
D. Calling emergency number.
ANSWER: ____A____
RATIO:Since the stinger will continue to release venom into the skin, removing the stinger should be the first action that the nurse
should direct to the client. Options B and C: After removing the stinger, Antihistamine and cold compress follow. Option D: The caller
should be further advised about symptoms that require 911 assistance.

2. An osmotic diuretic used in emergency, trauma, critical care, and neurosurgical settings to treat cerebral edema and to reduce
increased intracranial pressure.
A. Mannitol
B. Lidocaine
C. Furosemide
D. Albuterol
ANSWER: _A_______
RATIO: Mannitol is a sugar alcohol solution which is sometimes effective in reducing brain swelling after head injury.

3. A client arrives in the emergency unit and reports that a concentrated household cleaner was splashed in both eyes. Which of the
following nursing actions is a priority?
A. Use Restasis (Allergan) drops in the eye.
B. Flush the eye repeatedly using sterile normal saline.
C. Examine the client’s visual acuity.
D. Patch the eye.
ANSWER: ___B_____

RATIO:Initial emergency action during a chemical splash to the eye includes immediate continuous irrigation of the affected eye with
normal saline. Option A: Restasis (Allergan) drops are used to treat dry eyes. Option C: Patching the eye is not part of the first line
treatment of a chemical splash. Option D: After irrigation, visual acuity then is assessed.

4. A 5-year-old client was admitted to the emergency unit due to ingestion of unknown amount of chewable vitamins for children at
an unknown time. Upon assessment, the child is alert and with no symptoms. Which of the following information should be reported
to the physician immediately?
A. The child has been treated multiple times for injuries caused by accidents.
B. The vitamin that was ingested contains iron.
C. The child was nauseated and vomited once at home.
D. The child has been treated several times for toxic substance ingestion
ANSWER: ____B____
RATIO:Iron is a toxic substance that can lead to massive hemorrhage, shock, coma, and kidney failure. Options A, C, and D: These
information needs further investigation but will not change the immediate diagnostic testing or treatment plan.

5. Which of the following is a concentrated, high-carbohydrate solution given to treat insulin-induced hypoglycemia or insulin shock?
A. Dextrose 50%
B. Glocagon
C. Mannitol
D. Sodium bicarbonate
ANSWER: ________
RATIO:Dextrose 50%: concentrated, high-carbohydrate solution given to treat insulin-induced hypoglycemia or
insulin shock
Glucagon: pancreas-produced hormone that elevates blood sugar by stimulating glycogenolysis

6. Epinephrine is administered to a female patient. The nurse should expect this agent to rapidly affect:
A. Adrenergic receptors.
B. Muscarinic receptors.
C. Cholinergic receptors.
D. Nicotinic receptors.
ANSWER: ___A_____
RATIO:Acetaminophen is extensively metabolized by pathways in the liver. Toxic doses of acetaminophen deplete hepatic
glutathione. resulting in accumulation of the intermediate agent. quinine. which leads to hepatic
necrosis._____________________________________________________________________

7. An anxious female client complains of chest tightness, tingling sensations, and palpitations. Deep, rapid breathing, and carpal
spasms are noted. Which of the following priority action should the nurse do first?
A. Provide oxygen therapy.
B. Notify the physician immediately.
C. Administer anxiolytic medication as ordered.
D. Have the client breathe into a brown paper bag.
ANSWER: ___D_____
RATIO:The client is suffering from hyperventilation secondary from anxiety, the initial action is to let the client breathe in a paper bag
that will allow the rebreathing of carbon dioxide.

8. An intoxicated client comes into the emergency unit with an uncooperative behavior, mild confusion, and with slurred speech. The
client is unable to provide a good history but he verbalizes that he has been drinking a lot. Which of the following is a priority action
of the nurse?
A. Administer IV fluid incorporated with Vitamin B1 as ordered.
B. Administer Naloxone (Narcan) 4 mg as ordered.
C. Contact the family to get information of the client.
D. Obtain an order for the determination of blood alcohol level
ANSWER: __A______
RATIO:The client has symptoms of alcohol abuse and there is a risk for Wernicke syndrome, which is caused by a deficiency in Vitamin
B. Option B: Multiple drug abuse is not uncommon; however, there is currently nothing to suggest an opiate overdose that requires
the administration of naloxone. Options C and D: Additional information or the results of the blood alcohol testing are part of the
management but should not delay the immediate treatment.

8. A nurse is providing discharge instruction to a woman who has been treated for contusions and bruises due to a domestic
violence. What is the priority intervention for this client?
A. Making a referral to a counselor.
B. Making an appointment to follow up on the injuries.
C. Advising the client about contacting the police.
D. Arranging transportation to a safe house.
ANSWER: __D______
RATIO:Safety is a priority for this client and she should not return to a place where violence could recur. Options A, B, and C: These
are important for the long-term management of this case.

9. In the work setting, what is the primary responsibility of the nurse in preparation for disaster management, that includes natural
disasters and bioterrorism incidents?
A. Being aware of the signs and symptoms of potential agents of bioterrorism.
B. Making ethical decisions regarding exposing self to potentially lethal substances.
C. Being aware of the agency’s emergency response plan.
D. Being aware of what and how to report to the Centers for Disease Control and Prevention.
ANSWER: ___C_____
RATIO:In disasters preparedness, the nurse should know the emergency response plan. This gives guidance that includes the roles of
the team members, responsibilities and mechanism of reporting.

10. Early signs and symptoms of local anesthetic toxicity include ALL BUT ONE of the following. Indicate the exception:
A. Tinnitus
B. Perioral numbness
C. Dizziness
D. Hypertension
ANSWER: ___D_____
RATIOManifestations of local anesthetic toxicity typically appear 1-5 minutes after the injection, but onset may range from 30
seconds to as long as 60 minutes. Initial manifestations may also vary widely.
11. Which of the following is udes to treat significant ventricular dysrhythmias?
A. Lidocaine
B. Mannitol
C. Naloxone
D. Dopamine
ANSWER: __A______
RATIO:Lidocaine is a class IB antiarrhythmic drug used in the treatment of ventricular arrhythmias, specifically ventricular
tachycardia and ventricular fibrillation. Lidocaine blocks cardiac sodium channels shortening the action potential and is used
intravenously only for arrhythmia.
12. A patient presents to the emergency department with severe respiratory distress, hives, and edema after being stung on the face
by a bee. Which are accurate nursing assessments of the situation?
A. Hypotension and bronchospasm will progress rapidly if treatment is delayed.
B. The patient is suffering from anaphylactic shock.
C. Epinephrine and diphenhydramine are the drugs of choice as first-line agents.
D. Prompt treatment with drug therapy will prevent this syndrome from occurring again.
ANSWER: ____ABC____
RATIO:Treatment will not prevent anaphylaxis from occurring again. All other statements are true.
13. Which finding indicates that administration of glucagon has been effective? The patient experiences
A. an improvement in level of consciousness.
B. an elevation in respiratory rate.
C. suppression of heart rate.
D. reduction in blood pressure.
ANSWER: ___a____
RATIO: Glucagon is used for treatment of patients with severe hypoglycemia. An improved level of consciousness indicates elevation
of blood sugar. A blood sugar analysis should also be obtained

14. A patient is experiencing both atrial and ventricular dysrhythmias. The nurse anticipates administration of which medication?
A. Adenosine
B. Amiodarone
C. Atropine
D. Epinephrine
ANSWER: ________
RATIO:Amiodarone is used for the treatment of atrial and ventricular dysrhythmias. Adenosine is used for the treatment of PSVT.
Atropine is used for the treatment of symptomatic bradycardia. Epinephrine is used for the treatment of cardiac arrest.

15. A dopamine infusion was started in a patient's antecubital vein during resuscitation after cardiac arrest. The electronic infusion
device is now sounding an alert for an occlusion. What is the most important immediate concern for the nurse?
A. Infiltration with phentolamine will be necessary if there is extravasation
B. An interruption in the infusion can produce hypotension in the patient.
C. The device will need to be reported to the hospital's clinical engineering department for service
D. The patient could develop hypertension as a result of the alarm.
ANSWER: ________
RATIO: Dopamine (dopamine hydrochloride) is a catecholamine drug that acts by inotropic effect on the heart muscle (causes more
intense contractions) that, in turn, can raise blood pressure. At high doses, Dopamine may help correct low blood pressure due to
low systemic vascular resistance.

SAS 30
SAS # 30
1. Nurse Ejay is assigned to a telephone triage. A client called who was stung by a honeybee and is asking for help. The client reports
of pain and localized swelling but has no respiratory distress or other symptoms of anaphylactic shock. What is the appropriate initial
action that the nurse should direct the client to perform?
A. Removing the stinger by scraping it.
B. Applying a cold compress.
C. Taking an oral antihistamine.
D. Calling emergency number.
ANSWER: ____A____
RATIO:Since the stinger will continue to release venom into the skin, removing the stinger should be the first action that the nurse
should direct to the client. Options B and C: After removing the stinger, Antihistamine and cold compress follow. Option D: The caller
should be further advised about symptoms that require 911 assistance.
2. An osmotic diuretic used in emergency, trauma, critical care, and neurosurgical settings to treat cerebral edema and to reduce
increased intracranial pressure.
A. Mannitol
B. Lidocaine
C. Furosemide
D. Albuterol
ANSWER: _A_______
RATIO: Mannitol is a sugar alcohol solution which is sometimes effective in reducing brain swelling after head injury.

3. A client arrives in the emergency unit and reports that a concentrated household cleaner was splashed in both eyes. Which of the
following nursing actions is a priority?
A. Use Restasis (Allergan) drops in the eye.
B. Flush the eye repeatedly using sterile normal saline.
C. Examine the client’s visual acuity.
D. Patch the eye.
ANSWER: ___B_____

RATIO:Initial emergency action during a chemical splash to the eye includes immediate continuous irrigation of the affected eye with
normal saline. Option A: Restasis (Allergan) drops are used to treat dry eyes. Option C: Patching the eye is not part of the first line
treatment of a chemical splash. Option D: After irrigation, visual acuity then is assessed.

4. A 5-year-old client was admitted to the emergency unit due to ingestion of unknown amount of chewable vitamins for children at
an unknown time. Upon assessment, the child is alert and with no symptoms. Which of the following information should be reported
to the physician immediately?
A. The child has been treated multiple times for injuries caused by accidents.
B. The vitamin that was ingested contains iron.
C. The child was nauseated and vomited once at home.
D. The child has been treated several times for toxic substance ingestion
ANSWER: ____B____
RATIO:Iron is a toxic substance that can lead to massive hemorrhage, shock, coma, and kidney failure. Options A, C, and D: These
information needs further investigation but will not change the immediate diagnostic testing or treatment plan.

5. Which of the following is a concentrated, high-carbohydrate solution given to treat insulin-induced hypoglycemia or insulin shock?
A. Dextrose 50%
B. Glocagon
C. Mannitol
D. Sodium bicarbonate
ANSWER: ____A____
RATIO:Dextrose 50%: concentrated, high-carbohydrate solution given to treat insulin-induced hypoglycemia or
insulin shock
Glucagon: pancreas-produced hormone that elevates blood sugar by stimulating glycogenolysis

6. Epinephrine is administered to a female patient. The nurse should expect this agent to rapidly affect:
A. Adrenergic receptors.
B. Muscarinic receptors.
C. Cholinergic receptors.
D. Nicotinic receptors.
ANSWER: ___A_____
RATIO:Acetaminophen is extensively metabolized by pathways in the liver. Toxic doses of acetaminophen deplete hepatic
glutathione. resulting in accumulation of the intermediate agent. quinine. which leads to hepatic necrosis.

7. An anxious female client complains of chest tightness, tingling sensations, and palpitations. Deep, rapid breathing, and carpal
spasms are noted. Which of the following priority action should the nurse do first?
A. Provide oxygen therapy.
B. Notify the physician immediately.
C. Administer anxiolytic medication as ordered.
D. Have the client breathe into a brown paper bag.
ANSWER: ___D_____
RATIO:The client is suffering from hyperventilation secondary from anxiety, the initial action is to let the client breathe in a paper bag
that will allow the rebreathing of carbon dioxide.

8. An intoxicated client comes into the emergency unit with an uncooperative behavior, mild confusion, and with slurred speech. The
client is unable to provide a good history but he verbalizes that he has been drinking a lot. Which of the following is a priority action
of the nurse?
A. Administer IV fluid incorporated with Vitamin B1 as ordered.
B. Administer Naloxone (Narcan) 4 mg as ordered.
C. Contact the family to get information of the client.
D. Obtain an order for the determination of blood alcohol level
ANSWER: __A______
RATIO:The client has symptoms of alcohol abuse and there is a risk for Wernicke syndrome, which is caused by a deficiency in Vitamin
B. Option B: Multiple drug abuse is not uncommon; however, there is currently nothing to suggest an opiate overdose that requires
the administration of naloxone. Options C and D: Additional information or the results of the blood alcohol testing are part of the
management but should not delay the immediate treatment.

8. A nurse is providing discharge instruction to a woman who has been treated for contusions and bruises due to a domestic
violence. What is the priority intervention for this client?
A. Making a referral to a counselor.
B. Making an appointment to follow up on the injuries.
C. Advising the client about contacting the police.
D. Arranging transportation to a safe house.
ANSWER: __D______
RATIO:Safety is a priority for this client and she should not return to a place where violence could recur. Options A, B, and C: These
are important for the long-term management of this case.

9. In the work setting, what is the primary responsibility of the nurse in preparation for disaster management, that includes natural
disasters and bioterrorism incidents?
A. Being aware of the signs and symptoms of potential agents of bioterrorism.
B. Making ethical decisions regarding exposing self to potentially lethal substances.
C. Being aware of the agency’s emergency response plan.
D. Being aware of what and how to report to the Centers for Disease Control and Prevention.
ANSWER: ___C_____
RATIO:In disasters preparedness, the nurse should know the emergency response plan. This gives guidance that includes the roles of
the team members, responsibilities and mechanism of reporting.

10. Early signs and symptoms of local anesthetic toxicity include ALL BUT ONE of the following. Indicate the exception:
A. Tinnitus
B. Perioral numbness
C. Dizziness
D. Hypertension
ANSWER: ___D_____
RATIOManifestations of local anesthetic toxicity typically appear 1-5 minutes after the injection, but onset may range from 30
seconds to as long as 60 minutes. Initial manifestations may also vary widely.
11. Which of the following is udes to treat significant ventricular dysrhythmias?
A. Lidocaine
B. Mannitol
C. Naloxone
D. Dopamine
ANSWER: __A______
RATIO:Lidocaine is a class IB antiarrhythmic drug used in the treatment of ventricular arrhythmias, specifically ventricular tachycardia
and ventricular fibrillation. Lidocaine blocks cardiac sodium channels shortening the action potential and is used intravenously only
for arrhythmia.
12. A patient presents to the emergency department with severe respiratory distress, hives, and edema after being stung on the face
by a bee. Which are accurate nursing assessments of the situation?
A. Hypotension and bronchospasm will progress rapidly if treatment is delayed.
B. The patient is suffering from anaphylactic shock.
C. Epinephrine and diphenhydramine are the drugs of choice as first-line agents.
D. Prompt treatment with drug therapy will prevent this syndrome from occurring again.
ANSWER: ____ABC____
RATIO:Treatment will not prevent anaphylaxis from occurring again. All other statements are true.
13. Which finding indicates that administration of glucagon has been effective? The patient experiences
A. an improvement in level of consciousness.
B. an elevation in respiratory rate.
C. suppression of heart rate.
D. reduction in blood pressure.
ANSWER: ___a____
RATIO: Glucagon is used for treatment of patients with severe hypoglycemia. An improved level of consciousness indicates elevation
of blood sugar. A blood sugar analysis should also be obtained

14. A patient is experiencing both atrial and ventricular dysrhythmias. The nurse anticipates administration of which medication?
A. Adenosine
B. Amiodarone
C. Atropine
D. Epinephrine
ANSWER: ___B_____
RATIO:Amiodarone is used for the treatment of atrial and ventricular dysrhythmias. Adenosine is used for the treatment of PSVT.
Atropine is used for the treatment of symptomatic bradycardia. Epinephrine is used for the treatment of cardiac arrest.

15. A dopamine infusion was started in a patient's antecubital vein during resuscitation after cardiac arrest. The electronic infusion
device is now sounding an alert for an occlusion. What is the most important immediate concern for the nurse?
A. Infiltration with phentolamine will be necessary if there is extravasation
B. An interruption in the infusion can produce hypotension in the patient.
C. The device will need to be reported to the hospital's clinical engineering department for service
D. The patient could develop hypertension as a result of the alarm.
ANSWER: _____B___
RATIO: Dopamine (dopamine hydrochloride) is a catecholamine drug that acts by inotropic effect on the heart muscle (causes more
intense contractions) that, in turn, can raise blood pressure. At high doses, Dopamine may help correct low blood pressure due to
low systemic vascular resistance.
The nurse is providing education to a patient on the primary uses of cholinergic drugs. Which do you think the nurse should NOT
include in the teaching? *
2/2

Decrease intraocular pressure


Dilate pulmonary airways

Stimulate bladder emptying


Stimulate peristalsis

The nurse teaches the patient receiving atropine (Atreza) to expect which side effect?
2/2

Diarrhea
Bradycardia
Blurred vision

Frequent urination

A patient has been taking atorvastatin (Lipitor) for several months to treat hyperlipidemia. The patient reports muscle weakness
and tenderness. The nurse will counsel the patient to *
2/2

Contact the provider to report these symptoms.

Stop the medication


Change the medication with the same generic name.
None of the above

Patient Kim Soo has chronic alcoholism. He asks the nurse why his doctor put him on folic acid (Folvite) since he promised the
doctor that he would stop drinking. What is the best response by the nurse? *
0/2

"You should ask your doctor since you promised him that you would not drink anymore."

"You have been drinking instead of eating, and alcohol interferes with folate metabolism in your liver."
"You need folic acid to make up for the vitamin B12 deficiency that was caused by your alcoholism."
"You need folic acid because you have not been compliant with taking your vitamins and attending Alcoholics Anonymous (AA)
meetings."

Correct answer
"You have been drinking instead of eating, and alcohol interferes with folate metabolism in your liver."

A patient admitted for cerebral swelling complained of feeling light-headed and nauseous while receiving mannitol. What should
the nurse do? *
2/2

Document, withdraw, and notify doctor.

Decrease mannitol flow.


Provide comfort measures.
Increase mannitol flow.

Nifedipine (Procardia) 30 mg p.o. is prescribed for a client. The nurse teaches the side effects and instructs the client to
immediately report: *
0/2

Blood pressure 110/70-114/78 for two successive readings.


Dizziness when changing positions.
Increased shortness of breath and orthopnea.

Weight loss of two pounds per week.

Correct answer
Weight loss of two pounds per week.

Biperiden hydrochloride (Akineton) is added to a list of antiparkinsonian medications that an elderly client is taking. Which of the
following instructions made by the nurse that needs further learning? *
0/2

To avoid smoking, alcohol and caffeine.


To use ice chips, candy or gum for dry mouth.
Walking in the morning to have a daily source of direct sunlight.
Eating foods rich in fiber and increase fluid intake.

Correct answer
Walking in the morning to have a daily source of direct sunlight.

A nurse is giving dietary instructions to a client receiving levodopa. Which of the following food items should be avoided by the
client? *
2/2

Goat yogurt.
Whole grain cereal.

Asparagus.
Apples

What is the therapeutic level for digoxin? *


2/2

0.5-2 ng/Ml

1.5-2 ng/mL
0.5-1.5 mg/mL
0.5-2 mg/Ml

Montelukast (Singulair) is prescribed to a client with asthma. During the medication therapy, which of the following laboratories
should be monitored? *
2/2

Complete blood count (CBC).


Sodium and Potassium.
Calcium and Platelet count.
ALT and AST.

Mr. Rafols a geriatric patient is prescribed with Cimetidine (Tagamet) for the treatment of heartburn. Which of the following is the
most frequent CNS side effect? *
2/2

Agitation.

Drowsiness.
Headache.
Somnolence

The client has an order for 2 tablespoons of Milk of Magnesia. The nurse converts this dose to the metric system and gives the
client how many ml? *
2/2

30 ml

20 ml
40 ml
15 ml

Patient Gong Yoo is being discharged home and will be required to self-administer injectable Heparin. You are observing the
patient administer their scheduled dose of Heparin to confirm that the patient knows how to do it correctly. What action by the
patient requires you to re-educate them about how to administer Heparin? *
0/2

The patient injects the needle into the fatty tissue of the abdomen.

The patient injects the needle 1 inch away from the umbilicus.
The patient rotated the injection site from the previous dose of Heparin.
The patient does not massage the injection site after administering the medication.

Correct answer
The patient injects the needle 1 inch away from the umbilicus.

Patient Corina is receiving Sulfasalazine (Azulfidine) for the treatment of ulcerative colitis. Which of the following assessment
finding will concern the nurse most? *
0/2

Drowsiness.
Decreased urine output.
Urine discoloration.
Vomiting.

Correct answer
Decreased urine output.

A client is receiving a continuous infusion of streptokinase (Streptase). The client suddenly complaints of a difficulty of breathing,
itchiness, and nausea. Which of the following should be the priority action of the nurse *
0/2

Stop the infusion and notify the physician.


Administer protamine sulfate and provide oxygen therapy.
Administer antihistamine then continue the infusion.

Slow the infusion and administer oxygen.

Correct answer
Stop the infusion and notify the physician.

A dopamine infusion was started in a patient's antecubital vein during resuscitation after cardiac arrest. The electronic infusion
device is now sounding an alert for an occlusion. What is the most important immediate concern for the nurse? *
2/2

Infiltration with phentolamine will be necessary if there is extravasation


An interruption in the infusion can produce hypotension in the patient.

The device will need to be reported to the hospital's clinical engineering department for service
The patient could develop hypertension as a result of the alarm.

A client with deep vein thrombosis is receiving Streptokinase (Streptase). The nurse would notify the physician if which of the
following assessment is noted? *
2/2

A temperature of 99.2° Fahrenheit.


A pulse rate of 99 beats per minute.
A respiratory rate of 21 breaths per minute.
A blood pressure of 185/110 mm Hg.

Which statement below BEST describes how Heparin works as an anticoagulant? *


2/2

“It inhibits clotting factors from synthesizing Vitamin K.”


“It inactivates the extrinsic pathways of coagulation.”
“It prevents Factor Xa from activating prothrombin to fibrinogen.”
“It enhances the activation of antihrombin III, which prevents the activation of thrombin and the conversion of fibrinogen to
fibrin.”

What is the priority nursing intervention for a client receiving an antiemetic? *


2/2

Monitor intake and output.


Keep items far away from the bed.
Give the client privacy by letting him walking around the room.
Keep bed in low position with side rails up.

Which antidiabetic agent works best in patients with insulin resistance? *


0/2

Biguanides

Dipeptidyl peptidase-4 inhibitors


Thiazolidinediones
Meglitinides

Correct answer
Thiazolidinediones

Karen, a client with myasthenia gravis, begins to experience increased difficulty in swallowing. To prevent aspiration of food, the
nursing action that would be most effective would be to: *
0/2

Change her diet order from soft foods to clear liquids

Place an emergency tracheostomy set in her room


Assess her respiratory status before and after meals
Coordinate her meal schedule with the peak effect of her medication, Mestinon

Correct answer
Coordinate her meal schedule with the peak effect of her medication, Mestinon

What is the antidote for digoxin intoxication? *


2/2

Diphenhydramine
Atropine sulfate
Digoxin immune fab (Digibind, DigiFab).

Phosphodiesterase inhibitors

The health care provider prescribes donepezil (Aricept) for a patient. The patient states to the nurse, "I have no idea why I take
this medication." The nurse knows the administration of donepezil (Aricept) is MOST commonly associated with what condition? *
2/2
Bladder retention
Alzheimer's disease

Parkinson's disease
Urinary retention

Which of the following is an indication for withholding indirect-acting cholinergic agonists? *


2/2

Visual acuity of 20/150


Blood pressure of 140/90 mmHg
GFR of 30 mL/hour
Adult heart rate of 55 beats per minute

The nurse, caring for a client receiving a cholinergic drug, should observe for increased symptoms of which of the following
processes? *
0/2

Hypertension
Heart failure
Cardiac dysrhythmias

Asthma

Correct answer
Heart failure

Nurse Renee is giving instructions to her client who is taking antihistamine. Which of the following nurse teachings is appropriate
for the client? *
2/2

Expect a relief in 24 hours


Be aware that you may have increased saliva
Be aware that you may need to take a decongestant
Avoid ingesting alcohol

The nurse is teaching a client about myasthenia gravis. Which statement, if made by the patient indicates the need for further
teaching? *
2/2

The doctor will take me off of my beta blocker because it could exacerbate my symptoms
I should report any signs of infection to my doctor
I can take an OTC ibuprofen to help with pain that may occur with spasms

I should avoid taking long walks

A client is receiving intravenous heparin therapy. The nurse ensures the availability in which of the following medication? *
2/2

Acetylcysteine (Mucomyst)
Calcium gluconate.
Vitamin K (Mephyton).
Protamine sulfate.

Which organ(s) produce(s) clotting factors? *


2/2

Bone marrow
Liver

Stomach
Lymphoid tissues

The three primary elements that determine the pressure in the cardiovascular system are heart rate, stroke volume, and: *
2/2

Baroreceptors
Peripheral resistance

Renin-Angiotensin-Aldosterone System (RAAS)


All of the above

Adrenocortical agents are primary used for which of the following?


2/2

curative efforts for inflammatory disorders


short term inflammation relief

long term circulatory issues


increasing urinary output & excess K+

Which of the following suggestions would the nurse make to help a patient who is receiving an anticholinergic agent reduce the
risks associated with decreased sweating? *
2/2

Covering the head and using sunscreen


Ensuring hydration and temperature control

Changing position slowly and protecting from the sun


Monitoring for difficulty swallowing and breathing

Anticholinergic drugs like atropine are given preoperatively to: *


2/2

Minimize the amount of analgesia needed postoperatively.


Provide a more rapid induction of anesthesia
Decrease the risk of bradycardia during surgery.

Induce relaxation prior to induction of anesthesia

Which of the following is the most common type of thyroid dysfunction *


2/2

Cretinism
Myxedema coma
Hyperthyroidism
Hypothyroidism

The initial nursing goal for a client with myasthenia gravis during the diagnostic phase of her hospitalization would be to: *
2/2

Develop a teaching plan


Facilitate psychologic adjustment
Maintain the present muscle strength

Prepare for the appearance of myasthenic crisis

In making a diagnosis of myasthenia gravis Edrophonium HCI (Tensilon) is used. The nurse knows that this drug will cause a
temporary increase in which of the following? *
0/2

Muscle strength
Symptoms
Blood pressure

Consciousness

Correct answer
Muscle strength

Which diuretic is the drug of choice for the prevention of tissue damage after a closed head injury? *
2/2

Mannitol

Metolazone
Spironolactone
Hydrochlorothiazide

Patient William is given epinephrine (Adrenalin), an adrenergic agonist (sympathomimetic). The nurse should monitor the patient
for which condition? *
2/2

Decreased pulse
Pupil constriction
Bronchial constriction
Increased blood pressure

What is the goal of therapy for Parkinson’s disease? *


2/2

To decrease dopamine and to increase cholinergic neurons


To balance dopamine and cholinergic neurons

To excite neurons more


To inhibit neurons more

A patient has a prescription for oxybutynin (Ditropan), an anticholinergic drug. When reviewing the patient's medical history,
which condition, if present, would be considered a contraindication to therapy with this drug? *
2/2

Diarrhea
Hypertension
Neurogenic bladder
Uncontrolled angle-closure glaucoma

What finding would the nurse anticipate will be absent as the result of a lack of testosterone production? *
2/2

male secondary sex characteristics

stomach acid production


basal metabolism
bile secretion

Name *

Noval, Lyn-Je Kirstene M.

A client arrives in the emergency unit and reports that a concentrated household cleaner was splashed in both eyes. Which of the
following nursing actions is a priority? *
2/2

Use Restasis (Allergan) drops in the eye.


Flush the eye repeatedly using sterile normal saline.

Examine the client’s visual acuity.


Patch the eye.

Patient Lee Jong receives epoetin alfa (Epogen) subcutaneously, and says to the nurse, "My doctor said I have anemia. Are there
little red blood cells in that shot?" What are the best responses by the nurse? *
0/2
"No, we do not give blood for anemia anymore."
"No, this medication stimulates your body to make red blood cells."
" Yes, this small amount of red blood cells will stimulate your bone marrow to produce more cells in the kidney."
"No, this medication promotes clotting so you will not lose even more red blood cells."

Correct answer
"No, this medication stimulates your body to make red blood cells."

All of the following should be included in the nursing interventions for patients taking thyroid hormones, except: *
2/2

consistent administration before breakfast each day


monitor cardiac response
advise patient to frequently exercise to develop the core

provide for temperature control and safety precautions

What is the drug of choice in children with parkinsonian symptom? *


0/2

Levodopa

Artane
Benadryl
Benztropine

Correct answer
Benadryl

The nurse administering donepezil (Aricept) to a patient with Alzheimer's disease understands this drug has which expected
therapeutic action? *
2/2

Helps control associated urinary and fecal incontinence


Blocks the effect of norepinephrine at the presynaptic cell membrane
Increases levels of acetylcholine in the brain by blocking its breakdown

Relief of anxiety and restless behavior of the patient

Cholinergic (parasympathomimetic) drugs have which therapeutic effect? *


0/2

Increased gastrointestinal (GI) motility


Blood vessel vasoconstriction
Urinary retention
Pupil dilation

Correct answer
Increased gastrointestinal (GI) motility
A nurse is monitoring a client who is taking Carvedilol (Coreg CR). Which of the following assessment made by the nurse would
warrant a possible complication with the use of this medication? *
2/2

Baseline blood pressure of 160/100 mm hg followed by a blood pressure of 120/70 mm hg after 3 doses.
Baseline heart rate of 97 bpm followed by a heart rate of 62 bpm after 3 doses.
Complaints of nightmares and insomnia.
Complaints of dyspnea.

Why is the hypothalamus considered the master gland in the human body? *
2/2

it monitors homeostasis and coordinates response through the nervous and endocrine system.

it secretes releasing hormones


it stimulates the pituitary gland
it has various neurocenter

Nurse Hyun makes a home visit to the client with diabetes mellitus. During the visit, Nurse Hyun notes the client’s additional
insulin vials are not refrigerated. What is the best action by the nurse at this time? *
0/2

Instruct the client to label each vial with the date when opened.
Tell the client there is no need to keep additional vials.

Have the client place the insulin vials in the refrigerator.


Have the client discard the vials.

Correct answer
Have the client discard the vials.

A patient presents to the emergency department with severe respiratory distress, hives, and edema after being stung on the face
by a bee. Which are of the following is not an accurate nursing assessment of the situation? *
2/2

Hypotension and bronchospasm will progress rapidly if treatment is delayed.


The patient is suffering from anaphylactic shock.
Epinephrine and diphenhydramine are the drugs of choice as first-line agents.
Prompt treatment with drug therapy will prevent this syndrome from occurring again.

The nurse is about to administer a stat dose of intravenous atropine sulfate to a patient who is experiencing a symptomatic
cardiac dysrhythmia. During administration of this drug, the nurse will monitor the patient closely for which adverse effect? *
2/2

Tachycardia

Bradycardia
Ectopic beats
Cardiac standstill

What does thyroid hormones do? *


2/2

stimulate overproduction of TSH


increase oxygen consumption

increase the heart rate without affecting the respirations


retard growth and maturation

Mrs. Quwzon, a 40-year-old patient, came in due to acute hypertensive episode. Her chart says she is asthmatic. Which of the
following drugs would be safest to give? *
2/2

Nebivolol
Timolol
Propranolol
Atenolol

Renita will be having her exam in pharmacology tomorrow. She should be aware that antitussive is indicated to: *
2/2

Encourage removal of secretions through coughing


Relieve rhinitis
Relieve a dry cough

Control a productive cough

What vitamin plays a role in lowering cholesterol concentration? *


2/2

Vitamin C
Vitamin E
Vitamin B3

Vitamin B2

All of the following should be included in the nursing interventions for patients taking thyroid hormones, EXCEPT: *
2/2

consistent administration before breakfast each day


monitor cardiac response
advise patient to frequently exercise to develop the core

provide for temperature control and safety precautions

Your patient with asthma is taking Theophylline. Which product below should the patient avoid consuming? *
2/2

Caffeine

Dairy
Wheat
Shellfish

Which instruction should be included in the discharge teaching for a patient with a transdermal nitroglycerin (Nitro-Dur) patch? *
2/2

"If you get chest pain, apply a second patch next to the first patch."
"If you get a headache, remove the patch for 4 hours and then reapply."
"Make sure to rub a lotion or cream on the skin before putting on a new patch."
"Apply the patch to a hairless, nonirritated area of the chest, upper arm, back or shoulder."

Which medication below blocks the function of Leukotriene for the treatment of asthma? *
2/2

Salmeterol
Theophylline
Tiotropim
Montelukast
A patient is admitted to the emergency department with an overdose of a barbiturate. The nurse
immediately prepares to administer which of the following from the emergency drug cart? *
0/2
Naloxone HCl (Narcan
Incorrect

Activated charcoal
Flumazenil (Romazicon)
Ipecac syrup
Correct answer
Activated charcoal

The nurse explains to a patient that using caffeine may exacerbate which health condition?
2/2
Cardiac dysrhythmias
Correct

Constipation
Heart block
Myelin degeneration

A patient arrives at the emergency department complaining of back pain. He reports taking at least
3 acetaminophen tablets every three hours for the past week without relief. Which of the following
symptoms suggests acetaminophen toxicity? *
2/2
Tinnitus.
Diarrhea.
Hypertension.
Hepatic damage.
Correct
The nurse is reviewing the history of a patient who will be starting the triptan sumatriptan (Imitrex)
as part of a treatment for migraine headaches. Which condition, if present, may be a
contraindication to triptan therapy? *
2/2
Cardiovascular disease
Correct

Chronic bronchitis
History of renal calculi
Diabetes mellitus type 2

The patient’s chart notes the administration of dantrolene (Dantrium) immediately postoperatively.
The nurse suspects that the patient experienced: *
2/2
Delirium tremens
Malignant hyperthermia
Correct

A tonic-clonic seizure
Respiratory arrest

Which of the following is an appropriate nursing intervention for patients who are receiving CNS
depressants? *
2/2
Make sure that the patient knows that sedation should be minimal with these agents.
Prevent any activity within the hospital setting while on oral muscle relaxants
Make sure the patient’s call light is close by in case of the need for assistance with activities.
Correct

Cardiovascular stimulation, a common side effect, would lead to hypertension

Abrupt withdrawal of barbiturates can precipitate what medical condition?


2/2
Ascites
Hypertensive crisis
Status epilepticus
Correct

Coma

Which client is most likely to receive opioids for extended periods of time? *
2/2
A client with fibromyalgia
A client with phantom limb pain
A client with progressive pancreatic cancer
Correct

A client with trigeminal neuralgia

In caring for a young child with pain, which assessment tool is the most useful? *
2/2
Simple description pain intensity scale
0-10 numeric pain scale
Faces pain-rating scale
Correct

McGill-Melzack pain questionnaire

In applying the principles of pain treatment, what is the first consideration? *


2/2
Treatment is based on client goals.
A multidisciplinary approach is needed.
The client must be believed about perceptions of own pain.
Correct

Drug side effects must be prevented and managed


A client is being tapered off opioids and the nurse is watchful for signs of withdrawal. What is one
of the first signs of withdrawal? *
2/2
Fever
Nausea
Diaphoresis
Correct

Abdominal cramp

What is the antidote for narcotic overdose.


2/2
Naloxone
Correct

Nubain
Morphine
Codeine

Which of the following medical condition(s) can be considered as contraindication(s) to use of


anxiolytics and hypnotics? *
2/2
Psychosis
Alcoholic intoxication
Acute gastroenteritis
Both A and B
Correct

Required
A patient with narcolepsy will begin treatment w/a CNS stimulant. The nurse expects to see which
adverse effect? *
0/2
bradycardia
Incorrect

nervousness
mental clouding
drowsiness at nigh
Correct answer
nervousness

A patient diagnosed with narcolepsy is prescribed a central nervous system (CNS) stimulant. Which
statement best describes the action of CNS stimulants? *
2/2
CNS stimulants activate cyclic adenosine monophosphate.
CNS stimulants block or reduce the activity of inhibitory neurons.
CNS stimulants increase release of and block reuptake of neurotransmitters.
Correct

The nurse will monitor the patient taking albuterol (Proventil) for which conditions? *
2/2
Palpitations
Correct

Hypoglycemia
Bronchospasm
Uterine contractions

Patient William is given epinephrine (Adrenalin), an adrenergic agonist (sympathomimetic). The


nurse should monitor the patient for which condition? *
2/2
Decreased pulse
Pupil constriction
Bronchial constriction
Increased blood pressure
Correct
A nurse is giving dietary instructions to a client receiving levodopa. Which of the following food
items should be avoided by the client? *
2/2
Goat yogurt.
Whole grain cereal.
Correct

Asparagus.
Apples

Biperiden hydrochloride (Akineton) is added to a list of antiparkinsonian medications that an


elderly client is taking. Which of the following instructions made by the nurse that needs further
learning? *
0/2
To avoid smoking, alcohol and caffeine.
Incorrect

To use ice chips, candy or gum for dry mouth.


Walking in the morning to have a daily source of direct sunlight.
Eating foods rich in fiber and increase fluid intake.
Correct answer
Walking in the morning to have a daily source of direct sunlight.

What is the goal of therapy for Parkinson’s disease? *


0/2
To decrease dopamine and to increase cholinergic neurons
Incorrect

To balance dopamine and cholinergic neurons


To excite neurons more
To inhibit neurons more
Correct answer
To balance dopamine and cholinergic neurons

What is the drug of choice in children with parkinsonian symptom? *


0/2
Levodopa
Incorrect

Artane
Benadryl
Benztropine
Correct answer
Benadryl

The nurse teaches the patient receiving atropine (Atreza) to expect which side effect?
2/2
Diarrhea
Bradycardia
Blurred vision
Correct

Frequent urination

Mrs. Quwzon, a 40-year-old patient, came in due to acute hypertensive episode. Her chart says she
is asthmatic. Which of the following drugs would be safest to give? *
2/2
Nebivolol
Timolol
Propranolol
Atenolol
Correct
Your patient with asthma is taking Theophylline. Which product below should the patient avoid
consuming? *
2/2
Caffeine
Correct

Dairy
Wheat
Shellfish

Which medication below blocks the function of Leukotriene for the treatment of asthma? *
2/2
Salmeterol
Theophylline
Tiotropim
Montelukast
Correct

Karen, a client with myasthenia gravis, begins to experience increased difficulty in swallowing. To
prevent aspiration of food, the nursing action that would be most effective would be to: *
0/2
Change her diet order from soft foods to clear liquids
Incorrect

Place an emergency tracheostomy set in her room


Assess her respiratory status before and after meals
Coordinate her meal schedule with the peak effect of her medication, Mestinon
Correct answer
Coordinate her meal schedule with the peak effect of her medication, Mestinon

Your patient with asthma is taking Theophylline. Which product below should the patient avoid
consuming? *
2/2
Caffeine
Correct
Dairy
Wheat
Shellfish

A nurse is giving dietary instructions to a client receiving levodopa. Which of the following food
items should be avoided by the client? *
2/2
Goat yogurt.
Whole grain cereal.
Correct

Asparagus.
Apples

The nurse is providing education to a patient on the primary uses of cholinergic drugs. Which do
you think the nurse should NOT include in the teaching? *
0/2
Decrease intraocular pressure
Dilate pulmonary airways
Stimulate bladder emptying
Stimulate peristalsis
Incorrect

Correct answer
Dilate pulmonary airways

Which of the following is an indication for withholding indirect-acting cholinergic agonists? *


2/2
Visual acuity of 20/150
Blood pressure of 140/90 mmHg
GFR of 30 mL/hour
Adult heart rate of 55 beats per minute
Correct
The nurse administering donepezil (Aricept) to a patient with Alzheimer's disease understands this
drug has which expected therapeutic action? *
2/2
Helps control associated urinary and fecal incontinence
Blocks the effect of norepinephrine at the presynaptic cell membrane
Increases levels of acetylcholine in the brain by blocking its breakdown
Correct

Relief of anxiety and restless behavior of the patient

A patient has a prescription for oxybutynin (Ditropan), an anticholinergic drug. When reviewing the
patient's medical history, which condition, if present, would be considered a contraindication to
therapy with this drug? *
2/2
Diarrhea
Hypertension
Neurogenic bladder
Uncontrolled angle-closure glaucoma
Correct

Mrs. Quwzon, a 40-year-old patient, came in due to acute hypertensive episode. Her chart says she
is asthmatic. Which of the following drugs would be safest to give? *
2/2
Nebivolol
Timolol
Propranolol
Atenolol
Correct
The initial nursing goal for a client with myasthenia gravis during the diagnostic phase of her
hospitalization would be to: *
2/2
Develop a teaching plan
Facilitate psychologic adjustment
Maintain the present muscle strength
Correct

Prepare for the appearance of myasthenic crisis

Which of the following suggestions would the nurse make to help a patient who is receiving an
anticholinergic agent reduce the risks associated with decreased sweating? *
2/2
Covering the head and using sunscreen
Ensuring hydration and temperature control
Correct

Changing position slowly and protecting from the sun


Monitoring for difficulty swallowing and breathing

Which of the following is the most common type of thyroid dysfunction *


2/2
Cretinism
Myxedema coma
Hyperthyroidism
Hypothyroidism
Correct

Patient William is given epinephrine (Adrenalin), an adrenergic agonist (sympathomimetic). The


nurse should monitor the patient for which condition? *
2/2
Decreased pulse
Pupil constriction
Bronchial constriction
Increased blood pressure
Correct

The health care provider prescribes donepezil (Aricept) for a patient. The patient states to the nurse,
"I have no idea why I take this medication." The nurse knows the administration of donepezil
(Aricept) is MOST commonly associated with what condition? *
2/2
Bladder retention
Alzheimer's disease
Correct

Parkinson's disease
Urinary retention

Which medication below blocks the function of Leukotriene for the treatment of asthma? *
2/2
Salmeterol
Theophylline
Tiotropim
Montelukast
Correct

What is the goal of therapy for Parkinson’s disease? *


2/2
To decrease dopamine and to increase cholinergic neurons
To balance dopamine and cholinergic neurons
Correct
To excite neurons more
To inhibit neurons more

A client diagnosed with Alzheimer's disease is prescribed donzepil (Aricept) for treatment. The
client has a medical history of osteoarthritis and GERD. For what possible complications of this
drug therapy should the nurse assess in the client?
2/2
Tachycardia
Bradycardia
Aspiration
Correct

Muscle pain

All of the following should be included in the nursing interventions for patients taking thyroid
hormones, EXCEPT: *
0/2
consistent administration before breakfast each day
monitor cardiac response
Incorrect

advise patient to frequently exercise to develop the core


provide for temperature control and safety precautions
Correct answer
advise patient to frequently exercise to develop the core

Biperiden hydrochloride (Akineton) is added to a list of antiparkinsonian medications that an


elderly client is taking. Which of the following instructions made by the nurse that needs further
learning? *
2/2
To avoid smoking, alcohol and caffeine.
To use ice chips, candy or gum for dry mouth.
Walking in the morning to have a daily source of direct sunlight.
Correct
Eating foods rich in fiber and increase fluid intake.

What does thyroid hormones do? *


0/2
stimulate overproduction of TSH
increase oxygen consumption
increase the heart rate without affecting the respirations
Incorrect

retard growth and maturation


Correct answer
increase oxygen consumption

The nurse is caring for a client admitted with suspected myasthenia gravis. Which finding is usually
associated with a diagnosis of myasthenia gravis? *
2/2
Visual disturbances, including diplopia
Ascending paralysis and loss of motor function
Cogwheel rigidity and loss of coordination
Progressive weakness that is worse at the day s end
Correct

In making a diagnosis of myasthenia gravis Edrophonium HCI (Tensilon) is used. The nurse knows
that this drug will cause a temporary increase in which of the following? *
2/2
Muscle strength
Correct

Symptoms
Blood pressure
Consciousness

The nurse is teaching the female client with myasthenia gravis about the prevention of myasthenic
and cholinergic crises. The nurse tells the client that this is most effectively done by: *
0/2
.Taking medications on time to maintain therapeutic blood levels
Doing all chores early in the day while less fatigued
Doing muscle-strengthening exercises
Eating large, well-balanced meals
Incorrect

Correct answer
.Taking medications on time to maintain therapeutic blood levels
Name *
Noval, Lyn-Je Kirstene M.

The nurse teaches the patient receiving atropine (Atreza) to expect which side effect?
0/2
Diarrhea
Incorrect

Bradycardia
Blurred vision
Frequent urination
Correct answer
Blurred vision

The nurse is about to administer a stat dose of intravenous atropine sulfate to a patient who is
experiencing a symptomatic cardiac dysrhythmia. During administration of this drug, the nurse will
monitor the patient closely for which adverse effect? *
2/2
Tachycardia
Correct
Bradycardia
Ectopic beats
Cardiac standstill

Cholinergic (parasympathomimetic) drugs have which therapeutic effect? *


0/2
Increased gastrointestinal (GI) motility
Blood vessel vasoconstriction
Incorrect

Urinary retention
Pupil dilation
Correct answer
Increased gastrointestinal (GI) motility

What is the drug of choice in children with parkinsonian symptom? *


2/2
Levodopa
Artane
Benadryl
Correct

Benztropine

The nurse is teaching a client about myasthenia gravis. Which statement, if made by the patient
indicates the need for further teaching? *
2/2
The doctor will take me off of my beta blocker because it could exacerbate my symptoms
I should report any signs of infection to my doctor
I can take an OTC ibuprofen to help with pain that may occur with spasms
Correct

I should avoid taking long walks


The nurse will monitor the patient taking albuterol (Proventil) for which conditions? *
2/2
Palpitations
Correct

Hypoglycemia
Bronchospasm
Uterine contractions

Which of the following is not an autoimmune disease? *


0/2
Insulin-dependent diabetes mellitus
Myasthenia gravis
Incorrect

Alzheimer's disease
Graves disease
Correct answer
Alzheimer's disease

Anticholinergic drugs like atropine are given preoperatively to: *


2/2
Minimize the amount of analgesia needed postoperatively.
Provide a more rapid induction of anesthesia
Decrease the risk of bradycardia during surgery.
Correct
The nurse creates a plan of care for a child at risk for tonic-clonic seizures. In the plan of care, the
nurse identifies seizure precautions and documents that which item(s) need to be placed at the
child's bedside? *
2/2
Emergency cart
Tracheotomy set
Padded tongue blade
Suctioning equipment and oxygen
Correct

A 15-year-old client is taking Phenytoin (Dilantin) for the treatment of seizures. Phenytoin blood
level reveals to be 25 mcg/ml. Which of the following symptoms would be expected as a result of
the laboratory result?
2/2
No symptoms, because the value is within the normal range.
Hypoglycemia.
Tachycardia.
Nystagmus.
Correct

Patient Anna is receiving IV antiepileptic therapy. Which parenteral antiepileptic drug is effective
against generalized seizures? *
2/2
Phenytoin (Dilantin)
Fosphenytoin (Cerebyx)
Phenobarbital (Luminal)
Valproic acid (Depakene)
Correct

What information should the nurse provide to a patient who will self-administer an antiepileptic
agent for the first time at home? *
2/2
Report any vision or hearing problems with levetiracetam (Keppra).
Stir phenobarbital (Luminal) elixir into a cold carbonated beverage.
Have a complete blood count monthly while taking carbamazepine (Tegretol).
Correct

Administer pregabalin (Lyrica) every morning on an empty stomach.

The nurse is caring for a client who begins to experience seizure activity while in bed. Which
actions should the nurse not take? *
2/2
Loosening restrictive clothing
Restraining the client's limbs
Correct

Removing the pillow and raising padded side rails


Positioning the client to the side, if possible, with the head flexed forward

A client with myasthenia gravis has become frequently complaining of weakness and fatigue. The
physician plans to identify whether the client is responding to an overdose of the medication or a
worsening of the disease. A tensilon test is performed. Which of the following would indicate that
the client is experiencing an overdose of the medication? *
2/2
Temporarily worsening of the condition.
Correct

Improvement of the weakness and fatigue.


No change in the condition.
Complaints of muscle spasms

Tyrone with a tentative diagnosis of myasthenia gravis is admitted for diagnostic make up.
Myasthenia gravis can confirmed by: *
2/2
Kernigs sign
Brudzinski’s sign
A positive sweat chloride test
A positive edrophonium (Tensilon) test
Correct

Option 1

Warren is diagnosed with myasthenia gravis and pyridostigmine bromide (Mestinon) therapy is
started. The Mestinon dosage is frequently changed during the first week. While the dosage is being
adjusted, the nurse’s priority intervention is to:
2/2
Administer the medication exactly on time
Administer the medication with food or mild
Evaluate the client’s muscle strength hourly after medication
Correct

Evaluate the client’s emotional side effects between doses

Which of the following is the priority nursing intervention for children receiving muscle
relaxants? *
2/2
Assess bowel sounds and activity.
Correct

Provide skin care.


Administer drug with food.
Both A and B.

Mayer a client with myasthenia gravis is to receive immunosuppressive therapy. The nurse
understands that this therapy is effective because it: *
2/2
Promotes the removal of antibodies that impair the transmission of impulses
Stimulates the production of acetylcholine at the neuromuscular junction.
Decreases the production of autoantibodies that attack the acetylcholine receptors.
Correct
TEST BANK
Drug Calculation

The doctor ordered Magnesium Sulfate 4gm IV to be given over 20


minutes. Magnesium Sulfate come in 40gm/1000mL IV bottle. You would
set your pump at _____mL/hour.
250 mL/hr
300 mL/hr
350 mL/hr
380 mL/hr

The doctor ordered Magnesium Sulfate 2gm/ hour IV. Magnesium Sulfate
come in 40gm/1000mL IV bottle. You would set your pump at
_____mL/hour.
50 mL/hr
60 mL/hr
55 mL/hr
53 mL/hr

The physician ordered Ritodrine IV 70mcg/min. The pharmacy sent up


Ritodrine 150mg premixed in 500mL D5W. You would set your IV pump at
________mL/hour.
15 mL/hr
16 mL/hr
13 mL/hr
14 mL/hr

The physician ordered Ritodrine IV 50 mcg/min. The pharmacy sent up


Ritodrine 150 mg premixed in 500mL D5W. You would set your IV pump
at ________mL/hour.
9 mL/hr
10 mL/hr
11 mL/hr
12 mL/hr
The physician ordered the Pitocin infusion to run at 6 mu/min. The
pharmacy sent up 10 units of Pitocin in 500 mL of D5LR. You would set
your pump at _____________ mL/hour.
18 mL/hr
19 mL/hr
20 mL/hr
21 mL/hr

The physician ordered the Pitocin infusion to run at 16 mu/min. The


pharmacy sent up 10 units of Pitocin in 500 mL of D5LR. You would set
your pump at _____________ mL/hour.
45 mL/hr
46 mL/hr
47 mL/hr
48 mL/hr

The medications scheduled for your patient include Keflex 1.5 grams in
50 ml of a 5% Dextrose solution. According to the pharmacy, this
preparation should be administered in 30 minutes. You would set your IV
pump at __________mL/hour.
100 mL/hr
110 mL/hr
115 mL/hr
120 mL/hr

The physician reduces an IV to 30 ml/hour. The IVAC indicates that 270


ml are remaining in the present IV bag. You notice that it is exactly 10:30
am. At what time will the infusion be completed?
7:20 P.M.
7:25 P.M.
7:30 P.M.
7:35 P.M
The physician orders 1.5 liters of Lactated Ringer's solution to be
administered intravenously to your patient over the next 12 hours.
Calculate the rate of flow if the IV tubing delivers 20 gtt/ml.
40 gtts
41 gtts
42 gtts
43 gtts

The 10 am medications scheduled for your patient include Keflex 2.0 g in


100 ml of a 5% Dextrose solution. According to the pharmacy, this
preparation should be administered in thirty minutes. You should set your
IV pump at _____mL/hour.
200 mL/hr
205 mL/hr
206 mL/hr
250 mL/hr

1.) MD writes an order for Tylenol 160 mg by mouth as needed every 6 hours for pain for a
child. Pharmacy dispenses you with 80 mg per ml. How many ml will you administer per dose?

 A. 2 ml/dose
 B. 6 ml/dose
 C. 0.5 ml/dose
 D. 25 ml/dose
The answer is A.

2.) MD writes an order for Mucomyst 300 mg by mouth one dose before heart cath. Pharmacy
dispenses you with 600 mg/2 ml. How many ml will you administer per dose?

 A. 150 ml/dose
 B. 2 ml/dose
 C. 1 ml/dose
 D. 0.25 ml/dose
The answer is C.

3.) MD writes an order for Ibuprofen 3 mg/kg by mouth every 6 hours for pain for a child. The
child weighs 73 lb. Pharmacy dispenses you with 50 mg/2 ml. How many ml will you
administer per dose?
 A. 1 ml/dose
 B. 3.9 ml/dose
 C. 2.9 ml/dose
 D. 6 ml/dose
The answer is B.

4.) MD writes an order for Lactulose 20 grams by mouth BID. Pharmacy dispenses you with 10
gram/15 ml. How many ml with you administer per dose?

 A. 30 ml/dose
 B. 15 ml/dose
 C. 20 ml/dose
 D. 7.5 ml/dose
The answer is A.

5.) MD writes an order for Lortab Elixir 5mg by mouth as needed for pain every 4 hours.
Pharmacy dispenses you with 7.5mg/15ml. How many tablespoons will you administer per
dose?

 A. 1 Tbsp/dose
 B. 8 Tbsp/dose
 C. 0.6 Tbsp/dose
 D. 0.25 Tbsp/dose
The answer is C.

6.) MD writes an order for Robitussin AC 100 mg by mouth 3 times per day as needed for
cough. Pharmacy dispenses you with 100mg/5ml. How many teaspoons will you administer
per dose??

 A. 0.5 tsp/dose
 B. 1 tsp/dose
 C. 2.5 tsp/dose
 D. 3 tsp/dose
The answer is B.
7.) MD writes an order for Viscous Lidocaine 15 mg by mouth every 4 hours as needed for
sore throat. Pharmacy dispenses you 30 mg/3ml. How many ml with you administer per dose?

 A. 0.5 ml/dose
 B. 3 ml/dose
 C. 1.5 ml/dose
 D. 6 ml/dose
The answer is C.

8.) MD writes an order for Nystatin Oral Suspension 250,000 units by mouth every 6 hours for
thrust in mouth. Pharmacy dispense you with 500,000 units/5ml. How many ml will you
administer per dose?

 A. 0.25 ml/dose
 B. 10 ml/dose
 C. 5 ml/dose
 D. 2.5 ml/dose
The answer is D.

9.) MD writes an order for GI cocktail 150 mg by mouth every 6 hours for 24 hours. Pharmacy
dispenses you with 75mg/3ml. How many ml will you administer daily?

 A. 15 ml/day
 B. 9 ml/day
 C. 24 ml/day
 D. 48 ml/day
The answer is C.

10. MD writes an order for Posaconazole 75 mg by mouth once a day. Pharmacy dispenses
you with 20 mg/ 2 ml. How many ml will you administer per dose?

 A. 7.5 ml/dose
 B. 3 ml/dose
 C. 2.5 ml/dose
 D. 12 ml/dose
The answer is A.
11.) MD writes an order for Noxafil oral suspension 100 mg by mouth daily. Pharmacy
dispenses you with 40mg/ml. How many ml will you administer per dose?

 A. 0.5 ml/dose
 B. 3 ml/dose
 C. 2.5 ml/dose
 D. 40 ml/dose
The answer is C.

12.) MD writes an order for Cipro oral suspension 250 mg every 12 hours for 3 day for a UTI.
Pharmacy dispenses you with 5 gram/100 ml. How many tsp will you administer per dose?

 A. 25 tsp/dose
 B. 1 tsp/dose
 C. 6 tsp/dose
 D. 2.5 tsp/dose
The answer is B.

13.) MD writes an order for Digoxin Elixir 250 mcg by mouth daily. Patient’s apical pulse is 65
bpm. Pharmacy dispenses you with 0.5 mg/2 ml. How many ml will you administer per dose?

 A. 30 ml/dose
 B. 0.5 ml/dose
 C. 2 ml/dose
 D. 1 ml/dose
The answer is D.

14.) MD writes an order for Theodur Elixir 200 mg by mouth every 8 hours. Pharmacy
dispenses you with 125mg/15ml. How many ml will you administer per dose?

 A. 24 ml/dose
 B. 7.5 ml/dose
 C. 30 ml/dose
 D. 100 ml/dose
The answer is A.
15.) MD writes an order for Carafate 2 gram by mouth tid. Pharmacy dispenses you with 500
mg/ 2.5 ml. How many ml will you administer daily?

 A. 5 ml/day
 B. 30 ml/day
 C. 15 ml/day
 D. 250 ml/day
The answer is B.

16.) MD writes an order for Demerol oral suspension 50 mg by mouth every 6 hours for pain.
Pharmacy dispenses you with 25 mg/ 2 ml. How many teaspoons with you give per dose?

 A. 0.5 tsp/dose
 B. 2 tsp/dose
 C. 0.8 tsp/dose
 D. 250 ml/dose
The answer is C.

17.) MD writes an order for Ibuprofen 300 mg by mouth every 4 hours for pain. Pharmacy
dispenses you with 50 mg/ 2.5 ml. How many ml will you administer per dose?

 A. 17 ml/dose
 B. 5 ml/dose
 C. 15 ml/dose
 D. 150 ml/dose
The answer is C.

18.) MD writes an order for Amoxicillin 300 mg by mouth daily. Pharmacy dispenses you with
200 mg/ 2 ml. How many ml will you administer per dose?

 A. 100 ml/dose
 B. 2 ml/dose
 C. 3 ml/dose
 D. 10 ml/dose
The answer is C.
19.) MD writes an order for Valium 1 mg by mouth via PEG tube. Pharmacy dispenses you with
3 mg/ml. How many ml will you administer per dose?

 A. 0.3 ml/dose
 B. 0.6 ml/dose
 C. 9 ml/dose
 D. 2 ml/dose
The answer is A.

20.) MD writes an order for Colace 50 mg daily via PEG tube. Pharmacy dispenses you with
100 mg/ 15 ml. How many ml will you administer per dose?

 A. 50 ml/dose
 B. 7.5ml/dose
 C. 15 ml/dose
 D. 2.5 ml/dose
The answer is B.

1. A patient is prescribed Pravastatin. This medication is known as a


______________________?

 A. Fibric Acid Derivative


 B. Bile-acid Sequestrant
 C. HMG-CoA Reductase Inhibitor
 D. Cholesterol Absorption Inhibitor
The answer is C: HMG-CoA Reductase Inhibitor. Pravastatin is a STATIN medication which is part of
the HMG-CoA Reductase Inhibitor family.

2. Your patient is prescribed a statin medication for the treatment of hyperlipidemia. What
medication below may increase the patient's risk of developing rhabdomyolysis if taken with a
statin?

 A. Ezetimibe
 B. Gemfibrozil
 C. Cholestyramine
 D. Colesevelam
The answer is B. The usage of fibric acid derivatives (fibrates....such as Gemfibrozil or Fenofibrate)
and statins together can increase the risk of rhabdomyolysis (break down of skeletal muscle which
leads to renal failure).

3. Which statement below accurately describes how statin medications work to lower
cholesterol levels?

 A. Statins increase the activity of LDL receptors in the liver by increasing the
bioavailability of fibric-acid.
 B. Statins inhibit bile acid in the GI tract from being absorbed and as a result the liver
turns cholesterol into bile acid.
 C. Statins increases hydroxymethylglutaryl–coenzyme A enzyme and this increases renal
excretion of cholesterol.
 D. Statins inhibit HMG-CoA reductase which in turn hinders cholesterol synthesis
in the liver.
The answer is D: Statins inhibit HMG-CoA reductase which in turn hinders cholesterol synthesis in the
liver.

4. Your patient is taking Simvastatin. Which of the following findings below demonstrate this
medication is providing therapeutic effects to this patient? Select-all-that-apply:

 A. HDL 80 mg/dL
 B. LDL 72 mg/dL
 C. Total Cholesterol 350
 D. HDL 30 mg/dL
 E. LDL 190 mg/dL
 F. Total Cholesterol 120
The answers are A, B, F. Statins should lower LDL (the bad cholesterol…..approximate normal range
<100), increase HDL (the good cholesterol…. approximate normal range >60), and lower total
cholesterol (normal range less than 200).

5. You're providing education to a group of participants in the community about cholesterol


and cardiac disease. Which statements below verbalized by some of the participants are
inaccurate and require you to provide clarification?

 A. "HDL levels should be less than 60 mg/dL to maintain good cardiac health."
 B. "LDL is a fatty substance that sticks to vessels walls and can lead to atherosclerosis."
 C. "HDL is a fatty substance that helps remove low-density lipoprotein from the blood so
it doesn’t have the chance to stick to vessel walls.”
 D. "LDL levels should be greater than 100 mg/dL to maintain good cardiac
health."
The answers are A and D. These statements are false. LDL levels should be LESS (not greater) than
100 mg/dL, and HDL levels should be HIGHER (not lower) than 60 mg/dL. The other statements are
true.

6. A patient has been taking Simvastatin and reports to you that they are experiencing
soreness and weakness of the muscles. The physician orders lab work. What lab result below
will you immediately report to the physician?

 A. Increased High-density Lipoprotein level


 B. Decreased Creatine Kinase level
 C. Decreased Low-density Lipoprotein level
 D. Increased Creatine Kinase level
The answer is D: Increased Creatine Kinase level. Statins have a slight risk of causing "statin-induced
rhabdomyolysis". This is the breaking down of skeletal muscle. If a patient reports muscle soreness or
weakness while taking a statin the CK levels (Creatine Kinase) should be measured. Creatine Kinase
is a protein found in skeletal muscle (it’s also found in the heart). If skeletal muscle is breaking down
CK will be found in the blood in high amounts and this tells us there is muscle damage. The CK level
will be 10 x's the upper limit of normal…if this occurs the statin should be discontinued.

7. Your patient is prescribed Lovastatin due to their risk factors for developing cardiac
disease. You're providing education about the importance of continuing exercise and a
healthy diet while taking this medication. You are helping the patient create a weekly food
plan. Which food item below should the patient avoid including in the food plan while taking
this medication?

 A. Bananas
 B. Spinach
 C. Grapefruit
 D. Aged-cheese
The answer is C. Grapefruit or grapefruit juice can increase the toxicity of statins.

8. Your patient who is taking a statin develops yellowing of the skin, abdominal pain in the
right upper quadrant, and nausea. What lab result below correlates with this finding?

 A. Elevated ALT and AST level


 B. Elevated BUN and creatinine
 C. Decreased WBC
 D. Decreased Hbg
The answer is A. Liver injury is a risk with statins. Therefore, a liver functions test (LFTs) should be
ordered to assess the liver's function. The patient is presenting with signs and symptoms of liver
injury. An elevated ALT and AST level would correlate with these presenting signs and symptoms.

Go Here for More Quizzes


(NOTE: When you hit submit, it will refresh this same page. Scroll down to see your results.)

Statins (HMG-CoA Reductase Inhibitors) Nursing Pharmacology Questions


1. A patient is prescribed Pravastatin. This medication is known as a ______________________?

A. Fibric Acid Derivative

B. Bile-acid Sequestrant

C. HMG-CoA Reductase Inhibitor

D. Cholesterol Absorption Inhibitor

The answer is C: HMG-CoA Reductase Inhibitor. Pravastatin is a STATIN medication which is part of
the HMG-CoA Reductase Inhibitor family.

2. Your patient is prescribed a statin medication for the treatment of hyperlipidemia. What medication
below may increase the patient’s risk of developing rhabdomyolysis if taken with a statin?

A. Ezetimibe

B. Gemfibrozil

C. Cholestyramine

D. Colesevelam

The answer is B. The usage of fibric acid derivatives (fibrates….such as Gemfibrozil or Fenofibrate)
and statins together can increase the risk of rhabdomyolysis (break down of skeletal muscle which
leads to renal failure).

3. Which statement below accurately describes how statin medications work to lower cholesterol
levels?

A. Statins increase the activity of LDL receptors in the liver by increasing the bioavailability of fibric-
acid.

B. Statins inhibit bile acid in the GI tract from being absorbed and as a result the liver turns
cholesterol into bile acid.

C. Statins increases hydroxymethylglutaryl–coenzyme A enzyme and this increases renal excretion of


cholesterol.
D. Statins inhibit HMG-CoA reductase which in turn hinders cholesterol synthesis in the liver.

The answer is D: Statins inhibit HMG-CoA reductase which in turn hinders cholesterol synthesis in the
liver.

4. Your patient is taking Simvastatin. Which of the following findings below demonstrate this
medication is providing therapeutic effects to this patient? Select-all-that-apply:

A. HDL 80 mg/dL

B. LDL 72 mg/dL

C. Total Cholesterol 350

D. HDL 30 mg/dL

E. LDL 190 mg/dL

F. Total Cholesterol 120

The answers are A, B, F. Statins should lower LDL (the bad cholesterol…..approximate normal range
<100), increase HDL (the good cholesterol…. approximate normal range >60), and lower total
cholesterol (normal range less than 200).

5. You’re providing education to a group of participants in the community about cholesterol and
cardiac disease. Which statements below verbalized by some of the participants are inaccurate and
require you to provide clarification?

A. “HDL levels should be less than 60 mg/dL to maintain good cardiac health.”

B. “LDL is a fatty substance that sticks to vessels walls and can lead to atherosclerosis.”

C. “HDL is a fatty substance that helps remove low-density lipoprotein from the blood so it doesn’t
have the chance to stick to vessel walls.”

D. “LDL levels should be greater than 100 mg/dL to maintain good cardiac health.”

The answers are A and D. These statements are false. LDL levels should be LESS (not greater) than
100 mg/dL, and HDL levels should be HIGHER (not lower) than 60 mg/dL. The other statements are
true.

6. A patient has been taking Simvastatin and reports to you that they are experiencing soreness and
weakness of the muscles. The physician orders lab work. What lab result below will you immediately
report to the physician?

A. Increased High-density Lipoprotein level

B. Decreased Creatine Kinase level

C. Decreased Low-density Lipoprotein level

D. Increased Creatine Kinase level


The answer is D: Increased Creatine Kinase level. Statins have a slight risk of causing “statin-induced
rhabdomyolysis”. This is the breaking down of skeletal muscle. If a patient reports muscle soreness or
weakness while taking a statin the CK levels (Creatine Kinase) should be measured. Creatine Kinase
is a protein found in skeletal muscle (it’s also found in the heart). If skeletal muscle is breaking down
CK will be found in the blood in high amounts and this tells us there is muscle damage. The CK level
will be 10 x’s the upper limit of normal…if this occurs the statin should be discontinued.

7. Your patient is prescribed Lovastatin due to their risk factors for developing cardiac disease. You’re
providing education about the importance of continuing exercise and a healthy diet while taking this
medication. You are helping the patient create a weekly food plan. Which food item below should the
patient avoid including in the food plan while taking this medication?

A. Bananas

B. Spinach

C. Grapefruit

D. Aged-cheese

The answer is C. Grapefruit or grapefruit juice can increase the toxicity of statins.

8. Your patient who is taking a statin develops yellowing of the skin, abdominal pain in the right upper
quadrant, and nausea. What lab result below correlates with this finding?

A. Elevated ALT and AST level

B. Elevated BUN and creatinine

C. Decreased WBC

D. Decreased Hbg

The answer is A. Liver injury is a risk with statins. Therefore, a liver functions test (LFTs) should be
ordered to assess the liver’s function. The patient is presenting with signs and symptoms of liver
injury. An elevated ALT and AST level would correlate with these presenting signs and symptoms.

1. A patient is experiencing Digoxin Toxicity. What is the antidote for this medication?

 A. Naloxone
 B. Digibind
 C. Acetylcysteine
 D. Protamine Sulfate
Answer...B. Digibind

2. Protamine Sulfate is an antidote for what medication below?

 A. Tricyclic antidepressants
 B. Warfarin
 C. Beta Blockers
 D. Heparin
Answer...D. Heparin

3. A patient is admitted to the hospital with Diazepam overdose. What is an antidote for this
medication?

 A. Flumazenil
 B. Naloxone
 C. Fomepizole
 D. Deferoxamine
Answer...A. Flumazenil

4. Vitamin K can be used to treat toxicity of what medication below?

 A. Anticholinergic medications
 B. Magnesium Sulfate
 C. Warfarin
 D. Heparin
Answer...C. Warfarin

5. Your patient is post-op from surgery. The patient is receiving Morphine. The patient is
experiencing severe respiratory depression. What medication below do you anticipate the
physician to order for this patient?

 A. Hydroxocobalamin
 B. Succimer
 C. Atropine
 D. Naloxone
Answer...D. Naloxone

6. What type of overdose does Acetylcysteine treat?

 A. Opioid
 B. Acetaminophen
 C. Ethylene glycol
 D. Benzodiazepines
Answer...B. Acetaminophen

7. A 76-year-old male is admitted to the ER. The patient accidently overdosed on Atenolol. The
patient's heart rate is 38 bpm and blood pressure 70/52. The patient is lethargic, sweaty, and
vomiting. What is the antidote for this medication?

 A. Digibind
 B. Calcium Gluconate
 C. Glucagon
 D. Calcium Disodium Edetate
Answer...C. Glucagon

8. Physostigmine is an antidote that can be used to treat what condition below?

 A. Cyanide Poisoning
 B. Cholinergic Toxicity
 C. Iron Toxicity
 D. Anticholinergic Toxicity
Answer...D. Anticholinergic Toxicity

9. A 53-year-old male is admitted to the ER. The patient is a farmer and was out in a field
spraying an insecticide that is an organophosphate agent. The patient is experiencing severe
muscle tremors, drooling, and diaphoresis. The diagnosis is organophosphate poisoning.
What medication below do you anticipate the physician to order for this patient?

 A. Atropine
 B. Glucagon
 C. Physostigmine
 D. Succimer
Answer...A. Atropine

10. The antidote for Magnesium Sulfate is?

 A. Calcium Disodium
 B. Calcium Gluconate
 C. Protamine Sulfate
 D. Acetylcysteine
Answer...B. Calcium Gluconate

11. Hydroxocobalamin is an antidote to treat what condition below?

 A. Iron Toxicity
 B. Lead Toxicity
 C. Cyanide Poisoning
 D. Ethylene Glycol Poisoning
Answer...C. Cyanide Poisoning

12. EMS calls in to report that they are transporting a 22-year-old patient who attempted
suicide by ingesting Ethylene Glycol. Prior to the patient's arrival, what medication will you
have readily available?

 A. Fomepizole
 B. Naloxone
 C. Deferoxamine
 D. Acetylcysteine
Answer...A. Fomepizole

13. A child's lab result demonstrates lead toxicity. What is an antidote for this condition?

 A. Vitamin K
 B. Calcium disodium edetate
 C. Flumazenil
 D. Hydroxocobalamin
Answer...B. Calcium disodium edetate

14. Deferoxamine is an antidote used to treat what condition below?

 A. Iron Toxicity
 B. Lead Toxicity
 C. Digoxin Toxicity
 D. Cholinergic Toxicity
Answer...A. Iron Toxicity
1. A patient is prescribed a beta blocker for a cardiac condition. You know this medication
blocks the beta receptors in the body so ____________ and __________ cannot bind to the
receptor site and elicit a _______ ________ _________ response.

 A. angiotensin II and angiotensin I; sympathetic nervous system


 B. dopamine and norepinephrine; parasympathetic nervous system
 C. norepinephrine and epinephrine; sympathetic nervous system
 D. dopamine and acetylcholine; parasympathetic nervous system
The answer is C. Beta blockers block the beta receptors in the body so norepinephrine and
epinephrine cannot bind to the receptor site and elicit a sympathetic nervous system response.

2. Beta 1 receptors can be found in the? Select all that apply:

 A. GI tract
 B. Uterus
 C. Heart
 D. Lungs
 E. Kidneys
 F. Vascular smooth muscle
The answers are C and E. Beta 1 receptors can be found in the HEART and KIDNEYS.

3. Beta 2 receptors can be found in the? Select all that apply:

 A. Heart
 B. Lungs
 C. GI system
 D. Kidneys
 E. Vascular smooth muscle
 F. Skeletal muscle
The answers are B, C, E, and F. Beta 2 receptors can be found in the lungs (bronchioles), GI system,
vascular smooth muscle, skeletal muscle and even the ciliary body of the eye (not listed).

4. A patient is admitted with a dysrhythmia. The physician prescribes Propranolol. Which


statement by the patient requires the nurse to hold the ordered dose and notify the physician
for further orders?

 A. "I use an inhaler at home for asthma."


 B. "My heart feels like it is racing, and I feel very weak."
 C. "I had caffeine this morning with breakfast."
 D. "I smoke 2 packs of cigarettes per day."
The answer is A. Propranolol is a nonselective beta blocker. Therefore, it affects beta 1 and beta 2
receptors. Beta 2 receptors are found in the lungs (specifically the bronchioles). This medication
could cause bronchoconstriction in patients with asthma or COPD and should be avoided. Selective
beta blockers (Atenolol, Esmolol, Metorprolol) block only beta 1 (which are found in the heart and
kidneys). Therefore, they are less likely to cause bronchoconstriction.

5. Select all the beta blocker medications listed below that affect ONLY beta 1 receptors:

 A. Timolol
 B. Atenolol
 C. Metoprolol
 D. Esmolol
The answers are B, C, D. Atenolol, Metoprolol, and Esmolol are selective and affect ONLY beta 1
receptors, which are found in the heart and kidneys. Timolol is a nonselective beta blocker and affects
both beta 1 and beta 2 receptors.

6. At 1000 your patient is scheduled to take a dose of Atenolol. What finding below would
require you to hold the scheduled dose and notify the physician?

 A. The patient's heart rate is 120 beats per minute.


 B. The patient's blood pressure is 102/76.
 C. The patient has swelling in lower extremities, dyspnea, and crackles in lung
fields.
The answer is C. In option C, the patient is showing signs and symptoms of uncompensated heart
failure (beta blockers are not used when uncompensated heart failure presents, but they can be
sometimes be used when the patient is in compensated/stable heart failure). The reason is because
beta blockers slow down the heart rate and decrease the strength of contractions (negative inotropic
effect), which can lead to heart failure in SOME patients, and this is why the nurse must monitor for
this.

7. A patient with diabetes and hypertension is being discharged home. The patient will be
taking Sotalol and insulin per sliding scale. Which statement by the patient demonstrates they
did NOT understand your discharge instructions about the side effects of Sotalol?

 A. "This medication can affect my blood glucose levels."


 B. "I will monitor my heart rate and blood pressure everyday while taking this
medication."
 C. "While taking this medication I will monitor for a fast heart rate because this is
an early indication that my blood glucose level is low."
 D. "I will report to my physician if I develop shortness of breath, weight gain, or swelling
in my feet."
The answer is C. Beta blockers can prevent tachycardia (increased heart rate). An increased heart
rate is a sign for most diabetics that they are experiencing a low blood glucose level. Therefore, the
patient who is taking beta blocker (especially a nonselective type) needs to be re-educated that a fast
heart rate is NOT a reliable way of determining hypoglycemia and should check their blood glucose
regularly.

8. Which category of beta blockers block beta 1 and beta 2 receptors?

 A. Selective beta blockers


 B. Nonselective beta blockers
The answer is B. Nonselective beta blockers BLOCK beta 1 and beta 2...therefore they're not
selective. Selective beta blockers block ONLY beta 1 receptors and are sometimes referred to as
cardioselective.

9. A patient is prescribed Metoprolol. Which statement by the patient requires the nurse to re-
educate the patient on how to take the medication properly?

 A. "After I stop taking this medication I will let my physician know."


 B. "I take this medication with my breakfast every morning."
 C. "I will change positions slowly while I'm taking this medication."
 D. "While I'm taking this medication I will monitor my heart rate."
The answer is A. The patient should NOT just stop taking the medication. It must be tapered off over
a period of time (usually about 2 weeks). This will prevent the development of rebound hypertension,
myocardial ischemia, and angina.

10. What signs and symptoms below would demonstrate a patient is experiencing an
overdose of a beta blocker medication? Select all that apply:

 A. Blood pressure 200/110


 B. Heart rate 35 beats per minute
 C. EKG shows atrial fibrillation with rapid ventricular rate (RVR)
 D. Patient is maniac and agitated
 E. Dyspnea
 F. Patient is severely drowsy
 G. EKG shows 3rd degree AV block
The answers are B, E, F, and G. Signs and symptoms of beta blocker overdose would be the
opposite of the effects of the sympathetic nervous system. Beta blockers slow down the heart rate
(overdose: severe bradycardia...heart rate of 35 and heart block 2nd or 3rd degree), decrease blood
pressure (overdose: severe hypotension), and severely drowsy (due to very slow heart rate and
decreased cardiac output to the brain).

1. The physician prescribes the patient a potassium-sparing diuretic. Which statement below
best describes how this medication works to cause diuresis?

 A. These medications work to inhibit the sodium-chloride cotransporter in the early part of
the distal convoluted tubule.
 B. These medications work to inhibit the sodium-potassium-chloride cotransporter in the
thick ascending limb of the loop of Henle.
 C. These medications work to inhibit the sodium and potassium exchange within
the sodium channels in the distal tubule and collecting duct.
 D. These medications work to inhibit the sodium channels within the proximal convoluted
tubule by decreasing sodium reabsorption.
The answer is C. Potassium-sparing diuretics work to inhibit the sodium and potassium exchange
within the sodium channels in the distal tubule and collecting duct.

2. Which potassium-sparing diuretics are known to antagonize aldosterone in order to cause


diuresis? Select all that apply:

 A. Triamterene
 B. Spironolactone
 C. Amiloride
 D. Eplerenone
The answers are B and D. Spironolactone and Eplerenone are types of potassium-sparing diuretics
that are known as the aldosterone antagnoists or aldosterone receptor blockers. These medications
work against aldosterone (hence the name aldosterone "antagnoists"). Aldosterone normally works to
influence epithelial sodium channels and the sodium-potassium pumps in the distal tubule and
collecting duct by increasing this number. This leads to the reabsorption of sodium into the
bloodstream from the filtrate and secretion of potassium into the filtrate from the blood. However,
when this is inhibited (due to these medications) sodium is NOT reabsorbed back into the blood and
potassium is NOT secreted into the filtrate but stays in the blood (hence this can lead to
HYPERkalemia). Triamterene and Amiloride are potassium-sparing diuretics, BUT they work to inhibit
the epithelial sodium channels, which causes the same affects as the aldosterone antagonists but in
a different way.

3. Potassium-sparing diuretics alter how sodium is reabsorbed in what part of the nephron?
Select all that apply:

 A. Loop of Henle
 B. Proximal Convoluted Tubule
 C. Distal Convoluted Tubule
 D. Collecting Duct
 E. Thick ascending limb of the loop of Henle
The answers are C and D. Potassium-sparing diuretics work in the distal parts of the nephron (the
late part of the distal tubule and collecting duct) to alter the sodium and potassium exchange within
the nephron.

4. TRUE or FALSE: Potassium-sparing diuretics are the most effective type of diuretic and are
often combined with loop or thiazide diuretics.

 True
 False
The answer is FALSE. Potassium-sparing diuretics are the WEAKEST (not most effective) type of
diuretic. However, yes, they are often combined with loop or thiazide diuretics to help prevent
HYPOkalemia, which is a common side effect of loop and thiazide diuretics.

5. What conditions below are potassium-sparing diuretics ordered to treat? Select all that
apply:

 A. Edema due to renal failure


 B. Hyperaldosteronism
 C. Hypertension
 D. Hypokalemia secondary to loop or thiazide diuretic usage
The answers are B, C, and D. Potassium-sparing diuretics are NOT for patients who have renal
failure (therefore, option A is wrong). Yes, they treat edema/swelling due to heart failure, liver
impairment, or nephrotic syndrome, but are contraindicated in patients with renal failure. However,
potassium-sparing diuretics help treat hyperaldosteronism (specifically Spironolactone).
Hyperaldosteronism is where the body produces too much aldosterone, which leads to low potassium
levels, high sodium levels, and hypertension. Spironolactone will antagonize the aldosterone and
decrease it. Many times potassium-sparing diuretics are ordered with loop or thiazide diuretics to treat
hypertension. In addition, since potassium-sparing diuretics increase the blood level of potassium,
they may be ordered for patients who are experiencing hypokalemia while taking diuretics that waste
potassium (loop or thiazide diuretics).

6. A patient is currently taking a potassium-sparing diuretic. The patient is experiencing EKG


changes with tall-peaked T-waves, nausea, diarrhea, and paresthesia. The patient's morning
lab results are back and the nurse makes it priority to check what lab result?

 A. Calcium
 B. Potassium
 C. Magnesium
 D. Sodium
The answer is B. Potassium-sparing diuretics can increase a patient's potassium level and based on
the patient's signs and symptoms in the scenario above this correlates with hyperkalemia. The nurse
should check the morning lab result for potassium.

7. A patient has a dose of Spironolactone due at 1000. Which finding below would require the
nurse to hold the dose and notify the physician for further orders?

 A. Magnesium 1.5 mg/dL


 B. BUN 18
 C. Sodium 140 mEq/L
 D. Potassium 7 mEq/L
The answer is D. A normal potassium level is 3.5-5 mEq/L. Spironolactone can increase potassium
levels because it's a potassium-sparing diuretic. Therefore, the nurse should hold the scheduled dose
and notify the MD for further orders.

8. Your patient is prescribed to take Lisinopril and Triamterene. As the nurse you know that
these medications increase a patient's risk for developing?

 A. Hyperkalemia
 B. Hypertension
 C. Hypernatremia
 D. Renal failure
The answer is A. Lisinopril is an ACE inhibitor (ACEI) and this medication can increase potassium
levels due to how it affects the RAAS. Triamterene is a potassium-sparing diuretic, which can
increase potassium levels. Therefore, the nurse should monitor for HYPERkalemia.

9. You're providing discharge teaching to a patient who will be taking Amiloride. Which items
below should the patient limit in their diet? Select all that apply:

 A. Salt-substitutes
 B. Spinach
 C. Bananas
 D. Rice
 E. Cheese
The answers are A, B, and C. All of these items are high in potassium. Amiloride is a potassium-
sparing diuretic that can increase the blood levels of potassium. Therefore, the patient should limit
these items.

10. Which potassium-sparing diuretic can cause antiandrogenic effects in a patient, such as
menstrual irregularities, gynecomastia, and decreased sex drive?

 A. Triamterene
 B. Spironolactone
 C. Amiloride
 D. Eplerenone
The answer is B. Spironolactone can have antiandrogen effects on some patients (gynecomastia,
increase hair growth in women, menstrual irregulars, sexual dysfunction). This is because
Spironolactone affects the androgen and progesterone receptors.

1. The physician prescribes the patient a thiazide diuretic. The patient is experiencing
increased urination. As the nurse you know that this medication causes this type of affect by
how it affects the nephron. It does this by?

 A. inhibiting the sodium-potassium-chloride cotransporter in the distal convoluted


tubule
 B. inhibiting the sodium-chloride cotransporter in the distal convoluted tubule
 C. inhibiting the water reabsorption in the distal convoluted tubule
 D. inhibiting the sodium-potassium transporter in the distal convoluted tubule
The answer is B. Thiazide diuretics inhibit the sodium-chloride cotransporter in the early part of the
distal convoluted tubule. When this occurs it prevents sodium from being reabsorbed back into the
blood. Since the sodium stays in the tubule, it will keep water with it. This will decrease the amount of
water reabsorbed into the blood/body…thus increasing urination and extra fluid removal from the
blood (causing a diuretic effect).

2. Your patient will be started on a thiazide diuretic to help manage blood pressure. Select all
the medications below that are considered a type of thiazide diuretic:

 A. Furosemide
 B. Indapamine
 C. Hydrochlorothiazide
 D. Spironolactone
 E. Metolazone
 F. Chlorothiazide
The answers are B, C, E, and F. Option A is a loop diuretic, and option D is a potassium-sparing
diuretic.

3. True or False: Thiazide diuretics are more potent than loop diuretics and provide immediate
diuresis affects.

 True
 False
The answer is FALSE. Thiazide diuretics are NOT more potent than loop diuretics and do not provide
immediate diuresis affects like loop diuretics. Loop diuretics inhibit sodium reabsorption in the loop of
Henle, which is responsible for about 25% of sodium reabsorption….whereas, thiazide diuretics
inhibit sodium reabsorption in the early part of the distal convoluted tubule, which is responsible for
about 5-7% of sodium reabsorption. Therefore, the higher percentage of sodium reabsorption
inhibited the stronger the effects, which makes loop diuretics more potent.

4. A 55-year-old-male with hypertension is prescribed a thiazide diuretic. The patient has a


health history of diabetes type I and gout. What lab results below represent some known side
effects of this medication and requires the nurse to report the lab results to the physician?
Select all that apply:

 A. Potassium 5.2 mEq/L


 B. Calcium 9 mg/L
 C. Blood glucose 300 mg/dL
 D. Ammonia 20 mcg/dL
 E. Uric acid level 15 mg/dL
 F. Blood glucose 45 mg/dL
The answers are C and E. Thiazide diuretics can cause an increase in blood glucose
(hyperglycemia), hyperuricemia (high uric acid levels…this increases a gout attack), hypokalemia
(low potassium levels), and hypercalcemia (high calcium levels). Due to the patient’s health history of
diabetes and gout, the nurse should monitor the patient’s glucose and uric acid levels. A normal blood
glucose level is about 70-100 mg/dL, and a normal uric acid level varies for female and males…with
anything greater than 7 for males or 6 for female being considered high. Therefore, the patient’s uric
acid level of 15 mg/dL and glucose of 300 mg/dL is abnormal and should be reported.

5. A patient is scheduled to take a dose of Lithium at 1000. The patient is also prescribed a
thiazide diuretic. What lab result below requires the nurse to hold the dose of Lithium and
notify the physician for further orders?

 A. Potassium 3.2 mEq/L


 B. Sodium 115 mEq/L
 C. Calcium 10.5 mg/L
 D. Magnesium 2 mg/dL
The answer is B. The nurse must monitor for Lithium toxicity in patients who are taking Lithium and a
thiazide diuretic. These types of diuretics lower the amount of sodium in the blood because it
prevents the reabsorption of it within the nephron…so more is excreted into the urine. Therefore, low
sodium levels in the blood INCREASE the risk of Lithium toxicity. A normal blood sodium level is 135-
145 mEq/L.

6. Your patient is prescribed hydrochlorothiazide. The patient’s potassium level is 2 mEq/L.


What medication below should the nurse hold and notify the physician for further orders?

 A. Dilantin
 B. Digoxin
 C. Warfarin
 D. Lithium
The answer is B. Hypokalemia (low potassium level in the blood) can increase Digoxin toxicity.
Hydrochlorothiazide causes the body to lose potassium. A normal blood potassium level is 3.5-5
mEq/L. Therefore, the nurse should hold the Digoxin and notify the physician for further orders.

7. Based on your nursing knowledge of how thiazide diuretics work, which patients below
would benefit from these types of medications? Select all that apply:

 A. A patient with a glomerular filtration rate (GFR) of less than 30 cc/hr.


 B. A patient with a recurrent history of renal calcium calculi.
 C. A patient with primary hypertension.
 D. A patient with heart failure and frequent gout attacks.
 E. A patient with diabetes that has uncontrolled hyperglycemia.
The answers are B and C. Why? Option B: thiazide diuretics help INCREASE reabsorption of calcium
into the blood (hence causes hypercalcemia)…therefore it removes it from the urine and can help
prevent renal calculi that are composed of calcium. Option C: thiazide diuretics help remove extra
fluid volume in the blood and this can decrease blood pressure. Thiazides are not for patients who
have a decrease in renal function (example GFR of less than 30 cc/hr), gout attacks (they can cause
an increase in uric acid levels), and uncontrolled hyperglycemia (they increase blood glucose levels).

8. You’re developing a plan of care for a patient with heart failure that will be prescribed a
thiazide diuretic. What nursing interventions will you include in this patient’s plan of care?
Select all that apply:

 A. Encourage the patient to limit the consumption of bananas, avocadoes, spinach,


strawberries, and potatoes.
 B. Measure the patient’s intake and output daily.
 C. Weigh the patient daily using a bedside scale.
 D. Assess lab results for electrolyte imbalances like hypercalcemia and hyperkalemia.
The answers are B and C. Option A is wrong because the nurse should encourage the patient to
consume (NOT limit) these foods because they are high in potassium. Remember thiazide diuretics
waste potassium. Option D is wrong because these medications can cause electrolyte imbalances
like hypercalcemia and HYPOkalemia (NOT hyperkalemia). Options B and C are correct because
these medication increase urination (hence they cause fluid depletion)…therefore, the nurse must
monitor the patient’s fluid status.

9. You’re providing discharge teaching to a patient who will be taking a thiazide diuretic for the
treatment of hypertension. You note that the patient has type I diabetes. What statement by the
patient represents they misunderstood your teaching points about this medication and
requires you reinforce some of the education pieces?

 A. “I must monitor my blood glucose closely while taking this medication.”


 B.” This medication can cause sudden and severe drops in my blood glucose.”
 C. “I will try my best to eat a diet that includes foods rich in potassium.”
 D. “I will report to my physician if I experience excessive thirst, little or no urination, and
extreme fatigue.”
The answer is B. Thiazide diuretics can INCREASE the blood glucose (lead to hyperglycemia)… not
hypoglycemia. Therefore, the nurse should educate the patient to monitor the blood glucose level
closely.

1. The physician prescribes the patient a loop diuretic. As the nurse you know that this type of
diuretic causes diuresis by MAINLY affecting what structure in the nephron?

 A. Distal convoluted tubule


 B. Descending limb of the loop of Henle
 C. Proximal convoluted tubule
 D. Ascending limb of the loop of Henle
The answer is D. Loop diuretics affect the loop of Henle, specifically the ASCENDING limb (the thick
part of this limb).

2. Loop diuretics are effective with inhibiting sodium reabsorption within the nephron because
it inhibits?

 A. the sodium-chloride transporter


 B. the effects of aldosterone on the distal convoluted tubule
 C. the sodium-potassium-chloride cotransporter
 D. the transport of bicarbonate by the proximal convoluted tubule
The answer is C. Loop diuretics INHIBIT the sodium-potassium-chloride (NKCC2) cotransporter in the
thick ascending limb of the loop of Henle. This inhibits the amount of sodium that is reabsorbed by the
kidneys, which will cause the nephron to decrease the amount of water it reabsorbs...hence leading
to more water leaving the kidneys via the urine (leading to its diuretics affects). Option A is how
thiazide diuretics work, option B is how potassium-sparing diuretics work, and option D is how
carbonic anhydrase inhibitors work.

3. Select all the medications below that are considered loop diuretics:

 A. Chlorothiazide
 B. Bumetanide
 C. Triamterene
 D. Spironolactone
 E. Furosemide
 F. Torsemide
 G. Hydrochlorothiazide
The answers are B, E, and F. Options A and G are thiazide diuretics, and Options C and D are
potassium-sparing diuretics.

4. Your patient is ordered a loop diuretic at 1000. Which finding below would require you to
hold the dose and notify the physician for further orders?

 A. Calcium level 9 mg/L


 B. Potassium level 1.5 mEq/L
 C. Blood pressure 102/78
 D. Sodium level 144
The answer is B. Loop diuretics are known to decrease potassium levels due to the way they affect
how the nephron wastes potassium (this happens in the distal convoluted tubule due to the high
concentrated amount of sodium in the filtrate...this tubule will exchange potassium and hydrogen ions
for sodium ions under the influence of aldosterone, which will lead potassium to enter the filtrate and
exit the body as urine.)

5. You're developing a plan of care for a patient with fluid volume overload related to heart
failure exacerbation. The physician has prescribed an IV loop diuretic. What nursing
interventions will you include in the patient's plan of care? Select all that apply:

 A. Perform and assess daily weights.


 B. Educate the patient about consuming a low potassium diet.
 C. Strict measuring of the patient's daily intake and output.
 D. Encourage the patient to drink 2 L of fluids per day.
 E. Assess lung sounds every shift.
The answers are A, C, and E. The patient with heart failure exacerbation can experience fluid volume
overload because the heart is failing to pump blood forward. This causes blood to backflow into the
lungs causing pulmonary edema and respiratory distress. In addition, it can cause edema in the
extremities. A loop diuretic will help remove this extra fluid by altering the way the kidneys reabsorb
sodium and water. Due to the way loop diuretic works by removing extra fluid, the nurse should
monitor the patient for dehydration, effectiveness of the medication (decrease in edema, clear lung
fields), electrolyte imbalances, and measuring intake and output. Therefore daily weights, measuring I
and O's, and assessing lungs sounds are the answers. Option B is not the answer because the
patient should be consuming a diet with potassium (loop diuretics can decrease the potassium level).
Option D is wrong because this is too much fluid for a patient to consume with heart failure. This
medication would be unable to do its job if the patient consumed 2L of fluid per day...it would not be
effective.

6. Which patients below may be prescribed a loop diuretic based on their diagnosis? Select all
that apply:

 A. A 58-year-old male with congestive heart failure.


 B. A 69-year-old female with pulmonary edema.
 C. A 45-year-old male with hypercalcemia.
 D. A 50-year-old male experiencing a gout attack.
The answers are A, B, and C. Option D is not a candidate for loop diuretics because these
medications can increase uric acid levels, which is already an issue for this patient with a gout attack
(remember gout occurs due to increase uric acids levels). Loop diuretics help remove extra fluid from
the blood. This is helpful for patients with heart failure and pulmonary edema. Loop diuretics also
decrease calcium reabsorption (hence causing the calcium to be excreted rather than staying in the
blood), and this would help treat a high calcium level (hypercalcemia).

7. A patient with heart failure is prescribed Digoxin and a loop diuretic. The nurse knows that
what finding below would increase a patient's risk of developing Digoxin toxicity?

 A. Magnesium level 1.8 mg/dL


 B. Potassium 2 mEq/L
 C. BUN 15
 D. Albumin 5 g/dL
The answer is B. Hypokalemia (low potassium level in the blood) increases the risk of Digoxin toxicity
(normal potassium 3.5-5 mEq/L). Therefore, the nurse should always check a patient's potassium
result before administering Digoxin, especially if they are taking a loop diuretic (remember loop
diuretics waste potassium and can decrease the blood level).
8. You're educating a group of new nurse graduates about loop diuretics. One topic you
discuss is ototoxicity. You ask the new nurses to explain how to prevent this adverse side
effect in a patient prescribed a loop diuretic. Which response by one of the new nurses is
correct?

 A. "Always administer intravenous loop diuretics slowly."


 B. "Monitor for signs and symptoms of low potassium levels because this increases the
risk of inner ear damage while taking loop diuretics."
 C. "Administer the medication with meals."
 D. "Avoid administering loop diuretics with a tetracycline."
The answer is A. Rapid IV administration of a loop diuretic can increase the risk of ototoxicity in a
patient taking loop diuretics. Always give these medications slowly when ordered the IV route.

9. You're providing discharge instructions to a patient who will be taking a loop diuretic at
home. Which statement by the patient demonstrates they did NOT understand the teaching
material and requires that you reinforce some of the teaching points?

 A. "I will eat a diet rich in potatoes, bananas, avocadoes, strawberries, and spinach."
 B. "I will weight myself daily and report to my physician if I gain more than 3 lbs in 1 day."
 C. "I will change position slowly because I can become dizzy easily while taking this
medication."
 D. "This medication can cause dehydration, so I will stay hydrated by consuming
at least 2 L or more of fluid per day."
The answer is D. The patient should be educated on the signs and symptoms of dehydration
(excessive thirst, fatigue, hypotension etc.). However, they should not be taught to consume 2L or
more of fluid per day. Many patients are prescribed loop diuretics to treat issues with fluid volume
overload (example heart failure). These patients must monitor how much fluid they are drinking. If too
much fluid is consumed this will cancel out the effectiveness of the medication.

10. What electrolyte imbalances would the nurse monitor for in the patient who is taking a
loop diuretic? Select all that apply:

 A. Hyperkalemia
 B. Hypocalcemia
 C. Hypernatremia
 D. Hypokalemia
 E. Hypomagnesemia
The answers are B, D, and E. Loop diuretics can cause hypokalemia, hyponatremia, and
hypomagnesemia.
1. You’re teaching a patient about how angiotensin II receptor blockers (ARBs) work. Which
statement below BEST describes how these medications work on the body?

 A. “They prevent Angiotensin II Type I Receptors from binding with Angiotensin


II.”
 B. “These medications prevent the activation of Angiotensin II Type II Receptors from
binding with Angiotensin II.”
 C. “They inhibit angiotensin-converting-enzyme (ACE) from converting an Angiotensin I
to Angiotensin II.”
 D. “These medications prevent Angiotensin II Type I Receptors from binding with
angiotensin-converting-enzyme (ACE).”
The answer is A. ARBs prevent Angiotensin II Type I Receptors from binding with Angiotensin II.
Remember ACE Inhibitors inhibit angiotensin-converting-enzyme (ACE) from converting Angiotensin I
to Angiotensin II.

2. A physician writes a new medication order for a patient who has cardiovascular disease.
The medication is an angiotensin II receptor blocker (ARB). What new medication on the
patient’s scheduled medication list is an ARB?

 A. Metoprolol
 B. Losartan
 C. Lisinopril
 D. Nicardipine
The answer is B. Remember ARBs end with "SARTAN". Losartan is an ARB.

3. A patient who developed a dry, persistent cough while taking an ACE Inhibitor is switched to
an angiotensin II receptor blocker (ARB). The patient reports the cough is now gone, but asks
you to explain how this medication helped alleviate the cough. What is the correct response?

 A. “ARBs prevent ACE (angiotensin-converting-enzyme) from breaking down bradykinin


so a dry, persistent cough is less likely.”
 B. “ARBs increase ACE (angiotensin-converting-enzyme) which helps decrease
bradykinin levels and helps alleviate the cough.”
 C. “ARBs do not inhibit ACE (angiotensin-converting-enzyme), which is a
substance that inactivates bradykinin by breaking it down; therefore, a cough is
not likely with this medication.”
 D. “ARBs prevent Angiotensin II Type I receptor sites from activating bradykinin in the
lungs.”
The answer is C. ACE Inhibitors can cause a dry, persistent cough in some patients because it
inhibits ACE from inactivating bradykinin by breaking it down….therefore, levels of bradykinin
increase and lead to coughing. ARBs do NOT inhibit ACE (angiotensin-converting-enzyme), which is
a substance that inactivates bradykinin by breaking it down; therefore, a cough is not likely with this
medication.

4. Select all the pharmacodynamic effects of angiotensin II receptor blockers (ARBs):

 A. Vasodilation
 B. Vasoconstriction
 C. Sodium conservation
 D. Sodium excretion
 E. Water conservation
 F. Water excretion
The answers are A, D, and F. ARBs prevent Angiotensin II Type I Receptors from binding with
Angiotension II. This leads to vasodilation of vessels and decreases the release of aldosterone, which
leads to sodium and water excretion (potassium is conserved…so watch out for hyperkalemia).

5. What conditions are Angiotensin II Receptor Blockers (ARBs) used to treat? Select all that
apply:

 A. Hypertension
 B. Renal stenosis
 C. Diabetic nephropathy in type 2 diabetics
 D. Atrial flutter
 E. Heart failure
The answers are A, C, and E. ARBs can treat hypertension, diabetic nephropathy in type 2 diabetics
(which is kidney disease in type 2 diabetics), and heart failure.

6. A patient is taking an Angiotensin II Receptor Blocker (ARB). What finding below is an


adverse side effect of this medication and should be reported to the physician?

 A. BUN 10
 B. Creatinine 1 mg/dL
 C. Potassium 6.8 mEq/L
 D. ALT 20 U/L
The answer is C. ARBs can lead to hyperkalemia due to how they decrease the release of
aldosterone and cause the kidneys to excrete sodium and water, but conserve potassium. A normal
potassium level is 3.5-5 mEq/L.
7. As the nurse you know to tell the patient that the best way to PREVENT rebound
hypertension while taking an Angiotensin II Receptor Blocker (ARB) is to?

 A. assess the blood pressure daily with a self-monitoring device


 B. never abruptly stop taking the medication
 C. avoid taking the medication with milk or grapefruit juice
 D. avoid taking over-the-counter medications
The answer is B. The patient should NEVER just quit taking the medication because rebound
hypertension can occur.

8. You’re providing discharge teaching to a patient that will be taking an Angiotensin II


Receptor Blocker (ARB) at home. What statement by the patient requires you to re-educate
them about this medication?

 A. “This medication does not cure hypertension. Therefore, I will need to also make
lifestyle changes.”
 B. “I will always stand and change positions slowly.”
 C. “A persistent, dry cough is not common with this medication.”
 D. “This medication can decrease potassium levels. So, I will consume a diet rich
in potassium to help keep my level normal.”
The answer is D. ARBs can cause a HIGH potassium level. Therefore, the patient should avoid
consuming a diet rich in potassium and using salt-substitutes with potassium.

9. Your patient is prescribed an Angiotensin II Receptor Blocker (ARB) for the treatment of
blood pressure management. Which statement below BEST describes how this medication
will manage blood pressure in a patient?

 A. “This medication will increase systemic vascular resistance (SVR) and decrease blood
pressure, while decreasing blood volume through the excretion of sodium and water.”
 B. “This medication will decrease systemic vascular resistance (SVR) and
decrease blood pressure, while decreasing blood volume through the excretion of
sodium and water.”
 C. “ARBs will cause vasoconstriction and increase renal blood flow and volume.”
 D. “ARBs will cause vasodilation and the kidneys to retain sodium and water.”
The answer is B. ARBs will decrease systemic vascular resistance (SVR) and decrease blood
pressure, while decreasing blood volume through the excretion of sodium and water.”
1. A 65-year-old male patient is prescribed an ACE inhibitor for the treatment of hypertension.
Which medication below is an ACE inhibitor?

 A. Metoprolol
 B. Benazepril
 C. Losartan
 D. Amlodipine
The answer is B. Benazepril is an ACE Inhibitor. Remember ACE Inhibitors end in PRIL.

2. Which statements below CORRECTLY describe how ACE Inhibitors work? Select all that
apply:

 A. This group of medications inhibits the renin-angiotensin-aldosterone system


(RAAS).
 B. ACE Inhibitors prevent the conversion of Angiotensin I to Angiotensin II.
 C. ACE Inhibitors prevent Angiotensinogen from converting to Angiotensin I.
 D. ACE Inhibitors have a positive chronotropic and negative inotropic effect on the heart.
The answers are A and B. ACE inhibitors inhibits the renin-angiotensin-aldosterone system (RAAS),
which will prevent the conversion of Angiotensin I to Angiotensin II. Options C and D are false
statements about these medications.

3. Angiotensin-converting enzyme (ACE) performs what roles in the body? Select all that
apply:

 A. Inactivates bradykinin by breaking it down


 B. Dilates vessels
 C. Causes the kidneys to keep sodium and water
 D. Converts Angiotensin I to Angiotensin II
The answers are A and D. ACE inactivates bradykinin by breaking it down and converts Angiotensin I
to Angiotensin II. Option C describes the role of aldosterone, which is influenced by Angiotensin II,
and option D describes how ACE Inhibitors work….remember they block the actions of ACE.

4. Fill in the blanks: Angiotensin II causes ___________ of the vessels and triggers the release
of ____________.

 A. vasodilation; anti-diuretic hormone (ADH)


 B. vasodilation; aldosterone
 C. vasoconstriction; aldosterone
 D. vasoconstriction; anti-diuretic hormone (ADH)
The answer is C. Angiotensin II causes vasoconstriction of the vessels and triggers the release of
aldosterone.

5. A patient is prescribed an ACE Inhibitor after experiencing a myocardial infarction. What


effects on the body will this medication achieve? Select all that apply:

 A. Decreases SVR (systemic vascular resistance) and blood pressure


 B. Constriction of the vessels
 C. Kidneys will excrete water and sodium
 D. Kidneys will retain potassium.
 E. Increases SVR (systemic vascular resistance) and blood pressure
The answers are A, C, and D. ACE inhibitors will cause the opposite effects of Angiotensin II, which is
a major vasoconstrictor and triggers the release of aldosterone (remember this substance will cause
the kidneys to keep sodium and water and excrete potassium). However, ACE Inhibitors will perform
the opposite effects by decreasing the blood pressure via vasodilation of vessels (which decreases
SVR) and causes the kidneys to excrete water and sodium and retain potassium (hence the nurse
must monitor for hyperkalemia).

6. Which patient below would MOST benefit from an ACE Inhibitor?

 A. A 50-year-old female with systolic dysfunction heart failure.


 B. A 48-year-old male with severe renal failure.
 C. A 35-year-old female with chronic hepatitis.
 D. A 54-year-old male with hypovolemic shock.
The answer is A. ACE Inhibitors are used to treat patient with hypertension, systolic dysfunction heart
failure (the left ventricle is too weak to pump blood forward out of the heart…ACE inhibitors will
decrease afterload which will make it easier for the heart to pump blood), and after a myocardial
infarction. Patients with renal failure or liver disease are not candidates for ACE inhibitors because of
the effects of the drug on the kidneys and how it is cleared in the liver. In option D, the patient will be
hypotensive and need fluids or vasopressors….NOT a medication that will lower the blood pressure.

7. Your patient is taking an ACE Inhibitor to manage blood pressure. Which finding below
requires immediate nursing action?

 A. Urinary output is 190 mL within the past 4 hours.


 B. Patient has a persistent, dry cough.
 C. EKG shows tall, peaked t-waves.
 D. Patient has a negative Chvostek’s sign.
The answer is C. This EKG finding demonstrates hyperkalemia. Remember ACE Inhibitors can cause
a high potassium level because the kidneys will keep potassium, but excrete water and sodium (so it
has a diuretic effect too).

8. Some patients who take ACE Inhibitors may develop angioedema. What signs and
symptoms will you teach the patient to recognize that can present with this adverse reaction?
Select all that apply:

 A. Hyperkalemia
 B. Persistent, dry cough
 C. Swelling in the face
 D. Thin and shiny skin in the lower extremities
 E. Difficulty breathing
The answers are C and E. Angioedema is swelling deep in the skin (dermis and subcutaneous tissue)
…..it’s very dangerous! Signs and symptoms include: swelling in face (mouth, eyes, tongue, lips,
dyspnea, swelling of extremities). It most commonly occurs in African American patients.

9. A patient states they are experiencing an annoying, persistent dry cough that started once
they begin taking an ACE Inhibitor. The patient is not experiencing any other signs and
symptoms. As the nurse, your response is?

 A. Tell the patient to immediately stop taking the medication and seek medical treatment.
 B. Reassure the patient this is a harmless side effect of this medication and to not
abruptly stop taking the medication.
 C. Recommend the patient start taking the medication at night to decrease the coughing.
 D. Reassure the patient that the cough will disappear within 6 months of taking the
medication.
The answer is B. A persistent, dry cough can occur in some patients taking ACE Inhibitors and is
usually harmless. It is due to how ACE Inhibitors affect bradykinin (an inflammatory substance in the
body). A patient should NEVER abruptly stop taking an ACE inhibitor because it can cause rebound
hypertension. If the patient cannot tolerate the cough, they should speak with the MD about switching
to another type of medication.

10. You’re providing discharge instructions to a patient that will be taking an ACE Inhibitor at
home. Which statements by the patient demonstrate they understood your discharge
instructions? Select all that apply:

 A. “If I feel unwell, it is okay that I miss a dose.”


 B. “I will avoid using salt substitutes that contain potassium.”
 C. “I will make sure I incorporate a high amount of potatoes, bananas, oranges, and
tomatoes into my diet while taking this medication”.
 D. “I will regularly check my blood pressure and pulse rate while taking this
medication and report any significant changes to my doctor.”
The answers are B and D. It is very important a patient does not miss a dose of this medication (even
if they are unwell) because this medication can cause rebound hypertension. Also, the patient should
avoid salt substitutes with potassium and AVOID consuming foods high in potassium (like the foods in
option C) because this medication causes the kidneys to retain potassium. The patient should monitor
their blood pressure and pulse rate regularly and report any significant changes to their doctor.

11. A patient is taking an ACE Inhibitor and Spironolactone. It is priority the nurse teaches the
patient?

 A. To avoid consuming alcoholic beverages


 B. To limit foods high in potassium
 C. To limit salt intake
 D. To take the medications with food
The answer is B. ACE Inhibitors and spironolactone (it’s a potassium-sparing diuretic) will both cause
the kidneys to retain potassium. Therefore, it is very important the nurse teaches the patient to limit
foods high in potassium.

. Heparin is an anticoagulant. What family of anticoagulant medications does this drug belong
to?

 A. Direct thrombin inhibitors


 B. Indirect thrombin inhibitors
 C. Vitamin K antagonists
 D. Factor Xa inhibitors
The answer is B. Heparin is an anticoagulant that belongs to the Indirect Thrombin Inhibitors family.

2. Which statement below BEST describes how Heparin works as an anticoagulant?

 A. "It inhibits clotting factors from synthesizing Vitamin K."


 B. "It inactivates the extrinsic pathways of coagulation.”
 C. "It prevents Factor Xa from activating prothrombin to fibrinogen."
 D. "It enhances the activation of antihrombin III, which prevents the activation of
thrombin and the conversion of fibrinogen to fibrin."
The answer is D. Heparin enhances the activation of antihrombin III, which prevents the activation of
thrombin and the conversion of fibrinogen to fibrin. Therefore, this medication INDIRECTLY inhibits
thrombin via antithrombin III.
3. Which patients below would be at a HIGH risk for developing adverse effects of Heparin
drug therapy? Select all that apply:

 A. A 55-year-old male patient who is post-op day 1 from brain surgery.


 B. A 45-year-old female patient with a pulmonary embolism.
 C. A 36-year-old male patient with active peptic ulcer disease.
 D. A 43-year-old female with uncontrolled atrial fibrillation.
The answers are A and C. These patients are both at risk for major bleeding if placed on an
anticoagulant due to their condition (one patient is post-op from brain surgery and the other patient
has ulcers that could bleed). Option B and D are candidates from Heparin therapy because the
patient in option B has a blood clot (Heparin can prevent it from getting bigger and developing new
blood clots), and the patient in option D is at risk for developing a blood clot.

4. Your patient is started on a Heparin drip. You administer a bolus of Heparin and start the
drip per protocol as ordered by the physician. What will be your next important nursing
action?

 A. Collect a PT level in 6 hours per protocol.


 B. Collect an INR level in 4 hours per protocol.
 C. Collect a Troponin level in 6 hours per protocol.
 D. Collect an aPTT level in 6 hours per protocol.
The answer is D. An activated partial thromboplastin time (aPTT) is used to measure clotting time in
patients who are on Heparin. It is important that the nurse collect an aPTT in 6 hours (some protocols
may say 4 hours) after starting the drip. PT and INR are used to measure clotting times in patients
who are taking Warfarin (Coumadin). Troponin levels are used in cardiac patients to detect a
myocardial infarction.

5. A patient is ordered to start an IV continuous Heparin drip. Prior to starting the medication,
the nurse would ensure what information is gathered correctly before initiating the drip?

 A. Vital signs
 B. Weight
 C. PT/INR level
 D. EKG
The answer is B. The nurse would want to make sure the documented weight of the patient is current
and accurate. This medication is weight-based. Therefore, for proper dosing to be administered, a
correct weight should be used.
6. A patient is receiving continuous IV Heparin. In order for this medication to have a
therapeutic effect on the patient, the aPTT should be?

 A. 0.5-2.5 times the normal value range


 B. 2-3 times the normal value range
 C. 1.5-2.5 times the normal value range
 D. 1-3.5 times the normal value range
The answer is C. An aPTT should be 1.5-2.5 times the normal value range for Heparin to achieve a
therapeutic effect in a patient to prevent blood clots. If the aPTT is too low, blood clots can form. If the
aPTT is too high, bleeding can occur.

7. What is the approximate NORMAL level range for an activated partial thromboplastin time
(aPTT)?

 A. 20-25 seconds
 B. 2-3 seconds
 C. 30-40 seconds
 D. 60-80 seconds
The answer is C. This is considered a (approximate…varies in labs) normal aPTT level in someone
who is NOT on Heparin.

8. A patient, who is receiving continuous IV Heparin, has an aPTT of 105 seconds. What is
your next nursing action per protocol?

 A. Continue with the infusion because no change is needed based on this aPTT.
 B. Increase the drip rate per protocol because the aPTT is too low.
 C. Re-draw the aPTT STAT.
 D. Hold the infusion for 1 hour and decrease the rate per protocol because the
aPTT is too high.
The answer is D. The aPTT is 105 seconds, which is too high. Any aPTT value greater than 80
seconds places the patient at risk for bleeding. Most Heparin protocols dictate that the nurse would
hold the infusion for 1 hour and to decrease the rate of infusion. If the aPTT is less than 60 seconds,
the dose would need to be increased and a bolus may be needed. aPTT values should be around 60-
80 seconds to achieve a therapeutic response for Heparin.

9. Select all the TRUE statements about the medication Heparin:

 A. Heparin can be used during pregnancy.


 B. Heparin has a short half-life.
 C. Heparin works to affect the intrinsic pathways of clotting.
 D. Heparin can be administered orally, intravenously, or subcutaneously.
The answers are A, B, and C. The option that is wrong is D. Heparin can NOT be administered
orally….only subq or IV.

10. Your patient is being discharged home and will be required to self-administer injectable
Heparin. You are observing the patient administer their scheduled dose of Heparin to confirm
that the patient knows how to do it correctly. What action by the patient requires you to re-
educate them about how to administer Heparin?

 A. The patient injects the needle into the fatty tissue of the abdomen.
 B. The patient injects the needle 1 inch away from the umbilicus.
 C. The patient rotated the injection site from the previous dose of Heparin.
 D. The patient does not massage the injection site after administering the medication.
The answer is B. The patient should inject the needle 2 inches (NOT 1 inch) away from the umbilicus.
All the other options are correct.

11. A patient is on a continuous IV Heparin drip. As the nurse you are monitoring for any
adverse reactions. Select all the signs and symptoms that would indicate this patient is having
an adverse reaction to this medication:

 A. Hematuria
 B. Decreasing platelets
 C. Increased blood glucose
 D. Low hemoglobin and hematocrit
 E. Positive stool guaiac test
The answers are A, B, D and E. Hematuria, low hbg/hct and positive stool guaiac test all indicate the
patient is bleeding. A decrease in platelet level could indicate the patient is developing Heparin-
induced thrombocytopenia, which is also an adverse reaction to Heparin.

12. Your patient on Heparin develops Heparin-Induced Thrombocytopenia (HIT). What signs
and symptoms in the patient confirm this diagnosis? Select all that apply:

 A. Decrease in platelet level


 B. Increase in platelet level
 C. Development of a new thrombus
 D. Increase in hemoglobin level
The answers are A and C. HIT is where the body makes antibodies against Heparin because it’s
binding to platelet factor 4 (a blood protein). This creates antibodies that will bind to the heparin and
PF4 complex, which activate the platelets. Small clots will form (hence new clots or worsening of
clots) and the platelet count falls...hence thrombocytopenia.

13. A patient develops Heparin-Induced Thrombocytopenia (HIT). As the nurse, you would
expect the Heparin to be discontinued and the patient to be started on what other type of
anticoagulant?

 A. Direct thrombin inhibitor


 B. Protamine sulfate
 C. Switched to subcutaneous Heparin injections
 D. Vitamin-K agonist
The answer is A. The Heparin is discontinued and direct thrombin inhibitors can be started like:
Argatroban, Bivalirudin etc.

14. What is the antidote for Heparin?

 A. Protamine sulfate
 B. Vitamin K
 C. Flumazenil
 D. Narcan
The answer is A. Protamine sulfate is the antidote for Heparin.

15. You’re providing care to a patient who has been receiving long-term doses of Heparin.
What finding in this patient demonstrates the patient may be experiencing a complication that
can occur due to long-term use of this drug?

 A. Uncontrolled hypertension
 B. Bone fractures
 C. Hyperkalemia
 D. Raynaud’s Syndrome
The answer is B. Osteoporosis can occur due to long-term, high doses of Heparin. Bone fractures
would indicate this patient is experiencing this complication. Heparin can stimulate osteoclasts and
inhibits osteoblast, which affects the strength of the bones.
1. Warfarin (Coumadin) is an anticoagulant. What family of anticoagulant medications does
this drug belong to?

 A. Direct thrombin inhibitors


 B. Indirect thrombin inhibitors
 C. Vitamin K antagonists
 D. Factor Xa inhibitors
The answer is C. Warfarin (Coumadin) is an anticoagulant that belongs to the Vitamin K antagonists.

2. Which statement below BEST describes how Warfarin (Coumadin) works as an


anticoagulant?

 A. "It inhibits the formation of thrombin and fibrin in the clotting process."
 B. "It increases the production of Factor Xa, which blocks thrombin and decreases the
clotting process."
 C. "It binds to antithrombin and reverses the clotting process."
 D. "It stops the coagulation process by blocking the formation of clotting factors in
the liver that require Vitamin K to form."
The answer is D. Warfarin (Coumadin) stops the coagulation process by blocking the formation of
clotting factors in the liver that require Vitamin K to form. This is why this drug is part of the Vitamin K
antagonist family.

3. You're educating a patient's family member about Warfarin (Coumadin) and how it is used to
treat blood clots. Which statements by the family member require you to re-educate them
about how this medication works? Select all that apply:

 A. "This medication will help dissolve the blood clot."


 B. "This medication will prevent another blood clot from forming."
 C. "This medication will help prevent the blood clot from becoming bigger in size."
 D. "This medication starts working immediately after the first dose."
The answers are A and D. Warfarin (Coumadin) does NOT dissolve blood clots. It prevents blood
clots from forming, and if one is present, it will help prevent it from becoming bigger. If the blood clot
becomes bigger it may break off and travel in blood circulation. This can lead to a pulmonary
embolism, heart attack, or stroke. Warfarin (Coumadin) does NOT start working immediately. It takes
about 3-5 days of scheduled doses to start achieving a therapeutic INR level. It is very common that a
patient will be on Heparin while taking Warfarin until INR levels are therapeutic.

4. Warfarin (Coumadin) affects the formation of certain clotting factors. Select below ALL the
clotting factors this medication affects:
 A. I
 B. V
 C. II
 D. VII
 E. XI
 F. X
 G. IX
The answers are: C, D, F, and G. Warfarin (Coumadin) affects FOUR clotting factors that require
vitamin K to form. These clotting factors are: 2, 7, 9, and 10 (II, VII, IX, X)

5. Which patients below are NOT candidates for Warfarin (Coumadin)? Select all that apply:

 A. A 45-year-old male patient with alcoholism and an active GI ulcer.


 B. A 55-year-old female with a heart valve replacement.
 C. A 36-year-old female with a deep vein thrombosis.
 D. A 52-year-old male who needs a liver transplant.
The answers are: A and D...Option A is NOT a candidate because active ulcer disease places the
patient at risk for bleeding while taking Warfarin (Coumadin). In addition, patients should NOT drink
alcohol while taking Warfarin (Coumadin) because it majorly affects how the drug works. Option D is
NOT a candidate because of the liver disease. Warfarin affects the liver's clotting factors. Option B
and D would benefit because both are at risk for a blood clots (especially option B). While Option C
already has a blood clot, Warfarin (Coumadin) would help prevent it from becoming larger and new
clots from forming.

6. Your patient was started on Warfarin (Coumadin) a week ago for the treatment of a DVT.
Which findings below would indicate an adverse reaction to this medication? Select all that
apply:

 A. Patient reports a severe and sudden headache


 B. Melena
 C. Chvostek's Sign
 D. Hematuria
 E. Coffee ground emesis
 F. Bleeding gums
The answers are A, B, D, E, and F. An adverse reaction would be associated with bleeding. Patients
who take Warfarin should be monitored for signs and symptoms that bleeding is occurring internally.
Example would be: sudden/severe headache (represents intracranial bleed), melena (dark, tarry
stools...represent blood), hematuria (urine reddish or pink), coffee ground emesis (bleeding in the
stomach), or bleeding gums….along with tachycardia, hypotension, nosebleeds, or bruising for no
reason. Chvostek's Sign is associated with a low calcium level (hypocalcemia).

7. What is considered a normal INR level in a person who is NOT taking Warfarin (Coumadin)?

 A. 10-12 seconds
 B. 0.75-1.25
 C. 2-3
 D. 5-7
The answer is B. This is considered a normal INR level in someone who is NOT taking Warfarin
(Coumadin).

8. A patient is prescribed Warfarin (Coumadin) for the treatment of a blood clot. What is the
therapeutic INR range for this medication?

 A. 2-3
 B. 1-3
 C. 4-8
 D. 0.5-2.5
The answer is A. The therapeutic INR range is 2-3. It may be slightly higher if a patient is at a high
risk for clot formation….(ex: up to 4.5)

9. A patient is scheduled to take a dose of Warfarin (Coumadin) at 1800. The patient's INR level
is 6. As the nurse you will?

 A. Administer the dose as scheduled


 B. Hold the dose and notify the physician
 C. Double the dose
 D. Hold today's dose but administer tomorrow's dose as scheduled.
The answer is B. A therapeutic INR level should be 2-3 (it may be up to 4.5 if the patient is at a HIGH
risk for blood clot formation). A level of 6 is HIGH and places the patient at risk for bleeding. The
nurse should hold the dose and notify the physician.

10. TRUE OR FALSE: If a patient's INR level falls too low, there is a risk of clot formation and
the Warfarin (Coumadin) dose needs to be decreased.

 True
 False
The answer is False. If a patient's INR level falls too low, there is a risk of clot formation and the
Warfarin (Coumadin) dose needs to be INCREASED (not decreased).

11. What is the antidote for Warfarin (Coumadin)?

 A. Flumazenil
 B. Narcan
 C. Protamine Sulfate
 D. Vitamin K
The answer is D. Vitamin K is the antidote for Warfarin (Coumadin).

12. A patient is taking Warfarin (Coumadin). What order received from the physician requires
that you ask for an order clarification?

 A. Check PT/INR daily


 B. Administer Prednisone IM daily
 C. Ambulate three times per day
 D. Draw CBC in the morning
The answer is B. IM (intramuscular) injections should be AVOIDED in patients taking Warfarin
(Coumadin) due to the bleeding risk. Other routes should be ordered.

13. You're providing discharge teaching to a patient who will be going home on Warfarin
(Coumadin). Which statements by the patient demonstrate they understood the educational
material provided? Select all that apply:

 A. "I will switch and use a hard bristle toothbrush."


 B. "When I shave I will be sure to use an electric razor."
 C. "I will be sure that I eat a diet rich in spinach, kale, and broccoli."
 D. "I will avoid drinking any alcoholic beverages."
The answers are B and D. A patient should use a SOFT bristle toothbrush rather than a hard bristle to
prevent damaging the gums (this can lead to bleeding). Also, the patient should maintain a normal
diet, but avoid consuming excessive amounts of green leafy vegetables (like spinach, kale, broccoli,
lettuce etc.) because this can alter how the medication works. The patient should AVOID alcoholic
beverages and use electric razors.

14. A patient, who is taking Warfarin (Coumadin), asks what he should do if he misses a dose.
Your response would be:
 A. Double the dose the next day and notify the physician.
 B. Take the dose when remembered, if on the same day. However, if not
remembered on the same day but the next day, skip the missed dose and take the
next dose at the regular scheduled time.
 C. Hold all doses and call the doctor.
 D. Split the dose the next day.
The answer is B. If a dose is missed the patient should take the dose when remembered IF on the
SAME day. However, if not remembered on the SAME day but the next day, the patient should skip
the missed dose and take the next scheduled dose at its regular scheduled time.

1. Digoxin is part of what family of drugs?

 A. Angiotensin-converting enzyme inhibitors


 B. Beta-blockers
 C. Angiotensin II receptor blockers
 D. Cardiac glycosides
The answer is D: cardiac glycosides

2. Digoxin helps the heart pump more efficiently by altering the inotropic, chronotropic, and
dromotropic actions of the heart. Select all the options below that accurately describe these
actions created by Digoxin:

 A. Positive Chronotropic
 B. Positive Inotropic
 C. Negative Inotropic
 D. Negative Dromotropic
 E. Negative Chrontropic
 F. Positive Dromotropic
The answers are: B, D, and E. Digoxin creates a positive inotropic, negative chronotropic, and
negative dromotropic action on the heart. This helps the heart’s contraction to be stronger while it
pumps at a slower rate. Therefore, the heart will empty more efficiently (less back flow of blood) and
this will increase stroke volume, which will increase cardiac output.

3. A patient is taking Digoxin. What medication on the patient’s medication list increases the
patient’s risk of experiencing Digoxin toxicity?

 A. Furosemide
 B. Metformin
 C. Nitroglycerin
 D. Coumadin
The answer is A. Furosemide is a loop-diuretic and this medication wastes potassium. Remember
hypokalemia (low potassium level) increases the risk of a patient developing Digoxin toxicity.
Hypercalcemia and hypomagnesemia also increases Digoxin toxicity.

4. Your patient, who is 55-years-old, is prescribed to take Digoxin. What patient finding
requires that the nurse hold the dose of Digoxin and notify the physician?

 A. Digoxin level of 1.2 ng/mL


 B. Blood glucose 82
 C. Heart rate 61 bpm
 D. Potassium 2.8 mEq/L
The answer is D. The patient’s potassium level is low (<3.5 mEq/L). Remember hypokalemia
increases the chances of Digoxin toxicity developing. The nurse should notify the MD and hold the
ordered dose until further instructions are given by the doctor.

5. What is the therapeutic range of the drug Digoxin?

 A. 2-3.5 ng/mL
 B. 0.1-2 ng/mL
 C. 0.5-2 ng/mL
 D. 3.5-5 ng/mL
The answer is C. A normal Digoxin level should be 0.5 – 2 ng/mL. Any levels greater than 2 ng/mL is
considered toxic.

6. An adult patient is being discharged home on Digoxin. Which statements below verbalized
by the patient demonstrates they understand how to properly take this medication? Select all
that apply:

 A. “I will limit by intake of foods high in potassium.”


 B. “I will not take this medication and notify the physician if my heart rate is less than 70
bpm.”
 C. “I will measure my pulse rate before every dose I take.”
 D. “It is important that I immediately report any vision changes I may experience
while taking this medication.”
The answers are C and D. The patient should always measure their pulse rate before taking each
dose of Digoxin and hold the dose if it is less than 60 bpm (this is for adults). The patient should not
restrict foods high in potassium because this could lead to hypokalemia, which can lead to Digoxin
toxicity. Vision changes should be reported because this could indicate Digoxin toxicity.

7. What EARLY signs and symptoms should the nurse assess for in a patient taking Digoxin
that could indicate toxicity of this drug? Select all that apply:

 A. Dysrhythmias
 B. Anorexia
 C. Drowsiness
 D. Nausea
 E. Vomiting
The answers are B, D, and E. GI-related signs and symptoms are the earliest indications that the
patient may be having Digoxin toxicity. The other signs and symptoms occur later, especially
dysrhythmias.

8. An infant is ordered a scheduled dose of Digoxin. The patient’s apical pulse rate is 78 bpm.
The nurse would:

 A. Administer the dose as ordered


 B. Hold the dose and reassess the apical pulse rate in 1 hour
 C. Skip this dose but administer the next scheduled dose
 D. Hold the dose and notify the physician
The answer is D. The nurse should hold a dose of Digoxin and notify the physician if the INFANT’S
heart rate is less than 90-110 bpm.
1. This antibiotic can have an adverse effect of dizziness, vertigo, and loss of hearing.

A. Chloramphenicol
B. Aminoglycoside
C. Vancomycin
D. Fluoroquinolones

1. Answer: B. Aminoglycosides.

This antibiotic collect in the 8th cranial nerve causing neurotoxicity as manifested by dizziness,
vertigo, and loss of hearing.

2. This anti-infective agent exerts its effect by interfering with steps in protein synthesis.

A. Macrolide
B. Sulfonamides
C. Antimycobacterial
D. Trimethoprim-sulfamethoxazole

2. Answer: A. Macrolide.

Options B, C, and D exert their effect by depriving the invading organisms of the essential
substances they need for cell division or reproduction.

3. A 60 year old male patient on fourth day of his antibiotic therapy complains of dizziness
and nausea. Which is the most appropriate nursing action?

A. Institute safety precaution and raise side rails.


B. Collaborate with the doctor about antibiotic therapy’s dosage and duration.
C. Provide comfort measures.
D. Instruct client to inform you if symptoms get worse.

3. Answer: B. Collaborate with the doctor about antibiotic therapy’s dosage and duration.

All other options are correct but it is best to address the main cause of the patient’s
manifestations. Elderly patients are more sensitive to neurologic and GI adverse effects of drugs.

4. When an anti-infective agent is said to be no longer effective for a patient, the patient
has developed:
A. Immunity
B. Tolerance
C. Non-responsiveness
D. Resistance

4. Answer: D. Resistance

It occurs when the cells of the invading pathogens no longer respond to the drug.

5. A middle-aged woman came to the ER and complains of ringing in the ears, paresthesias
of the extremities, and erythema of the back. She also noticed that she had decreased
urine output. What history of drug intake should the nurse ask?

A. Oral contraceptive pills (OCPs)


B. Antifungals
C. Vancomycin
D. Trimethoprim-sulfamethoxazole

5. Answer: C. Vancomycin.

Its toxicity includes ototoxicity, renal failure, and red man syndrome characterized by erythema
on the neck and back, paresthesia, chills, severe hypotension, and fever.

1. The following are second-generation cephalosporins, except:

A. Cefuroxime
B. Cefoxitin
C. Ciprofloxacin
D. Cefaclor

1. Answer: C. Ciprofloxacin.

It is a fluoroquinolone.

2. A 24 y/o woman was admitted for Shigellosis and is currently receiving tetracycline.
Upon history taking, she admitted to be taking OCPs and is having multiple sexual
partners. What should be included in the nurse’s health teaching?

A. Patient should use condoms.


B. Patient should stop taking OCPs and abstain from any sexual activity for at least a
C. Patient should increase the dose of her OCPs to maximize its effectivity.
D. Patient should see a psychiatrist.

2. Answer: A. Patient should use condoms.

Tetracycline decreases the effectivity of OCPs and so patient should use extra form of
contraception.

3. Which group of antibiotics is not recommended for growing children because they are
associated with damage to developing cartilage?

A. Macrolides
B. Sulfonamides
C. Tetracyclines
D. Fluoroquinolones

3. Answer: D. Fluoroquinolones.

4. A pregnant patient in labor has concurrent intra-abdominal infection. Culture test


result revealed the pathogen as B. fragilis. She is pre-eclamptic and is receiving
intravenous magnesium sulfate supplemented with valproic acid. Which antibiotic is
supposed to be ideal but is contraindicated in this case?

A. Penicillins
B. Third-generation cephalosporins
C. Carbapenems
D. Sulfonamides

4. Answer: C. Carbapenems.

It is indicated for infections caused by B.fragilis. However, this can reduce serum levels of
valproic acid and can increase the risk of seizures in pre-eclamptic patient.

5. Which group of relatively new antibiotics can cause respiratory failure in patients with
myasthenia gravis?

A. Monobactams
B. Ketolides
C. Lipoglycopeptides
D. Lincosamides
5. Answer: B. Ketolides.

6. Drug of choice for patients with meningococcal meningitis

A. Penicillins and penicillinase-resistant antibiotics


B. Sulfonamides
C. Fluoroquinolones
D. Macrolides

6. Answer: A. Penicillins and penicillinase-resistant antibiotics.

Given in high doses, these drugs can treat meningitis.

7. The drug of choice for Legionnaire’s disease

A. Ciprofloxacin
B. Erythromycin
C. Cefixime
D. Azithromycin

7. Answer: B. Erythromycin.

8. Based on the mechanism of action, the nurse would be right to expect that in this
group of antibiotics patient may also have anemia.

A. Tetracyclines
B. Carbapenems
C. Sulfonamides
D. Penicillins

8. Answer: C. Sulfonamides.

This antibiotic inhibit synthesis of folic acid so bacteria would not have materials needed for
their DNA or RNA.

1. A patient receiving topical antifungal complains of blisters in her perineum. Which


is/are a possible explanation(s) for this?

A. Fungal infection is not healing.


B. Patient is allergic to the drug.
C. Both A and B
D. None of the above

1. Answer: C. Both A and B.

Development of rash, especially if accompanied by blisters and irritation signify worsening of


condition and development of hypersensitivity reaction.

2. Which of the following will alert the nurse for possible adverse effect in patients
receiving long-term itraconazole therapy?

A. Central obesity
B. Cataract
C. Thickening of the skin
D. Pathologic U wave

2. Answer: A. Central obesity.

Ketoconazole, fluconazole, and itraconazole block the activity of a sterol in a fungal wall.
Therefore, they may also block the activity of human steroids such as testosterone and cortisol.

3. An eight (8) month old infant is receiving antifungals. Which should be included in the
nurse’s health teaching to the mother?

A. Cover the area of lesions with diaper to prevent additional infection.


B. Make sure area is free from occlusive dressings.
C. Apply more topical cream on draining areas because it is where fungi are most
D. Advise that redness and rashes are negligible side effects of antifungal therapy and should
not be a cause of worry.

3. Answer: B. Make sure area is free from occlusive dressings.

This include tight diapers, which can facilitate systemic absorption and occurrence of drug
adverse effects. Rashes can be a sign of allergy to the drug.

4. Which of these antifungals can be used in pregnant women?

A. Fluconazole
B. Nystatin
C. Ketoconazole
D. Amphotericin B
4. Answer: D. Amphotericin B

5. Which antifungal can be used for patients who are undergoing hematopoietic stem cell
transplant?

A. Itraconazole
B. Echinocandin antifungals
C. Nystatin
D. Gentian violet

5. Answer: B. Echinocandin antifungals.

1. Chagas’ disease is passed to humans by which of the following vector?

A. female Anopheles mosquito


B. housefly
C. tsetse fly
D. dragonfly

1. Answer: B. housefly.

Option A is for malaria while option C is for African sleeping sickness.

2. Which of the following patient statements should alert the nurse for possible high
levels of quinine?

A. “I feel weak, especially after ambulating.”


B. “My throat is sore. I think I’m down with a flu.”
C. “When I went out of my bed, it felt like the room swayed.”
D. “I don’t feel like eating anything. I just want to sleep.”

2. Answer: C. “When I went out of my bed, it felt like the room swayed.”

This might be vertigo which is part of the constellation of manifestations of cinchonism which is
associated with high levels of quinine or primaquine.

3. A traveler diagnosed with malaria is also receiving sulfonamide for a respiratory


infection. What should the nurse watch out for in this drug combination?

A. increased intracranial pressure


B. urinary retention
C. blood component levels
D. serum BUN and creatinine

3. Answer: C. blood component levels.

This combination can lead to bone marrow suppression.

4. How many days should the patient avoid alcohol after treatment with metronidazole?

A. 1 day
B. 3 days
C. 5 days
D. 7 days

4. Answer: B. 3 days.

5. Disulfiram and tinidazole therapy can lead to increased psychotic reactions. How long
after tinidazole therapy can we safely start disulfiram therapy?

A. 7-10 days
B. 14 days
C. 3 days
D. One month

5. Answer. B. 14 days.

1. A 15 y/o patient came in for a severe case of respiratory flu and would need drug
therapy. Which of the following drugs should the nurse anticipate to be prescribed?

A. acyclovir (Zovirax)
B. amantadine (Symmetrel)
C. abacavir (Ziagen)
D. ganciclovir (Cytovene)

1. Answer: B. amantadine (Symmetrel).

 It is indicated for respiratory viruses.


 Options A and D are for herpes viruses and CMV
 Option C is an NRTI and is indicated for HIV and AIDS.
2. Which of the following antiviral agents is effective as treatment for H1N1?
A. zidovudine
B. acyclovir
C. oseltamivir
D. Selzentry

2. Answer: C. oseltamivir. It is the only antiviral agent effective for H1N1 and avian flu.

 Option A is the only antiviral agent allowed for use in pregnant women.
 Option B is the drug of choice for herpes and CMV infections in children.
 Lastly, option D is a brand name of maraviroc, a CCR5 coreceptor antagonist of HIV
virus.
3. A student nurse administering acyclovir was asked by the clinical instructor on how the
drug works. The student nurse would be correct by stating that this drug works in which
way?

A. unknown; but it is believed to be shedding the protein coat of the virus.


B. it competes with viral receptors found in the host cells
C. it takes away the necessary substances needed by viruses to form DNA chains
D. trapping the viruses and disintegrating them directly

3. Answer: C. it takes away the necessary substances needed by viruses to form DNA
chains.

Agents for CMV and herpes viruses are known to compete with the substrates needed by viruses
for this process leading to formation of shorter and noneffective DNA chains. Option A is the
mechanism of action of agents for influenza and respiratory viruses.

4. Which of the following prescribed antiviral should the nurse question for a patient with
severe hypokalemia?

A. Foscarnet
B. Cidofovir
C. Valganciclovir
D. Nevirapine

4. Answer: A. Foscarnet.

It can cause seizures in patients with electrolyte disturbances.

5. Which enzyme is essential for the maturation of infectious viruses?


A. reverse transcriptase
B. protease
C. nucleosidase
D. CCR5-ase

5. Answer: B. protease.

This is where the mechanism of action of protease inhibitors is anchored. As a result, viruses are
unable to fuse and inject into a cell.

6. A senior nurse would be correct to advise her colleague to monitor which parameter in
patients receiving cyclosporine and zidovudine at the same time?

A. skin lesion, temperature, and color


B. flu-like symptoms
C. level of consciousness
D. CBC with differential count

6. Answer: C. level of consciousness.

Severe drowsiness and lethargy can occur in patients taking these drugs at the same time.

7. The only protease inhibitor that is not teratogenic.

A. darunavir
B. indinavir
C. fosamprenavir
D. saquinavir

7. Answer: D. saquinavir.

However, this drug can cross into breast milk.

8. Anti-hepatitis B agent’s most severe adverse effect is ___________.

A. bone marrow suppression


B. steatosis
C. lactic acidosis
D. severe skin reactions

8. Answer: C. lactic acidosis.


This is accompanied by renal impairment. Option A and D are for anti-hepatitis C agents.

1. Which drug is indicated for threadworm infections?

A. albendazole
B. ivermectin
C. pyrantel
D. mebendazole

1. Answer: B. ivermectin.

 Option A is for tapeworms


 C for pinworms and roundworms, and
 D for whipworms and hookworms as well as pinworms and roundworms.
2. A nurse caring for a pediatric patient on anthelmintic therapy should be alarmed of the
following adverse drug reactions, except:

A. fever and chills


B. weight loss of 2kg in 3 days
C. difficulty in feeding
D. unusually sleepy child

2. Answer. A. fever and chills.

It might be associated with drug action on worms. Options B,C, and D are manifestations of GI
and CNS drug adverse effects.

3. Which of the following physical assessment findings will alert the nurse for
anthelmintic drug toxicity in elderly patients?

A. muscle strength +2
B. blunt liver edge upon palpation
C. 10 bowel sounds in one minute on right lower quadrant (RLQ) upon auscultation
D. non-palpable spleen

3. Answer: B. blunt liver edge upon palpation.

This may indicate hepatomegaly or ongoing inflammatory process which is adversely caused by
anthelmintics. All other options are normal PE findings.
4. A patient was receiving dexamethasone was prescribed with albendazole. Which of the
following is the most appropriate nursing action?

A. Ignore the order and refuse to administer the drug.


B. Increase the dose of albendazole and notify doctor.
C. Discuss with the doctor for possible dose adjustment or change of medication.
D. Decrease the dose of albendazole and notify doctor.

4. Answer: C. Discuss with the doctor for possible adjustment or change of medication.

This combination greatly increases the risk of renal failure.

5. All of the laboratory tests listed below should be arranged by the nurse for patients
receiving anthelmintics, except:

A. serum electrolytes
B. T3 and T4 levels
C. prothrombin time
D. stool culture

5. Answer: B. T3 and T4 levels.

 This is not related to monitoring of client’s condition while under therapy.


 Serum electrolytes is important as client may have fluid losses as side effect of the
drug.
 Prothrombin time is part of liver function test.
 Stool culture is important for establishing appropriate treatment.

1. All of the following statements are not true about antineoplastic agents, except:

A. These drugs assume the role of the innate defenses of humans in destroying rapidly-dividing
cells.
B. Normal cells are always spared from the effects of antineoplastic agents.
C. A and B
D. None of the above

1. Answer: D. None of the above.

Drugs boost the immune system in its efforts to combat the abnormal cells and do not
necessarily assume its roles. In more traditional antineoplastic chemotherapy, normal cells are
also affected. Hence, the severe adverse effects on bone marrow, GIT, and other rapidly-dividing
cells.

2. A student nurse was asked to give an example of an antimetabolite. He would be


correct to state which drug as an example?

A. mercaptopurine
B. cisplatin
C. dactinomycin
D. vincristine

2. Answer: A. mercaptopurine.

 Option B is an alkylating agent.


 Option C is an antineoplastic antibiotic.
 Option D is a mitotic inhibitor.
3. The goal of utilization of drugs as one component of chemotherapy is:

A. to disintegrate the tumors completely


B. to decrease the size of the neoplasm
C. to stop the genetic mutations responsible for cancers
D. to prevent metastasis of neoplasms

3. Answer. B. To decrease the size of the neoplasm.

This is to the point in which the human immune system can deal with it.

4. Which patient complaint would be the least of concern of a nurse taking care of
an elderly on antineoplastic agents?

A. RUQ pain
B. facial puffiness
C. keloid formation on previous IV site
D. oliguria

4. Answer: C. keloid formation on previous IV site.

Options A, B, and D, may be a sign of hepatic and renal toxicity.

5. At which stage of cell cycle is alkylating agent specific?


A. G1 phase
B. S phase
C. M phase
D. None. It is non-specific.

5. Answer: D. None. It is non-specific.

Antimetabolites are S phase-specific. Mitotic inhibitors alter the M phase of the cell cycle.

6. Antineoplastic antibiotics exert their therapeutic action through which mechanism?

A. Causing a mutant DNA molecule


B. Inhibiting DNA polymerase
C. Preventing the cells to divide.
D. Works with hormones to stop stimulating growth of tumors

6. Answer: A. Causing a mutant DNA molecule.

They do this by inserting themselves between base pairs in the DNA chain. Option B is for
antimetabolites. Option C is the work of mitotic inhibitors. Option D is for hormones and
hormone modulators.

7. As part of pre-operative preparation, a patient showing signs of problems with blood


clotting should be asked about usage of which herb?

A. Saw palmetto
B. Echinacea
C. St. John’s wort
D. Ginkgo

7. Answer: D. Ginkgo.

It inhibits blood clotting and can cause problems after surgery. Option A can increase the effects
of various estrogen hormones. Option B can increase the risk for hepatotoxicity. Option C can
increase photosensitivity.

8. How soon after therapy should blood tests be done to monitor bone marrow function?

A. At least a week after


B. 2 weeks after
C. 3 weeks after
D. 5 days

8. Answer: C. 3 weeks after.

9. Which drug has been approved for use in treating CML?

A. Bortezomib
B. Imatinib
C. Pazopanib
D. Erlotinib

9. Answer: B. Imatinib

10. Which is an absolute contraindication in patients receiving nilotinib, a cancer cell-


specific agent?

A. hypomagnesemia
B. short QT syndrome
C. hypercalcemia
D. intake of digitalis

10. Answer: A. hypomagnesemia.

Any condition that can prolong QT interval is a contraindication to the use of nilotinib. All other
options can shorten QT interval.

1. This is a substance released from cells in response to cytotoxic T-cell activity.

A. Interferons
B. Interleukins
C. Colony-stimulating factor
D. Monoclonal antibody

1. Answer: A. Interferons

Interferons are substances naturally-produced and released by human cells that have been
invaded by the viruses. It can be released from cells in response to other stimuli, such as
cytotoxic T-cell activity.

2. How do interleukins exert its therapeutic benefits inside the body?


A. By preventing viral replication inside cells
B. By inhibiting tumor growth and replication
C. By increasing the number of natural killer cells and lymphocytes
D. By producing antiviral proteins

2. Answer: C. By increasing the number of natural killer cells and lymphocytes

All other options are actions of interferons.

3. All of the following are not true about immune modulators, except:

A. Most can be safely recommended in children.


B. Infection control is an integral part of drug therapy.
C. Interferons and interleukins have many drug-to-drug interactions.
D. None of the above

3. Answer: B. Infection control is an integral part of drug therapy.

Patients on immune modulator therapy should be protected from injury and infection. Regular
follow-up and medical care should be stressed.

4. All of the following complaints will alert the nurse of colony stimulating factor adverse
effect, except:

A. alopecia
B. generalized weakness
C. coughing
D. vomiting

4. Answer: C. coughing

Other adverse effects include bone pain, GI upset, dermatitis, and fatigue.

5. Which immune stimulant is usually used for treatment of blood-related cancers?

A. interferons
B. interleukins
C. colony-stimulating factors
D. all of the above

5. Answer: C. colony-stimulating factors


Interferons are usually indicated for hepatitis C, Kaposi sarcoma, and AIDS-related complex.
Interleukins are usually for renal carcinomas and severe thrombocytopenia.

1. Nurse Kate is taking care of patients taking ibuprofen. Which of the following should be
included in her assessment and monitoring?

A. Blood pressure and bowel sounds


B. Weight and appetite
C. Muscle strength and range of motion
D. Respiratory rate, depth, and rhythm

1. Answer: A. Blood pressure and bowel sounds

NSAIDs are associated with CV (hypertension) and GI (constipation, diarrhea, etc.) adverse
effects. All other options are not related.

2. Upon checking the medication chart, Nurse Mike found out that his patient is taking
both acetaminophen and furosemide. Which of the following interventions is an
appropriate nursing action for these two drugs?

A. Administer medications as they are because they enhance drug actions.


B. Measure patient’s intake and output closely.
C. Arrange for SGPT monitoring.
D. Assess for signs and symptoms of bleeding.

2. Answer: B. Measure patient’s intake and output closely.

Acetaminophen can decrease the diuretic effect of loop diuretics so it is important to monitor
patient for fluid retention.

3. Which of the following patient manifests acetaminophen toxicity?

A. Marie, 26, with new-onset jaundice while on drug therapy


B. Eva, 54, with difficulty of breathing while on drug therapy
C. Steven, 37, with joint stiffness while on drug therapy
D. Robert, 29, with new-onset glaucoma while on drug therapy

3. Answer: A. Marie, 26, with new-onset jaundice while on drug therapy

Acetaminophen toxicity manifests as severe hepatotoxicity.


4. Which enzyme is found in active sites of trauma or injury?

A. Streptokinase
B. COX-1
C. alpha-ketoglutarate
D. COX-2

4. Answer: D. COX-2

The presence of this enzyme in active sites of trauma prompts increase in prostaglandins.
NSAIDs block COX-2 before signs and symptoms of inflammation can develop.

5. The patient asked the nurse of how soon pain relief will be achieved after a dose of
intravenous NSAIDs. The nurse would be correct by answering:

A. 5-10 minutes depending on the site involved


B. at the start of infusion
C. 30 minutes since pain is usually extensive
D. at the first hour of infusion

5. Answer: B. at the start of infusion

Onset of action of IV NSAIDs is immediately after infusion. In oral form, onset is 30 minutes after
administration.

6. Which of the following NSAID pairing is correct?

A. propionic acid: indomethacin


B. acetic acid: ibuprofen
C. oxicam derivatives: naproxen
D. fenamates: mefenamic acid

6. Answer: D. fenamates: mefenamic acid

Ibuprofen is a propionic acid and indomethacin is an acetic acid.

1. All of the following are true about salicylates, except:

A. It can be purchased without prescription.


B. Taking salicylates can significantly improve nonspecific signs and symptoms and help pinpoint
the exact diagnosis.
C. Salicylates block the release of chemicals released by active white blood cells that cause
increase in temperature.
D. Aspirin inhibits synthesis of thromboxane A2 to reduce platelet aggregation.

1. Answer: B. Taking salicylates can significantly improve nonspecific signs and symptoms
and help pinpoint the exact diagnosis.

Since salicylates are available OTC, it is prone to be overdosed. This can mask the signs and
symptoms and can possibly contribute to misdiagnosis.

2. In a medical mission, student nurse Patricia was asked which salicylates are safe for
children. After careful assessment of the patient, she would be right to name which
drug(s) as safe to recommend for children?

A. aspirin
B. mesalamine
C. choline magnesium trisalicylate
D. A and C only

2. Answer: D. A and C only

Aspirin and choline magnesium trisalicylate are the only two salicylates recommended for use in
children.

3. Which of the following patient statements will alert the nurse of possible
contraindication to aspirin?

A. “I have a bad case of asthma. My last attack was four weeks ago.”
B. “My doctor advised me to lessen my consumption of red meat but I can’t seem to follow that.”
C. “As a child, I never get to enjoy fruit loops and colored chips because I develop rashes when I
eat them.”
D. “I had a piece of my intestines last year.”

3. Answer: C. “As a child, I never get to enjoy fruit loops and colored chips because I
develop rashes when I eat them.”

Allergies to fruit loops and colored chips may indicate allergy to tartrazine, a dye with cross-
sensitivity to aspirin. Options A and B are not contraindications to aspirin. Option D may be
correct but only if it happened within a week.

4. At which adult dosage can acute salicylate toxicity start to manifest?


A. 10 grams
B. 15 grams
C. 22 grams
D. 50 grams

4. Answer: C. C. 22 grams

At 20-25 g in adults and 4 g in children.

5. All of the following should be monitored regularly to detect adverse effects of


salicylates, except:

A. cardiac markers
B. prothrombin time
C. SGPT level
D. urinalysis

5. Answer: A. cardiac markers

Adverse effects of salicylates include bleeding and hepatic and renal dysfunction which will
reflect on abnormal levels of B, C, and D. Cardiac markers are irrelevant.

1. How can antiarthritic drugs suppress some arthritis?

A. by suppressing autoantibodies produced by the body


B. by inhibiting phagocytosis and release of lysosomal enzymes
C. by coating the inflamed site and promoting healing
D. all of the above

1. Answer: B. These actions inhibit tissue destruction.

2. Which antiarthritic is an oral agent for long-term therapy of rheumatic disorders?

A. anakinra
B. allopurinol
C. auranofin
D. gold sodium thiomalate

2. Answer: C. auranofin
3. Gold compounds can be very toxic. This should be avoided in all of the following
patients, except:

A. Asthma
B. Congestive heart failure
C. Severe diabetes
D. A and C only

3. Answer: A. Asthma

Options B and C are contraindicated for use of gold compounds.

4. Which of the following is/are true about gold compounds?

A. This is the first line agent for the treatment of arthritis.


B. Gold compounds can cause interstitial pneumonitis.
C. It is safe for geriatric patients.
D. All of the above

4. Answer: B. Gold compounds can cause interstitial pneumonitis.

Two respiratory adverse effects of gold compounds are gold bronchitis and interstitial
pneumonitis. It can be particularly toxic to geriatric patients and can lead to GI, renal, and
hepatic adverse effects. This is only indicated for patients unresponsive to conventional therapy.

5. This is an injected drug for early treatment of rheumatic disorders.

A. anakinra
B. allopurinol
C. auranofin
D. gold sodium thiomalate

5. Answer: D. gold sodium thiomalate

1. Glucocorticoids are hormones that:

A. are released in response to high glucose levels


B. help to regulate water balance in the body
C. help to regulate electrolyte levels
D. promote the preservation of energy through increased glucose levels, protein breakdown,
and fat formation
1. Answer: D. promote the preservation of energy through increased glucose levels,
protein breakdown, and fat formation

Glucocorticoids are agents that stimulate an increase in glucose levels for energy. They also
increase the rate of protein breakdown and decrease the rate of protein formation from amino
acids to preserve energy. They are also capable of lipogenesis, or the formation and storage of
fat in the body for energy source.

2. Glucocorticoids can reduce inflammation and suppress the immune system by:

A. forming complex reactions needed to reduce inflammation


B. activating more lymphocytes to reduce inflammation
C. inhibiting the localization of phagocytes so immune system can rest
D. all of the above

2. Answer: A. forming complex reactions needed to reduce inflammation

They bind to cytoplasmic receptors of target cells to form complex reactions needed to reduce
inflammation and suppress the immune system. They also limit the activity of lymphocytes to
act within the immune system. Lastly, they inhibit the spread of phagocytes to the bloodstream
and injured tissues.

3. A patient on aldosterone can eat all of the following meals, except:

A. egg-and-vegetable breakfast crostini


B. bagels, salad, and cottage cheese
C. dried peas and beans
D. white bean and roasted red bell pepper hummus

3. Answer: B. bagels, salad, and cottage cheese

All of the following are low in sodium except bagels, salad (dressing is high in sodium), and
cottage cheese. High-sodium intake should be avoided to prevent severe hypernatremia.

4. A patient is started on a regimen of prednisone because of a crisis in her ulcerative


colitis. Nursing care of this patient would need to include:

A. immunizations to prevent infections


B. increased calories to deal with metabolic changes
C. fluid restriction to decrease water retention
D. administration of the drug around 8 or 9 AM to mimic normal diurnal rhythm.
4. Answer: D. administration of the drug around 8 or 9 AM to mimic normal diurnal
rhythm.

Administer drug daily at 8 to 9 AM to mimic normal peak diurnal concentration levels and
thereby minimize suppression of the hypothalamic-pituitary axis (HPA).

5. Which of the following patient complaint(s) will alert the nurse for possible aldosterone
toxicity?

A. muscle weakness and abdominal fullness


B. BP of 80/60 mmHg
C. parched skin and dry mucous membranes
D. all of the above

5. Answer: A. muscle weakness and abdominal fullness

Signs of toxicity include hypokalemia, weight gain, edema, and hypertension. Muscle weakness
and abdominal fullness can be signs and symptoms of hypokalemia.

1. A nurse is giving teachings to a client receiving Desloratadine (Clarinex). Which of the


following statements made by the client will need further instructions?

A. “I can eat gum after I drink the medicine”


B. “I can take the medicine on an empty stomach“
C. “I should avoid using alcohol”
D. “I will avoid driving while using this medication”

1. Answer: B. “I can take the medicine on an empty stomach”.

 Option B: This medicine should be taken with food or milk to minimize gastrointestinal
upset.
 Option A: Use gum or hard candy to minimize dry mouth.
 Options C and D: The medication causes drowsiness so avoid taking alcohol or
engaging in activities which require mental alertness such as driving a car.
2. Nurse Zeke is giving instructions to her client who is taking antihistamine. Which of the
following nurse teachings is appropriate for the client?

A. Expect a relief in 24 hours


B. Be aware that you may have increased saliva
C. Be aware that you may need to take a decongestant
D. Avoid ingesting alcohol
2. Answer: D. Avoid ingesting alcohol.

 Option D: Because alcohol and antihistamines have sedating properties, concurrent


administration of these drugs should be avoided.
 Option A: Not all antihistamines last 24 hours.
 Option B: Dry mouth is a common side effect, not increased salivation.
 Option C: Antihistamines and decongestants are often given together.
3. Andrew has vertigo, which antihistamine is best for his condition?

A. Terfenadine
B. Guaifenesin
C. Meclizine
D. Hydrocodone

3. Answer: C. Meclizine.

 Option C: Meclizine (Antivert) is given for vertigo and motion sickness.


 Option A: Terfenadine is an antihistamine, but meclizine is the standard drug in the
treatment of vertigo.
 Option B and D: Choices C and D are not antihistamines.
4. Raul, a 20-year-old student, used to buy OTC drugs whenever he feels sick. Which of the
following statements best describes the danger of self-medication with over-the-counter
drugs?

A. Clients are not aware of the action of over-the-counter drugs.


B. Clients are not aware of the side effects of over-the-counter drugs.
C. Clients minimize the effects of over-the-counter drugs because they are available without a
prescription.
D. Clients do not realize the effects of over-the-counter drugs.

4. Answer: C. Clients minimize the effects of over-the-counter drugs because they are
available without a prescription.

 Option C: This choice is correct because it includes the other three risks noted in
choices A, B, and D.
5. Which histamine-2 antagonist is associated with the most drug interactions?

A. Prilosec
B. Nizatidine
C. Ranitidine
D. Cimetidine

5. Answer: D. Cimetidine.

Option D: Cimetidine was the first histamine-2 antagonist developed and is associated with the
most toxic drug interactions of the group.

1. Immune modulators have the following therapeutic actions, except:

A. Decreasing immune activity.


B. Blocking the activation of lymphocytes.
C. Increasing the release of various cytokines involved in the inflammatory response.
D. None of the above.

1. Answer: C. Increasing the release of various cytokines involved in the inflammatory


response.

Immune modulators block the release of cytokines.

2. This is the first oral agent for the treatment of relapsing forms of multiple sclerosis.

A. thalidomide
B. fingolimod
C. lenalidomide
D. anakinra

2. Answer: B. fingolimod

3. The only available interleukin receptor antagonist.

A. thalidomide
B. fingolimod
C. lenalidomide
D. anakinra

3. Answer: D. anakinra

4. This drug(s) disable(s) T cells to act as an immune suppressor.


A. T-cell suppressors
B. Monoclonal antibodies
C. Interleukin receptor antagonist
D. A and B only

4. Answer: B. Monoclonal antibodies

This drug reacts as an antibody to human T cells thereby disabling them.

5. It is the first monoclonal antibody for use.

A. azathioprine
B. cyclosporine
C. infliximab
D. muromonab-CD3

5. Answer: D. muromonab-CD3

The nurse has administered the Varicella vaccine to her patient.


It is in
What should the nurse instruct the patient to do should injection site
soreness occur?
A. notify the doctor
B. apply a cold compress and massage the site
C. take tylenol or motrin and apply a warm compress
D. take 2 aspirin and call back if no improvement
dicated for treatment of acute allograft rejection in those receiving heart or liver
transplants.

The HPV vaccine is designed to prevent uterine cancer?


True
False

Pregnancy is a contraindication of the Varicella vaccine.


True
False

The patient you are caring for is being treated with steroids and asks
about getting the Varicella vaccine while in the office. The nurses best
response to this would be...
A. have you ever had the chicken pox?
B. patients on steroids have a decreased immune system and should wait 6
months before getting the vaccine
C. do you have any hypersensitivity to eggs or egg proteins?
D. have you been out of the country in the last 3 months?

Chemotherapy that is used to direclty kill the cancer but also kills healthy
cells is known as...
A. palliative
B. adjunvant
C. primary

You are caring for a patient who is recieving chemotherapy. The patient is
distressed due to her hair falling out and asks you why this is happening
to her. Your best response would be.
A. the cancer is destroying your hair
B. cancer patients are to sick to worry about their hair
C. don't worry about it, you can wear a hat.
D. this is a medication side effect, it will grow back when treatment is
completed.

Chemotherapy has a direct action on bone marrow cell production.


Which of the following are correct?
A. High WBCs
B. High Platelets
C. Low RBCs
D. High RBC's

1. Student nurse Anne is currently on her psychiatric ward rotation and her clinical
instructor engaged her to an oral quiz on psychotherapeutic agents. She would be right to
answer that loxapine, thiothixene, and fluphenazine belong to which psychotherapeutic
agent classification?
A. Typical antipsychotics
B. Atypical antipsychotics
C. Antimanic drugs
D. CNS stimulants

2. All of the following are not true about psychotherapeutic agents, except:

A. These agents are not indicated for behavioral disorders.


B. Psychotherapeutic agents perform their curative functions in patients with psychoses.
C. Antipsychotics address mental disorders mainly through their sedating effects.
D. Psychotherapeutic agents help patients perform activities of daily living with ease.

3. Antipsychotic which has a role in managing side effects of chemotherapy.

A. Risperidone
B. Haloperidol
C. Prochlorperazine
D. Aripiprazole

4. What is the top nursing consideration of a nurse who is taking care of an adult client
with schizophrenia receiving antipsychotics?

A. Monitor urine output.


B. Obtain ECG tracing regularly as ordered.
C. Assess bowel sounds.
D. Provide comfort measures.

5. All of the following are contraindications to neuroleptic agents, except:

A. Children below age 12


B. Dementia
C. Active alcoholism
D. None of the above

6. This drug is the mainstay treatment for mania.

A. quetiapine (Seroquel)
B. olanzapine (Zyprexa)
C. lithium salts (Lithotabs)
D. lamotrigine (Lamictal)
7. Antimanic drugs have effect on the following neurotransmitters, except:

A. Norepinephrine
B. Dopamine
C. Serotonin
D. None of the above

8. Which of the following is the most important patient education for a patient receiving
CNS stimulants?

A. Proper storage of drugs


B. Low-sodium diet
C. Light to moderate exercises
D. All of the above.

Answers and Rationale

1. Answer: A. Typical antipsychotics.

Loxapine (Loxitane), thiothixene (Navane), and fluphenazine (Prolixin) are some of the most
common examples of typical antipsychotics. Typical antipsychotics block dopamine receptors,
preventing dopamine from stimulating the postsynaptic neurons. They are also associated to
neurological adverse effects, which are relieved by atypical antipsychotics.

2. Answer: D. Psychotherapeutic agents help patients perform activities of daily living


with ease.

They are indicated for both perceptual and behavioral disorders. They were once called major
tranquilizers but later on it was changed because sedation is not their primary or main function.
Lastly, they don’t cure psychoses.

3. Answer: C. Prochlorperazine.

It is indicated for severe vomiting and nausea associated with surgery and chemotherapy.

4. Answer: B. Obtain ECG tracing regularly as ordered.

Usage of drugs in this population requires close monitoring for adverse effects. For example,
thioridazine and ziprasidone can cause changes in QT interval.

5. Answer: A. Children below age 12.


Caution is used in children younger than 12 years of age who have chickenpox or a CNS infection
because children are more likely to develop dystonia and this could cause confusion in the
diagnosis of Reye’s syndrome. Use of these agents in patients with dementia increases risk of CV
events and death. Lastly, antipsychotics or neuroleptics can potentiate CNS depression in
patients with active alcoholism.

6. Answer: C. lithium salts (Lithotabs).

All options are antimanic drugs but lithium is the drug of choice for mania.

7. Answer: C. Serotonin.

The primary action is to alter transport of sodium in nerve and muscle cells. They also inhibit the
release of norepinephrine and dopamine, but not serotonin from stimulated neurons.

8. Answer: A. Proper storage of drugs.

Majority of CNS stimulants are controlled substances so it is important for patients to be taught
how to secure them to prevent inappropriate use and distribution.

1. What drug refers to the ability to help patient feel calm and unaware of his
environment?

A. Anesthesia
B. Sedatives
C. Hypnotics
D. Anxiolytics

2. The most commonly used anxiolytics.

A. Benzodiazepines
B. Diphenhydramine
C. Barbiturates
D. Buspirone

3. Which of the following medical condition(s) can be considered as contraindication(s) to


use of anxiolytics and hypnotics?

A. Psychosis
B. Alcoholic intoxication
C. Acute gastroenteritis
D. Both A and B

4. What is the antidote of benzodiazepine?

A. Theophylline
B. Ranitidine
C. Flumazenil
D. Benadryl

5. Abrupt withdrawal of barbiturates can precipitate what medical condition?

A. Ascites
B. Hypertensive crisis
C. Status epilepticus
D. Coma

1. Nortriptyline and protriptyline are classified as what class of TCAs?

A. Amines
B. Secondary amines
C. Tetracyclics
D. None of the above

2. The following are true about depression, except:

A. External causes always play a part in assessment and diagnosis.


B. Patients with depression have trouble sleeping and eating.
C. Depression can lead to multiple physical problems.
D. None of the above.

3. Which TCA is also approved for use in patients with OCD?

A. Imipramine
B. Clomipramine
C. Sertraline
D. Amitriptyline

4. A combination of MAOIs and TCAs will precipitate which drug adverse effect?
A. Severe hypertensive crisis
B. Severe hyperpyretic crisis
C. Severe hypnotic crisis
D. Severe amnesia

5. What is the treatment for hypertensive crisis caused by MAOI?

A. Phentolamine
B. Adenosine
C. Propanolol
D. Nifedipine

6. How many weeks will the full therapeutic effects of SSRIs be realized?

A. 2 weeks
B. 4 weeks
C. 6 weeks
D. 8 weeks

1. Student nurse Anne is currently on her psychiatric ward rotation and her clinical
instructor engaged her to an oral quiz on psychotherapeutic agents. She would be right to
answer that loxapine, thiothixene, and fluphenazine belong to which psychotherapeutic
agent classification?

A. Typical antipsychotics
B. Atypical antipsychotics
C. Antimanic drugs
D. CNS stimulants

2. All of the following are not true about psychotherapeutic agents, except:

A. These agents are not indicated for behavioral disorders.


B. Psychotherapeutic agents perform their curative functions in patients with psychoses.
C. Antipsychotics address mental disorders mainly through their sedating effects.
D. Psychotherapeutic agents help patients perform activities of daily living with ease.

3. Antipsychotic which has a role in managing side effects of chemotherapy.

A. Risperidone
B. Haloperidol
C. Prochlorperazine
D. Aripiprazole
4. What is the top nursing consideration of a nurse who is taking care of an adult client
with schizophrenia receiving antipsychotics?

A. Monitor urine output.


B. Obtain ECG tracing regularly as ordered.
C. Assess bowel sounds.
D. Provide comfort measures.

5. All of the following are contraindications to neuroleptic agents, except:

A. Children below age 12


B. Dementia
C. Active alcoholism
D. None of the above

6. This drug is the mainstay treatment for mania.

A. quetiapine (Seroquel)
B. olanzapine (Zyprexa)
C. lithium salts (Lithotabs)
D. lamotrigine (Lamictal)

7. Antimanic drugs have effect on the following neurotransmitters, except:

A. Norepinephrine
B. Dopamine
C. Serotonin
D. None of the above

8. Which of the following is the most important patient education for a patient receiving
CNS stimulants?

A. Proper storage of drugs


B. Low-sodium diet
C. Light to moderate exercises
D. All of the above.

1. Sensory nerves that respond to stimulation by generating impulses that produce pain
sensations.
A. A-fibers
B. B-fibers
C. C-fibers
D. D-fibers

2. Pain that is caused by nerve injury.

A. Neurotraumatic pain
B. Neuropathic pain
C. Nociceptive pain
D. Psychogenic pain

3. Antidote for narcotic overdose.

A. Naloxone
B. Nubain
C. Morphine
D. Codeine

4. Narcotic agonist of choice for pregnant patients.

A. Meperidine
B. Tramadol
C. Fentanyl
D. Oxycodone

5. What is the treatment for opioid-induced constipation?

A. Psyllium
B. Relistor
C. Ritalin
D. Oxymorphone

1. The functional unit of muscles.

A. Myomere
B. Sarcomere
C. Actin and myosin
D. Neuromuscular junctions

2. Which of the following is a contraindication in the use of nondepolarizing NMJ blockers?


A. Systemic Lupus Erythematosus
B. Gunshot wound
C. Hypokalemia
D. All of the above

3. The only depolarizing NMJ blocker.

A. Succinylcholine
B. Vecuronium
C. Acetylcholine
D. Both A and C

4. This drug should not be combined with depolarizing NMJ blockers they will reverse the
blocking effect of NMJ blockers.

A. Calcium-channel blockers
B. NSAIDs
C. Xanthines
D. Aminoglycosides

5. What should be available in the bedside to overcome the excessive neuromuscular


blockade of nondepolarizing NMJ blockers?

A. Acetylcholine agonists
B. Cholinesterase inhibitors
C. Halothane
D. Isoflurane

1. All of the following are direct-acting skeletal muscle relaxants, except:

A. botulinum toxin type A


B. Myobloc
C. Dantrium
D. None of the above

2. Which of the following is the priority nursing intervention for children receiving muscle
relaxants?
A. Assess bowel sounds and activity.
B. Provide skin care.
C. Administer drug with food.
D. Both A and B.

3. What is the centrally acting muscle relaxant of choice for children with tetanus?

A. botulinum toxin type B


B. Dantrium
C. Baclofen
D. Methocarbamol

4. How long should Baclofen be tapered to prevent development of psychoses and


hallucinations?

A. 12-21 days
B. 1-10 days
C. 7-14 days
D. 7-21 days

5. A patient who had a dose of botulinum toxin type A complained of headache and
dizziness. What is the most appropriate nursing response?

A. Provide comfort measures.


B. Institute safety measures.
C. Assess for possible anaphylactic reactions
D. Both A and B

1. What is the goal of therapy for Parkinson’s disease?

A. To decrease dopamine and to increase cholinergic neurons


B. To balance dopamine and cholinergic neurons
C. To excite neurons more
D. To inhibit neurons more
2. What is the mainstay of treatment for Parkinson’s?

A. Symmetrel
B. levodopa
C. Cogentin
D. Parlodel

3. The drug of choice in children with parkinsonian symptom.

A. levodopa
B. Artane
C. Benadryl
D. benztropine

4. Which can decrease efficacy of levodopa?

A. Phenytoin
B. Pyridoxine
C. Niacin
D. Both A and B

5. A construction worker for 10 years is about to receive anticholinergics. What should the
nurse consider in handling this patient?

A. Do not give the drug to the patient.


B. Administer it with caution.
C. Discuss it with the doctor and have the order changed.
D. Arrange for a possible increase in the dose.

6. Which of the following is a contraindication for the use of anticholinergic agents?

A. Heart rate of 120 beats per minute


B. Myasthenia gravis
C. Active hepatitis B
D. All of the above.

1. The following are true about cholinergic agonists, except:

A. These drugs exert their effect by mobilizing the acetylcholine into their receptors found
throughout the body.
B. Some of their effects include vasodilation, bronchoconstriction, and bladder relaxation.
C. Ophthalmic agents can induce miosis or pupil constriction.
D. Edrophonium is used for diagnosis of myasthenia gravis only.

1. Answer: A. These drugs exert their effect by mobilizing the acetylcholine into their
receptors found throughout the body.

Cholinergic agonists act at the same site as the neurotransmitter acetylcholine (ACh) and
increase the activity of the ACh receptor sites throughout the body. They do not mobilize
acetylcholine into their receptors.

2. All of the following are direct-acting cholinergic agonists, except:

A. Pilocarpine
B. Neostigmine
C. Carbachol
D. Cevimeline

2. Answer: B. Neostigmine

It is an agent for myasthenia gravis and is therefore an indirect-acting cholinergic agonist.

3. This drug is usually indicated for postoperative and postpartum urinary retention as
well as bladder atony.

A. carbachol
B. bethanechol
C. tacrine
D. pyridostigmine

3. Answer: B. bethanechol

This drug has specific affinity for receptors in the urinary bladder.

4. Which of the following is an indication for withholding indirect-acting cholinergic


agonists?

A. Visual acuity of 20/150


B. Blood pressure of 140/90 mmHg
C. GFR of 30 mL/hour
D. Adult heart rate of 55 beats per minute
4. Answer: D. Adult heart rate of 55 beats per minute

It can be exacerbated by the parasympathetic effects of the drug

5. A patient on neostigmine complained of frequent diarrhea and salivation. Which is the


best nursing action for this situation?

A. Educate client about drug therapy to alleviate anxiety as these are expected drug effects.
B. Withhold the dose of the drug and refer to physician.
C. Arrange for increasing the dose of the drug as it is not effective.
D. Document patient response and endorse accordingly for bedside care.

5. Answer: B. Withhold the dose of the drug and refer to physician.

Discontinue drug if excessive salivation, diarrhea, emesis, or frequent urination becomes a


problem to decrease the risk of severe adverse reactions. The nurse should refer this to the
physician.

1. All of the following are true about dopamine and dobutamine, except:

A. They bind to both alpha- and beta-receptors


B. They can increase heart rate and cause bronchoconstriction
C. Dopamine is the sympathomimetic of choice for shock
D. Administration of these drugs can put patients with peripheral cardiovascular disease into
exacerbation

1. Answer: B. They can increase heart rate and cause bronchoconstriction

Dopamine and dobutamine are alpha- and beta-receptors. They can increase the heart rate and
cause bronchodilation as well as increasing the rate and depth of respirations. They can put
patients with peripheral CV disease into exacerbation because of their vasoconstrictive effects.
Lastly, dopamine is the sympathomimetic of choice for shock.

2. The sympathomimetic found in many OTC cold products.

A. salmeterol
B. albuterol
C. phenylephrine
D. terbutaline

2. Answer: C. phenylephrine
Phenylephrine is often found in OTC allergy and cold preparations so primary caregivers should
be instructed to check labels for ingredients and not combine drugs with similar ingredients.

3. Nurse Jake was doing his usual morning routine care for his patient receiving
parenteral isoproterenol when he noticed a developing extravasation in the site. Which is
the best nursing action for this situation?

A. Provide skin care and warm compress because it will subside in 1-2 hours.
B. Explain to the client that it is an expected side effect of the drug.
C. Document and endorse to next nurse on duty because extravasation need long hours of
observation before an appropriate course of nursing actions can be determined.
D. Inject 10 mL of saline with 5 mg of phentolamine to save the area from necrosis.

3. Answer: D. Inject 10 mL of saline with 5 mg of phentolamine to save the area from


necrosis.

The vasoconstrictive effects of isoproterenol can cause necrosis. Management for this is
infiltrating the site with 10 mL saline containing 5-10 mg of phentolamine within 12 hours to
save the area from necrosis.

4. This drug is used to control hypertension and is used as an epidural infusion for
patients suffering from cancer pain.

A. midodrine
B. clonidine
C. albuterol
D. isoproterenol

4. Answer: B. clonidine

Clonidine specifically stimulates alpha2-receptors of the CNS leading to decreased CNS outflow of
norepinephrine. Orally and transdermally, it is used to control hypertension and as an injection,
it is for epidural infusion for controlling cancer pain.

5. A pregnant patient on 32 weeks age of gestation came rushing to the emergency


complex because of increased watery discharge and frequent uterine contractions. Upon
physical examination, patient was confirmed to have ruptured of membranes and is in
preterm labor. Which of the additional PE findings will render beta-agonists
contraindicated as part of her management for uterine relaxation?

A. HbA1c of 6
B. blood pressure of 150/70 mmHg and +3 proteinuria
C. BUN: 10 mg/dL
D. few to many pus cells in urinalysis

5. Answer: B. blood pressure of 150/70 mmHg and +3 proteinuria

This could signify a possibility of eclampsia and this can be complicated by drug effects of beta-
agonists (e.g. uterine relaxation and increased blood pressure).

1. Student Nurse Aimee is reviewing for her licensure exam. She would be right to pick
which drug pairing (classification and corresponding example)?

A. alpha1-selective adrenergic blocking agent: phentolamine


B. nonselective adrenergic blocking agent: labetalol
C. nonselective alpha-adrenergic blocking agent: doxazosin
D. nonselective beta-adrenergic blocking agent: metoprolol

1. Answer: B. nonselective adrenergic blocking agent: labetalol

Other examples of nonselective adrenergic blocking agents aside from labetalol are amiodarone
and carvedilol. Phentolamine is a nonselective alpha blocker. Doxazosin is an alpha1-selective
blocker and metoprolol is a selective beta-blocker.

2. Which of the following drugs can improve urine flow in males?

A. prazosin
B. nebivolol
C. labetalol
D. amiodarone

2. Answer: A. prazosin

Prazosin, a selective alpha1-receptor adrenergic antagonist, can improve urine flow in male
patients and are used as treatment for benign prostatic hypertrophy (BPH). This is because they
can block smooth muscle receptors in the genitourinary tract which leads to relaxation of
prostate and bladder.

3. Nurse Sam is taking care of a patient on carvedilol therapy. Which parameter should he
monitor regularly to assess for possible dose adjustment?

A. serum sodium level


B. hemoglobin and hematocrit
C. SGPT
D. BUN

3. Answer: C. SGPT

Carvedilol has been associated with hepatic failure related to its effects on the liver.

4. Mrs. Smith, a 40-year-old patient, came in due to acute hypertensive episode. Her chart
says she is asthmatic. Which of the following drugs would be safest to give?

A. nebivolol
B. timolol
C. propranolol
D. atenolol

4. Answer: D. atenolol

Atenolol is a selective beta-blocker and it does not block the receptors responsible for
bronchodilation. All other options are nonselective beta-blockers and may cause
bronchoconstriction and spasms.

5. Which of the following drug is used for diagnosis of pheochromocytoma?

A. amiodarone
B. phentolamine
C. tamsulosin
D. acebutolol

5. Answer: B. phentolamine

1. Anticholinergic drugs are used

A. To allow the sympathetic system to dominate


B. To block the parasympathetic system, which is commonly hyperactive
C. As the drugs of choice for treating ulcers
D. To stimulate GI activity

1. Answer: A. To allow the sympathetic system to dominate


It decreases GI activity and secretions in the treatment of ulcers and to decrease other
parasympathetic activities to allow the sympathetic system to dominate. More specific and less
systemically toxic drugs are available for treatment of ulcers.

2. Atropine and scopolamine work by blocking what receptor(s) in the parasympathetic


nervous system?

A. Nicotinic receptors only


B. Muscarinic and nicotinic receptors
C. Muscarinic receptors only
D. Adrenergic receptors to allow cholinergic receptors to dominate

2. Answer: C. Muscarinic receptors only

Atropine and scopolamine work by blocking only the muscarinic effectors in the
parasympathetic nervous system and the few cholinergic receptors in the SNS.

3. Which of the following suggestions would the nurse make to help a patient who is
receiving an anticholinergic agent reduce the risks associated with decreased sweating?

A. Covering the head and using sunscreen


B. Ensuring hydration and temperature control
C. Changing position slowly and protecting from the sun
D. Monitoring for difficulty swallowing and breathing

3. Answer: B. Ensuring hydration and temperature control

This drug blocks sweating, which is the body’s way of cooling off. This places the patient at an
increased risk for heat stroke. Extremes of temperature should be avoided.

4. Which of the following would the nurse be least likely to include when developing a
teaching plan for a patient who is receiving an anticholinergic agent?

A. Encouraging the patient to void before dosing


B. Setting up a bowel program to deal with constipation
C. Encouraging the patient to use sugarless lozenges to combat dry mouth
D. Performing exercises to increase the heart rate

4. Answer: D. Performing exercises to increase the heart rate


Exercises should be avoided because this drug blocks sweating. This placed the patient at
increased risk for heat stroke.

5. Remembering that anticholinergics block the effects of PNS, the nurse would question
an order for an anticholinergic drug for patients with which of the following conditions?

A. Biliary spasms
B. Bradycardia
C. Glaucoma
D. Asthma

5. Answer: C. Glaucoma

All other options are indications for atropine.

1. Antidiuretic hormone (ADH)

A. can be used for treatment of hemophilia B and C


B. is deficient in diabetes insipidus
C. is produced and stored by the posterior lobe of the pituitary gland
D. can decrease the level of clotting factor VIII

1. Answer: B. is deficient in diabetes insipidus

Its deficiency causes patients with diabetes insipidus to produce a large amount of dilute urine.
It is used as a treatment of hemophilia A because it can increase the level of clotting factor VIII. It
is produced by the hypothalamus and stored in the posterior lobe of the pituitary gland.

2. Treatment with ADH preparations is associated with adverse effects, including

A. “hangover symptoms” like headache and sweating


B. constipation and paralytic ileus
C. cholecystitis and bile obstruction
D. nocturia and bedwetting

2. Answer: A.“hangover symptoms” like a headache and sweating

These are symptoms of water intoxication. Other adverse effects include cramps, dry mouth,
vomiting, and local reaction at injection site.

3. Which of the following is true regarding growth hormone (GH) deficiencies?


A. It occurs only in children.
B. It always results in dwarfism.
C. It is treated only in children because GH is usually produced only until puberty.
D. It can occur in adults as well as children.

3. Answer: D. can occur in adults as well as children

GH deficiencies can be caused either by congenital defects (children) and pituitary tumors and
trauma (usually adults), among others.

4. Octreotide should include periodic evaluation of

A. serum potassium
B. gallstone development
C. level of consciousness
D. muscular strength

4. Answer: B. gallstone development

Use of octreotide and lanreotide warrant periodic ultrasound evaluation of gallbladder to detect
any gallstone development.

5. Which of the following drugs can increase the toxicity of bromocriptine?

A. Phenothiazine
B. Opioids
C. Erythromycin
D. Statins

5. Answer: C. Erythromycin

Combining erythromycin and bromocriptine will increase the risk of toxicity. Phenothiazines
decrease the effectiveness of bromocriptine. Lastly, opioids will warrant an increase in dosage of
GH antagonists.

1. Hyperparathyroidism

A. Leads to low serum level of calcium has a primary form which is more common in elderly men
B. Has a primary form which is more common in elderly men
C. Benefits from antihypercalcemic agents
D. Secondary form is common in patients with chronic liver failure
1. Answer: C. Benefits from antihypercalcemic agents

Hyperparathyroidism is a condition characterized by excessive production of PTH leading to


elevated calcium level (hypercalcemia). Therefore, it will benefit from antihypercalcemic agents.
Primary hyperparathyroidism occurs more often in women between 60-70 years of age.
Secondary hyperparathyroidism occurs most frequently in patients with chronic renal failure.

2. A patient taking calcitriol is also prescribed with a magnesium-containing antacid.


Which of the following should be included in the nurse’s monitoring?

A. Color of stool
B. Signs of bleeding
C. Liver function test
D. Serum electrolyte level

2. Answer: D. Serum electrolyte level

Antihypocalcemic agents combined with magnesium-containing antacids increase the risk for
hypermagnesemia. Therefore, it is important to monitor serum magnesium level.

3. Which of the following should be included in patient teaching of patients taking


bisphosphonates?

A. Stay upright 30 minutes after taking the drug.


B. Take drug on empty stomach.
C. Chew drug instead of swallowing.
D. Both B and C

3. Answer: A. Stay upright 30 minutes after taking the drug.

Drug should be taken with a full glass of water and patient should stay upright for 30 minutes
because serious esophageal erosion can occur.

4. Common adverse effects of calcitonin include:

A. Headache
B. Flushing of the face and hands
C. Abdominal tenderness
D. Diarrhea

4. Answer: B. Flushing of the face and hands


Calcitonin is available in IM, subcutaneous, and nasal forms. Other adverse effects include skin
rash, nausea and vomiting, urinary frequency, and local inflammation at the site of injection.
Headache and diarrhea are common in bisphosphonates.

5. This is a genetically-linked condition with parathyroid dysfunction characterized by


deep bone pain, headaches, and hearing loss.

A. Tay-Sachs disease
B. Paget’s disease
C. Gaucher disorder
D. Marfan Syndrome

5. Answer: B. Paget’s disease

Paget’s disease is a genetically linked disorder characterized by overactive osteoclasts that are
eventually replaced by enlarged and softened bony structures. Manifestations include deep
bone pain, headaches, and hearing loss.

1. This is the most common type of thyroid dysfunction.

A. Cretinism
B. Myxedema coma
C. Hyperthyroidism
D. Hypothyroidism

1. Answer: D. Hypothyroidism

Hypothyroidism is a lack of sufficient levels of thyroid hormones to maintain a normal


metabolism. This is the most common type of thyroid dysfunction and is common among older
women and men. Symptoms include obesity and fatigue, among others.

2. Thyroid hormones can:

A. stimulate overproduction of TSH


B. increase oxygen consumption
C. increase the heart rate without affecting the respirations
D. retard growth and maturation

2. Answer: B. increase oxygen consumption


Thyroid hormones increase the metabolic rate of body tissues, increasing oxygen consumption,
respiration, heart rate, growth and maturation, and the metabolism of fats, carbohydrates, and
proteins.

3. All of the following should be included in the nursing interventions for patients taking
thyroid hormones, except:

A. consistent administration before breakfast each day


B. monitor cardiac response
C. advise patient to frequently exercise to develop the core
D. provide for temperature control and safety precautions

3. Answer: C. advise patient to frequently exercise to develop the core

As much as possible, patients should be provided for rest periods when on this therapy.

4. The antithyroid agent of choice for pregnant women.

A. methimazole
B. PTU
C. levothyroxine
D. iodine solution

4. Answer: B. PTU

Other antithyroid agents can lead to development of cretinism.

5. Which of the following history data will possibly render strong iodine products
contraindicated for the patient?

A. 35 years old
B. history of blood-tinged sputum for the past three weeks
C. intake of oral anticoagulants
D. both B and C

5. Answer: D. both B and C

A blood-tinged sputum for the past three weeks may signify tuberculosis. If this is the case,
strong iodine products are contraindicated. Also, strong iodine products can change the
metabolism and level of anticoagulants and may place the patient at greater risk for bleeding.
Lastly, I131 is only for patients over 30 years old because of adverse effects associated with
radioactivity.

1. Severe hypoglycemia is set to occur and would need a dose of glucose-elevating agents
at which blood glucose level?

A. 30 mg/dL
B. 65 mg/dL
C. 50 mg/dL
D. 75 mg/dL

1. Answer: A. 30 mg/dL

Glucose-elevating agents raise blood level of glucose when severe hypoglycemia occurs at <40
mg/dL.

2. Glucose-elevating agents exert their actions by:

A. Delaying the excretion of glucose


B. Decreasing insulin release
C. Delaying the breakdown of glycogen in the liver to release more glucose
D. Transforming into synthetic glucose molecules that the body can use in times of severe stress

2. Answer: B. Decreasing insulin release

The desired and beneficial action of glucose-elevating agents is increasing blood glucose by
decreasing insulin release and accelerating the breakdown of glycogen in the liver to release
glucose.

3. Glucagon is commonly associated with which adverse effects?

A. GI upset
B. Arrhythmia
C. Skin rashes
D. Hypokalemia

3. Answer: A. GI upset

Glucagon is associated with GI upset, nausea, and vomiting.

4. Diazoxide is commonly associated with which adverse effects?


A. Hypotension and headache
B. Myalgia and arthralgia
C. Jaundice
D. Oliguria

4. Answer: A. Hypotension and headache

Diazoxide is associated with vascular effects, including hypotension, headache, cerebral


ischemia, weakness, heart failure, and arrhythmias. This is because diazoxide has the ability to
relax arteriolar smooth muscle.

5. Which of the following drugs is to be avoided in patients taking diazoxide to prevent


toxicity?

A. ACE inhibitors
B. Thiazide diuretics
C. Phenothiazines
D. Antacids

5. Answer: B. Thiazide diuretics

Diazoxide with thiazide diuretics can increase risk of toxicity because these two are structurally
the same.

1. Which of the following statements are true about insulin?

A. It can be taken orally or injected subcutaneously to control high glucose level in the blood
B. It is an endogenous replacement for low insulin levels in patients with diabetes mellitus
C. It acts on various insulin receptor sites to facilitate entry of glucose into the cells
D. It should be avoided in pregnant patients with diabetes

1. Answer: C. Acts on various insulin receptor sites to facilitate entry of glucose into the
cells

It is the insulin’ mechanism of action to control hyperglycemia. It is in parenteral form only and is
an exogenous replacement for low insulin levels. Lastly, insulin is the treatment of choice in
pregnant patients with diabetes.

2. Andrea, a 15-year-old patient, was newly diagnosed with diabetes type 1. She is on
regular insulin. If she’s prescribed to have a daily shot of insulin every 8:30 am, when
would be the appropriate time that she should have her meals?
A. 9:00 AM
B. 11:00 AM
C. 12:00 NN
D. 1:00 PM

2. Answer: A. 9:00 AM.

The onset of regular insulin is 30-60 minutes. Andrea should be able to eat by that time to avoid
hypoglycemia.

3. Which of the following herbal therapies may increase the risk of hypoglycemia in
patients prescribed with insulin?

A. Aloe vera
B. Ginseng
C. Papaya
D. All of the above

3. Answer: B. Ginseng

Others include juniper berries, garlic, fenugreek, coriander, dandelion root, celery.

4. All of the following nursing interventions are needed for patients taking insulin, except:

A. Administering refrigerated pre-drawn syringes as old as 1 week.


B. Administering mixtures of insulins within 15 minutes after combining them.
C. Rotating injection sites.
D. Shaking the vial to ensure uniform suspension.

4. Answer: D. Shaking the vial to ensure uniform suspension

Gently rotating the vial instead of shaking it will ensure uniform suspension. Options A and B are
correct. Rotation of injection sites will prevent lipodystrophy.

5. A diabetic patient receiving insulin is in need of an antihypertensive. Which of the


following would be the least recommended in the following choices?

A. Propranolol
B. Enalapril
C. Hydrochlorothiazide
D. Nifedipine
5. Answer: A. Propranolol

Propranolol is a beta-blocker and it can also block the signs and symptoms of hypoglycemia.
Careful assessment is needed if this is to be given as well as other methods to identify
hypoglycemia.

1. All of the following are true about sulfonylureas, except:

A. It is indicated for all diabetic patients


B. First-generation sulfonylureas have shorter duration of action than second-generation
sulfonylureas.
C. They can increase the number of insulin receptors
D. Some patients develop resistance over time

1. Answer: A. It is indicated for all diabetic patients

It is only indicated for patients with functioning beta cells. Therefore, not all diabetic patients are
candidate for this drug. Patients with type 1 diabetes do not have functioning beta cells and
would have no benefit from the drug.

2. Nurse Janine is taking care of a diabetic patient for the first time. The patient is on
sulfonylureas and was asking about its role in managing diabetes. Which of the following
is the correct response?

A. Antidiabetic drugs is the backbone of antidiabetic therapy.


B. It can help cure diabetes together with proper diet and exercise.
C. Antidiabetic drugs are adjuncts to help control blood glucose levels.
D. They work by slowing release of insulin by beta cells, decreasing insulin receptor site
sensitivity, and allowing liver release of glucose.

2. Answer: C. Antidiabetic drugs are adjuncts to help control blood glucose levels.

Proper diet and exercise are the backbone of antidiabetic therapy; antidiabetic drugs are
adjuncts to help control blood glucose levels.

3. Which of the following is the most common adverse effect of taking sulfonylureas?

A. GI distress
B. Hypoglycemia
C. Skin reactions
D. Heartburn

3. Answer: B. Hypoglycemia

Hypoglycemia is the most common adverse effect of taking sulfonylureas. Other common
adverse effects include skin reactions and GI distress.

4. The patient taking glipizide is about to be discharged. Which of the following patient
statements indicate that there is a need for additional health teaching?

A. “I can take drugs and skip meals so I can lose excess sugar fast.”
B. “I still need to have exercise because this drug I’m taking is not the ultimate remedy to my
health problem.”
C. “I’ll have my private duty nurse take my blood glucose level as instructed by the doctor.”
D. Both A and B

4. Answer: A. “I can take drugs and skip meals so I can lose excess sugar fast.”

It is important for nurses to investigate nutritional intake, noting any problems with intake and
adherence to prescribed diet, to help prevent adverse reactions to drug therapy.

5. Which of the following is a first-generation sulfonylureas?

A. Metformin
B. Tolbutamide
C. Glimepiride
D. Pioglitazone

5. Answer: B. Tolbutamide

The first-generation sulfonylureas include chlorpropamide, tolbutamide, and tolazamide.


Glimepiride is a second-generation sulfonylurea. Metformin is a biguanide. Pioglitazone is a
thiazolidinedione.

1. Which of the following exerts its effect by inhibiting the enzyme that breaks down
glucose for absorption?

A. Miglitol
B. Metformin
C. Pramlintide
D. Linagliptin

1. Answer: A. Miglitol

The enzyme that breaks down glucose for absorption is called alpha glucosidase. Drugs under
alpha glucosidase inhibitors include acarbose and miglitol.

2. This antidiabetic agent works best in patients with insulin resistance.

A. Biguanides
B. Dipeptidyl peptidase-4 inhibitors
C. Thiazolidinediones
D. Meglitinides

2. Answer: C. Thiazolidinediones

Thiazolidinediones pioglitazone and rosiglitazone work by decreasing insulin resistance.

3. This antidiabetic drug is also used in women with polycystic ovarian syndrome.

A. Acarbose
B. Metformin
C. Rosiglitazone
D. Nateglinide

3. Answer: B. Metformin

4. Which antidiabetic agent works by modulating gastric emptying after a meal to cause a
feeling of fullness?

A. Incretin mimetics
B. Meglitinides
C. Human amylin
D. Biguanides

4. Answer: C. Human amylin

Synthetic human amylin pramlintide works to modulate gastric emptying after a meal to cause a
feeling of fullness or satiety. It also prevents the postmeal rise in glucagon that usually elevates
glucose levels. Human amylin is a hormone produced by beta cells in the pancreas that is
important in regulating postmeal glucose levels. It should not be used when patient is unable to
eat.

5. Use of pioglitazone for over a year is linked with which condition?

A. Loss of effectiveness of the drug


B. Bladder cancer
C. Esophageal cancer
D. Kidney failure

5. Answer: B. Bladder cancer

Pioglitazone is strongly linked with an increased risk of bladder cancer if it is used for over one
year.

1. The three primary elements that determine the pressure in the cardiovascular
system are heart rate, stroke volume, and:

A. Baroreceptors
B. Peripheral resistance
C. Renin-Angiotensin-Aldosterone System (RAAS)
D. All of the above

2. Untreated hypertension can result to which of the following:

A. Stroke
B. Renal failure
C. Loss of vision
D. All of the above

3. A student nurse was asked to take the blood pressure of a patient in the emergency
room. The BP reads 140/90 mmHg. She would be correct to say that this BP belongs to
____________ classification.

A. Normal
B. Elevated
C. Stage 1 Hypertension
D. Stage 2 Hypertension

4. Which of the following medications will decrease the therapeutic effects of quinapril
(Accupril)?
A. NSAIDs
B. Allopurinol
C. Antacids
D. All of the above

5. A patient receiving nitroprusside begun manifesting headache, distant heart sounds,


imperceptible pulses, and shallow breathing. The nurse knows that these are signs of?

A. Reflex tachycardia
B. Hypothyroidism
C. Cyanide toxicity
D. Severe alteration of blood pressure

1. The phase of cardiac muscle cell action potential characterized by calcium entering the
cell and potassium leaving the cell.

A. Phase 1
B. Phase 2
C. Phase 3
D. Phase 4

2. Beta-adrenergic blockers belong to which class of antiarrhythmics?

A. Class Ia
B. Class Ib
C. Class II
D. Class IV

3. Which class of antiarrhythmics block potassium channels?

A. Class Ia
B. Class IV
C. Class Ib
D. Class III

4. This is the preferred antiarrhythmic in Advanced Cardiac Life Support protocol.

A. ibutilide (Corvert)
B. propranolol (Inderal)
C. procainamide (Pronestyl)
D. amiodarone (Cordarone)
5. What is the difference between Class Ia and Ib antiarrhythmics?

A. Ia depresses phase 0 and Ib depressed phase I.


B. Ia shortens the duration of action potential and Ib prolongs the duration of action potential.
C. Ia prolongs the duration of action potential and Ib shortens the duration of action potential.
D. Ia extremely slows down conduction and Ib has no effect on conduction.

6. A patient who is receiving quinidine should avoid the following foods, except:

A. skim milk
B. broccoli
C. Both A and B
D. none of the above

7. The drug of choice for treatment of supraventricular tachycardia associated with Wolff-
Parkinson-White syndrome.

A. adenosine
B. verapamil
C. digoxin
D. lidocaine

1. Lipid levels of individuals with coronary artery disease (CAD) is usually high. All of the
following are factors of CAD, except:

A. Men
B. Gout
C. Untreated Chlamydia infections
D. None of the above

2. Antihyperlipidemic agent that is used to decrease plasma cholesterol levels.

A. HMG-CoA reductase inhibitors


B. Phosphodiesterase inhibitors
C. Bile acid sequestrants
D. Cholesterol absorption inhibitor

3. A pregnant woman needs a lipid-lowering agent. What would be the best class of lipid-
lowering agent for pregnant women?
A. HMG-CoA reductase inhibitors
B. Bile acid sequestrants
C. Cholesterol absorption inhibitors
D. Phosphodiesterase inhibitors

4. The only statin with outcome data to show effectiveness in decreasing CAD and
incidence of myocardial infarction.

A. atorvastatin (Lipitor)
B. simvastatin (Zocor)
C. pravastatin (Pravachol)
D. fluvastatin (Lescol)

5. Which of the following medical conditions will render antihyperlipidemics ineffective?

A. biliary obstruction
B. diabetes mellitus
C. both A and B
D. none of the above

6. This drug works on the brush border of the intestines.

A. ezetimibe
B. pitavastatin
C. gemfibrozil
D. colestipol

7. A patient who is taking a bile acid sequestrant complains of abdominal distention and
nausea. What should the nurse do?

A. Document, withdraw drug, and notify doctor.


B. Provide comfort measures.
C. Prepare emergency equipment at bedside.
D. Dismiss the complaint.

8. What vitamin plays a role in lowering cholesterol concentration?


A. Vitamin C
B. Vitamin E
C. Vitamin B3
D. Vitamin B2

9. When should statins be taken to maximize its therapeutic effects?

A. After meals
B. During meals
C. At night
D. In the mornings

10. The doctor ordered atorvastatin for an obese client’s hyperlipidemia. This patient is
receiving erythromycin for a toe infection at the same time. The nurse knows that this
combination should be avoided because?

A. It increases risk of atorvastatin toxicity.


B. It can result to potentially fatal lipolysis.
C. It can result to breakdown of muscles.
D. It decreases the effectiveness of erythromycin.

1. Type of angina which involves spasm of the blood vessels

A. Stable angina
B. Pre-infarction angina
C. Unstable angina
D. Prinzmetal angina

2. Contraction and relaxation in each cardiac cycle is controlled by:

A. autonomic nervous system


B. the heart
C. cranial nerves
D. central nervous system

3. All of the following can cause arrhythmia, except:

A. acidosis
B. respiratory depression
C. hyperkalemia
D. none of the above
4. Verapamil and diltiazem belong to which class of antiarrhythmics?

A. Class IV
B. Class III
C. Class Ia
D. Class II

5. The conduction system of the heart include the following:

A. Sinoatrial node
B. Bundle of Purkinje
C. His Fibers
D. Atriomyocardial node

1. The primary treatment for heart failure (HF) is ___________.

A. Increasing the heart rate so the heart can pump more blood
B. Decreasing the heart rate so the heart can rest
C. Increasing contractility so the heart will be able to pump more blood
D. Decreasing contractility to prevent muscle fatigue

2. The most common cause of HF is ______________.

A. Hypertension
B. Valvular heart diseases
C. Cardiomyopathy
D. Coronary artery disease (CAD)

3. Digoxin was prescribed to a patient with ventricular tachycardia. What should the
nurse do?

A. Administer the drug as ordered.


B. Discuss the order with the doctor.
C. Discontinue other intravenous medications before administering digoxin.
D. Count apical pulse for one full minute before administering.

4. What is the antidote for digoxin intoxication?


A. Diphenhydramine
B. Atropine sulfate
C. Digoxin immune fab (Digibind, DigiFab).
D. Phosphodiesterase inhibitors

5. An infant who is receiving cardiac glycosides has an apical pulse of 80 beats per minute.
Which is the best nursing intervention for this assessment finding?

A. Administer drug as ordered.


B. Withdraw the drug and notify doctor.
C. Assess apical pulse every hour for the next five hours.
D. Decrease drug dose and administer.

6. What signals the novice nurse that intravenous milrinone was combined to furosemide
in management of patients with heart failure?

A. Presence of bubbles
B. Pink discoloration of the solution
C. Formation of precipitates
D. No obvious sign. Solution is clear.

7. What is the therapeutic level for digoxin?

A. 0.5-2 ng/mL
B. 1.5-2 ng/mL
C. 0.5-1.5 mg/mL
D. 0.5-2 mg/mL

8. In severe cardiac glycoside toxicity, all of the following should be in the bedside, except
_________.

A. Lidocaine
B. Phenytoin
C. Calcium channel blocker
D. A and B only

1. Which organ(s) produce(s) clotting factors?

A. Bone marrow
B. Spleen and other lymphoid tissues
C. Liver
D. Both B and C
2. What is the first reaction to a blood vessel injury?

A. Vasodilation and swelling


B. Vasoconstriction
C. Bleeding
D. Blood coagulation

3. Which of the following substances will alert the nurse for an increased risk of bleeding
in a patient taking clopidogrel?

A. Chamomile tea
B. Orange juice
C. Candied mushrooms
D. Peanuts

1. Most diuretics’ effect in the body is __________.

A. loss of sodium
B. loss of potassium
C. loss of calcium
D. retention of potassium

2. What is the only potassium-sparing diuretics that can be used in children?

A. Furosemide
B. Spironolactone
C. Amiloride
D. Triamterene

3. A patient receiving diuretics should alert the nurse if she feels the following:

A. insomnia
B. low-grade fever
C. muscle weakness
D. all of the above

4. What is the first loop diuretic introduced?


A. Furosemide
B. Bumetanide
C. Torsemide
D. Ethacrynic acid

5. A patient admitted for cerebral swelling complained of feeling light-headed and


nauseous while receiving mannitol. What should the nurse do?

A. Document, withdraw, and notify doctor.


B. Decrease mannitol flow.
C. Provide comfort measures.
D. Increase mannitol flow.

1. Beta-adrenergic agonists such as albuterol are given to Carlo, a child with asthma. Such
drugs are administered primarily to do which of the following?

A. Reduce airway inflammation


B. Decrease postnasal drip
C. Reduce secondary infections
D. Dilate the bronchioles

1. Answer: D. Dilate the bronchioles

 Option D: Beta-adrenergic agonists, such as albuterol, are highly effective


bronchodilators and are used to dilate the narrow airways associated with asthma.
 Option A: Corticosteroids may be used for their anti-inflammatory effect.
 Option B: Decongestants may be given to decrease postnasal drip.

 Option C: Antibiotics are used to prevent secondary infection.


2. Maria is rushed to the emergency department during an acute, severe prolonged
asthma attack and is unresponsive to usual treatment. The condition is referred to as
which of the following?

A. Intrinsic asthma
B. Extrinsic asthma
C. Status asthmaticus
D. Reactive airway disease

2. Answer: C. Status asthmaticus

 Option C: Status asthmaticus is an acute, prolonged, severe asthma attack that is


unresponsive to usual treatment. Typically, the child requires hospitalization.
 Option A: Intrinsic is a term used to denote internal precipitating factors, such as
viruses.
 Option B: Extrinsic is a term used to denote external precipitating factors, such as
allergens.
 Option D: Reactive airway disease is another general term for asthma
3. A client is receiving theophylline intravenously. After several dosages, the client started
to become restless and complains of palpitations. The nurse determines that the client is
experiencing theophylline toxicity in which of the following?

A. Theophylline level of 2.5 mcg/ml


B. Theophylline level of 5 mcg/ml
C. Theophylline level of 20mcg/ml
D. Theophylline level of 25mcg/ml

3. Answer: D. Theophylline level of 25mcg/ml

 Option D: Theophylline toxicity is likely to occur when the serum level is higher than 20
mcg/ml. Early signs of toxicity include restlessness, nervousness, tachycardia, tremors,
and palpitations.
4. The nurse is giving medication teachings to a client receiving theophylline. The nurse
instructs the client to limit the intake of which of the following?

A. Strawberries and avocado


B. Butter and cheese
C. Salmon and tuna
D. Hot tea and cocoa

4. Answer: D. Hot tea and cocoa

 Option D: Theophylline is a methylxanthine bronchodilator. The nurse instructs the


client to limit the intake of xanthine-containing foods such as chocolate, cola, cocoa,
tea, and coffee.
5. When administering the methylxanthine theophylline, the nurse can expect:
A. Decreased pulmonary function
B. Decreased tidal volume
C. Increased pulmonary function
B. Increased residual volume

5. Answer: C. Increased pulmonary function.

 Option C: Theophylline will improve ventilation so there will be an overall


improvement of pulmonary measurements.
 Options A, B, D: Other choices are the opposite of what will actually occur with
theophylline administration.
1. A nurse is preparing to give a lung surfactant to a 36 weeks old baby with a respiratory
distress syndrome. Which of the following is the correct route of administration?

A. Intravenous
B. Intradermal
C. Intramuscular
D. Intratracheal

1. Answer: D. Intratracheal

 Option D: Lung surfactant is instilled through the catheter inserted into the newborn’s
endotracheal tube.
 Options A, B, and C: Options B, C, and D are not the routes of administration for
this medication.
2. A patient with acute respiratory distress syndrome (ARDS) is receiving oxygen by a
nonrebreather mask, but arterial blood gas measurements still show poor oxygenation.
As the nurse responsible for this patient’s care, you would anticipate a physician order for
what action?

A. Administer furosemide (Lasix) 100 mg IV push stat


B. Perform endotracheal intubation and initiate mechanical ventilation
C. Call a code for respiratory arrest
D. Immediately begin continuous positive airway pressure (CPAP) via the patient’s nose
and mouth

2. Answer: B. Perform endotracheal intubation and initiate mechanical ventilation.

 Option B: Usually at this stage, the patient is working very hard to breathe and may go
into respiratory arrest unless health care providers intervene by providing intubation
and mechanical ventilation to decrease the patient’s work of breathing.
3. The high-pressure alarm on a patient’s ventilator goes off. When you enter the room to
assess the patient, who has ARDS, the oxygen saturation monitor reads 87% and the
patient is struggling to sit up. Which action should you take next?

A. Manually ventilate the patient while assessing possible reasons for the high-pressure alarm
B. Reassure the patient that the ventilator will do the work of breathing for him
C. Insert an oral airway to prevent the patient from biting on the endotracheal tube
D. Increase the fraction of inspired oxygen on the ventilator to 100% in preparation for
endotracheal suctioning

3. Answer: A. Manually ventilate the patient while assessing possible reasons for the high-
pressure alarm

 Option A: Manual ventilation of the patient will allow you to deliver a FiO2 of 100% to
the patient while you attempt to determine the cause of the high-pressure alarm.
 Options B, C, D: The patient may need reassurance, suctioning, and/or insertion of an
oral airway, but the first step should be an assessment of the reason for the high-
pressure alarm and resolution of the hypoxemia.
4. To improve respiratory status, which medication should you be prepared to administer
to the newborn infant with respiratory distress syndrome (RDS)?

A. Decadron
B. Theolair
C. Exosurf
D. Proventil

4. Answer: C. Exosurf

 Option C: Exosurf neonatal is a form of synthetic surfactant. An infant with RDS may be
given two to four doses during the first 24 to 48 hours after birth. It improves
respiratory status, and research has shown a significant decrease in the incidence
of pneumothorax when it is administered.
5. A nurse in the newborn nursery is monitoring a preterm newborn infant for respiratory
distress syndrome. Which assessment signs, if noted in the newborn infant, would alert
the nurse to the possibility of this syndrome?

A. The presence of a barrel chest with grunting


B. Hypotension and Bradycardia
C. Acrocyanosis and grunting
D. Tachypnea and retractions

5. Answer: D. Tachypnea and retractions.


 Option D: The infant with respiratory distress syndrome may present with signs of
cyanosis, tachypnea or apnea, nasal flaring, chest wall retractions, or audible grunts.
1. A Cromolyn sodium (Intal) inhaler is prescribed to a client with asthma.
A nurse provides instructions regarding the side effects of this medication. The nurse tells
the client that which undesirable effect is associated with this medication?

A. Insomnia
B. Constipation
C. Wheezing
D. Hypotension

1. Answer: C. Wheezing

 Option C: Cromolyn Sodium (Intal) is used to prevent asthma attacks in people


with bronchial asthma. Undesirable side effects associated with the use of inhaler is
wheezing, cough, nasal congestion, bronchospasm, and throat irritation.
2. A nurse is about to administer Albuterol (Ventolin HFA) 2 puff and Budesonide
(Pulmicort Turbohaler) 2 puff by metered dose inhaler. The nurse plans to administer by?

A. Alternating with a single puff each, starting with albuterol.


B. Alternating with a single puff each, starting with budesonide.
C. Budesonide inhaler first then the albuterol.
D. Albuterol inhaler first then the budesonide.

2. Answer: D. Albuterol inhaler first then the budesonide.

 Option D: If two different inhaled medications are prescribed and one of the
medications contains a corticosteroid, administer the bronchodilator (Albuterol) first
and the corticosteroid (Budesonide) second. This will allow for the widening of the air
passages by the bronchodilator, making the corticosteroids more effective.
3. A nurse teaches a client about the use of a respiratory inhaler. Which action by the
client indicated a need for further teaching?

A. Removes the cap and shakes the inhaler well before use.
B. Presses the canister down with finger as he breathes in.
C. Inhales the mist and quickly exhales.
D. Waits 1 to 2 minutes between puffs if more than one puff has been prescribed.

3. Answer: C. Inhales the mist and quickly exhales.

 Option C: The client should be instructed to hold his or her breath at least 10 to 15
seconds before exhaling the mist.
4. A client with acute asthma is prescribed short-term corticosteroid therapy. What is the
rationale for the use of steroids in clients with asthma?

A. Corticosteroids promote bronchodilation.


B. Corticosteroids act as an expectorant.
C. Corticosteroids have an anti-inflammatory effect.
D. Corticosteroids prevent the development of respiratory infections.

4. Answer: C. Corticosteroids have an anti-inflammatory effect.

 Option C: Corticosteroids have an anti-inflammatory effect and act to decrease edema


in the bronchial airways and decrease mucus secretion.
 Options A, B, D: Corticosteroids do not have a bronchodilator effect, act as
expectorants, or prevent respiratory infections.
5. The nurse is teaching the client how to use a metered dose inhaler (MDI) to administer
a Corticosteroid drug. Which of the following client actions indicates that he is using the
MDI correctly? Select all that apply.

A. The inhaler is held upright.


B. Head is tilted down while inhaling the medication.
C. Client waits 5 minutes between puffs.
D. Mouth is rinsed with water following administration.
E. Client lies supine for 15 minutes following administration.

5. Answer: A and D.

 Option A: The inhaler is held upright.


 Option D: Mouth is rinsed with water following administration.

1. Gab, a 20-year-old student, used to buy OTC drugs whenever he feels sick. Which of the
following statements best describes the danger of self-medication with over-the-counter
drugs?

A. Clients are not aware of the action of over-the-counter drugs


B. Clients are not aware of the side effects of over-the-counter drugs
C. Clients do not realize the effects of over-the-counter drugs
D. Clients minimize the effects of over-the-counter drugs because they are available without a
prescription
1. Answer: D. Clients minimize the effects of over-the-counter drugs because they are
available without a prescription.

 Option D: This choice is correct because it includes the other three risks noted in
choices A, B, and C.
2. Which of the following would be an expected outcome for a client recovering from an
upper respiratory tract infection? The client will:

A. Experience less nasal obstruction and discharge


B. Cough productively without chest discomfort
C. Experience chills only once a day
D. Maintain a fluid intake of 800 ml every 24 hours

2. Answer: A. Experience less nasal obstruction and discharge.

 Option A: A client recovering from an upper respiratory infection (URI) should report
decreasing or no nasal discharge and obstruction.
 Option B: A productive cough with chest pain indicated pulmonary infection, not an
upper respiratory infection (URI).
 Option C: The temperature should be below 100*F (37.8*C) with no chills or
diaphoresis.
 Option D: Daily fluid intake should be increased to more than 1 L every 24 hours to
liquefy secretions.
3. Pseudoephedrine (Sudafed) has been ordered as a nasal decongestant. Which of the
following is a possible side effect of this drug?

A. Diplopia
B. Restlessness
C. Constipation
D. Bradycardia

3. Answer: B. Restlessness.

 Option B: Side effects of pseudoephedrine are experienced primarily in


the cardiovascular system and through sympathetic effects on the CNS. The most
common CNS effects include restlessness, dizziness, tension, anxiety, insomnia, and
weakness.
 Options A and C: Constipation and diplopia are not side effects of pseudoephedrine.
 Option D: Tachycardia, not bradycardia, is a side effect of pseudoephedrine.
4. A client has been taking flunisolide (Aerobid), two inhalations a day, for treatment
of asthma. He tells the nurse that he has painful, white patches in his mouth. Which
response by the nurse would be the most appropriate?

A. “You have developed a fungal infection from your medication. It will need to be treated with
an antibiotic.”
B. “Be sure to brush your teeth and floss daily. Good oral hygiene will treat this problem.”
C. “This is an anticipated side-effect of your medication. It should go away in a couple of weeks.”
D. “You are using your inhaler too much and it has irritated your mouth.”

4. Answer: A. “You have developed a fungal infection from your medication. It will need to
be treated with an antibiotic.”

 Option A: Use of oral inhalant corticosteroids, such as flunisolide, can lead to the
development of oral thrush, a fungal infection.
 Option B: Although good oral hygiene can help prevent the development of a fungal
infection, it cannot be used alone to treat the problem.
 Option C: Once developed, thrush must be treated by antibiotic therapy; it will not
resolve on its own.
 Option D: Fungal infections can develop even without overuse of the Corticosteroid
inhaler.
5. A nurse is about to administer Albuterol (Ventolin HFA) 2 puff and Budesonide
(Pulmicort Turbohaler) 2 puff by metered dose inhaler. The nurse is planning to
administer it by?

A. Budesonide inhaler first then the albuterol


B. Albuterol inhaler first then the budesonide
C. Alternating with a single puff each, starting with albuterol
D. Alternating with a single puff each, starting with budesonide

5. Answer: B. Albuterol inhaler first then the budesonide.

 Option B: If two different inhaled medications are prescribed and one of the
medications contains a corticosteroid, administer the bronchodilator (Albuterol) first
and the corticosteroid (Budesonide) second. This will allow for the widening of the air
passages by the bronchodilator, making the corticosteroids more effective.

1. Stephanie will be having her exam in pharmacology tomorrow. She should be aware
that antitussive is indicated to:
A. Encourage removal of secretions through coughing
B. Relieve rhinitis
C. Relieve a dry cough
D. Control a productive cough

1. Answer: C. Relieve a dry cough.

 Option C: An antitussive is a cough suppressant.


 Options A and D: Represents the action of an expectorant.
 Option B: Describe the action of a decongestant.
2. Which of the following pathophysiological mechanisms that occur in the lung
parenchyma allows pneumonia to develop?

A. Atelectasis
B. Bronchiectasis
C. Effusion
D. Inflammation

2. Answer: D. Inflammation.

 Option D: The common feature of all type of pneumonia is an inflammatory


pulmonary response to the offending organism or agent.
 Options A and B: Atelectasis and bronchiectasis indicate a collapse of a portion of the
airway that doesn’t occur in pneumonia.
 Option C: An effusion is an accumulation of excess pleural fluid in the pleural space,
which may be a secondary response to pneumonia.
3. Antitussives are useful in blocking the cough reflex and preserving the energy
associated with prolonged, nonproductive coughing. Antitussives are best used with the
following except:

A. Postoperative patients
B. COPD patients who tire easily
C. Patients with a dry, irritating cough
D. Asthma patients

3. Answer: D. Asthma patients.

 Option D: Patients with asthma need patent airways which coughing could provide.
 Options A, B, and C: These conditions may warrant the use of antitussives.
4. Antitussives are indicated for which of the following patients?
A. Timothy who has difficulty of breathing
B. David who has just recovered from narcotine addiction
C. Aubry who has dry cough
D. Kelley who is pregnant

4. Answer: C. Aubry who has dry cough.

 Option C: One of the indications of administration of antitussives is a nonproductive,


irritating cough.
 Option A: Antitussives suppress the cough reflex which could compromise the airway
of someone who has difficulty of breathing.
 Option B: Narcotine addiction is a contraindication for administering of antitussives.
 Option D: Antitussives are contraindicated for a patient who is pregnant.
5. Use of antitussives is associated with which of the following adverse effects?

A. Drowsiness
B. Constipation
C. Nausea
D. All of the above

5. Answer: D. All of the above.

 Options A, B, and C: All of these symptoms are adverse effects of antitussives.

1. Which of the following statements describes the action of antacids?

A. Antacids block the production of gastric acid


B. Antacids enhance the action of acetylcholine
C. Antacids block dopamine
D. Antacids neutralize gastric acid

1. Answer: D. Antacids neutralize gastric acid

 Option D: Antacids act to bring the pH above 3.


 Options A, B, and C: Other choices are incorrect because they describe actions of
antiacid drugs.
2. Patient Gavin is taking antacids, which instruction would be included in the teaching
plan?
A. “Avoid taking other medications within 2 hours of this one.”
B. “Continue taking antacids even when pain subsides.”
C. “Weigh yourself daily when taking this medication.”
D. “Take the antacids with 8 oz of water.”

2. Answer: A. “Avoid taking other medications within 2 hours of this one.”

 Option A: The client should be instructed to avoid taking other medications within 2
hours of the antacid.
 Option B: A histamine receptor antagonist should be taken even when the pain
subsides.
 Option C: Daily weights are indicated if the client is taking a diuretic, not an antacid.
 Option D: Water, which dilutes the antacid, should not be taken with an antacid.
3. Nurse Victoria is teaching a group of middle-aged men about peptic ulcers. When
discussing risk factors for peptic ulcers, the nurse should mention:

A. A history of hemorrhoids and smoking


B. A sedentary lifestyle and smoking
C. Alcohol abuse and smoking
D. Alcohol abuse and a history of acute renal failure

3. Answer: C. Alcohol abuse and smoking

 Option C: Risk factors for peptic (gastric and duodenal) ulcers include alcohol abuse,
smoking, and stress.
 Options A & B: A sedentary lifestyle and a history of hemorrhoids aren’t risk factors for
peptic ulcers.
 Option D: Chronic renal failure, not acute renal failure, is associated with duodenal
ulcers.
4. Which of the following tests can be used to diagnose ulcers?

A. Barium swallow
B. Abdominal x-ray
C. Esophagogastroduodenoscopy (EGD)
D. Computed tomography (CT) scan

4. Answer: C. Esophagogastroduodenoscopy (EGD)


 Option C: The EGD can visualize the entire upper GI tract as well as allow for
tissue specimens and electrocautery if needed.
 Option A: The barium swallow could locate a gastric ulcer.

 Options B and D: A CT scan and an abdominal x-ray aren’t useful in the diagnosis
of an ulcer.
5. The hospitalized client with GERD is complaining of chest discomfort that feels like
heartburn following a meal. After administering an ordered antacid, the nurse
encourages the client to lie in which of the following positions?

A. On the stomach with the head flat


B. Supine with the head of the bed flat
C. On the right side with the head of the bed elevated 30 degrees
D. On the left side with the head of the bed elevated 30 degrees

5. Answer: D. On the left side with the head of the bed elevated 30 degrees.

 Option D: The left side-lying position with the head of the bed elevated is most likely to
give relief to the client.
 Options A, B, C: These include lying flat on the back or on the stomach after a meal or
lying on the right side.

1. Which of the following best describes the method of action of medications, such as
ranitidine (Zantac), which are used in the treatment of peptic ulcer disease?

A. Neutralize acid
B. Reduce acid secretions
C. Stimulate gastrin release
D. Protect the mucosal barrier

1. Answer: B. Reduce acid secretions.

 Option B: Ranitidine is a histamine-2 receptor antagonist that reduces acid secretion


by inhibiting gastrin secretion.
2. A client is to take one daily dose of ranitidine (Zantac) at home to treat her peptic ulcer.
The nurse knows that the client understands proper drug administration of ranitidine
when she says that she will take the drug at which of the following times?

A. Before meals
B. With meals
C. At bedtime
D. When pain occurs

2. Answer: C. At bedtime.

 Option C: Ranitidine blocks secretion of hydrochloric acid. Clients who take only one
daily dose of ranitidine are usually advised to take it at bedtime to inhibit nocturnal
secretion of acid.
3. The nurse provides medication instructions to a client with peptic ulcer disease. Which
statement, if made by the client, indicates the best understanding of the medication
therapy?

A. “The cimetidine (Tagamet) will cause me to produce less stomach acid.”


B. “Sucralfate (Carafate) will change the fluid in my stomach.”
C. “Antacids will coat my stomach.”
D. “Omeprazole (Prilosec) will coat the ulcer and help it heal.”

3. Answer: A. “The cimetidine (Tagamet) will cause me to produce less stomach acid.”

 Option A: Cimetidine (Tagamet), a histamine H2 receptor antagonist, will decrease the


secretion of gastric acid.
 Option B: Sucralfate (Carafate) promotes healing by coating the ulcer.
 Option C: Antacids neutralize acid in the stomach.
 Option D: Omeprazole (Prilosec) inhibits gastric acid secretion.
4. When a client has peptic ulcer disease, the nurse would expect a priority intervention
to be:

A. Assisting in inserting a Miller-Abbott tube


B. Assisting in inserting an arterial pressure line
C. Inserting a nasogastric tube
D. Inserting an I.V.

4. Answer: C. Inserting a nasogastric tube.

 Option C: An NG tube insertion is the most appropriate intervention because it will


determine the presence of active GI bleeding.
 Option A: A Miller-Abbott tube is a weighted, mercury-filled ballooned tube used to
resolve bowel obstructions.
 Options B and D: There is no evidence of shock or fluid overload in the client;
therefore, an arterial line is not appropriate at this time and an IV is optional.
5. A 40-year-old male client has been hospitalized with peptic ulcer disease. He is being
treated with a histamine receptor antagonist (cimetidine), antacids, and diet. The nurse
doing discharge planning will teach him that the action of cimetidine is to:

A. Reduce gastric acid output.


B. Protect the ulcer surface.
C. Inhibit the production of hydrochloric acid (HCl).
D. Inhibit vagus nerve stimulation.

5. Answer: A. Reduce gastric acid output.

 Option A: These drugs inhibit the action of histamine on the H2 receptors of parietal
cells, thus reducing gastric acid output.
1. The physician has prescribed Nexium (esomeprazole) for a client with erosive gastritis.
The nurse should administer the medication:

A. 30 minutes after meals


B. 30 minutes before meals
C. With each meal
D. In a single dose at bedtime

1. Answer: B. 30 minutes before meals

 Option B: Proton pump inhibitors reduce the production of acid in the stomach.
Proton pump inhibitors work best when they are taken 30 minutes before the first
meal of the day.
2. Proton pump inhibitor use will likely result in:

A. Heartburn
B. Diverticulosis
C. Gastric ulcer formation
D. Achlorhydria

2. Answer: D. Achlorhydria.

 Option D: Because the proton pump inhibitors stop the final step of acid secretion,
they can block up to 90% of acid secretion, leading to achlorhydria (absence of acid).
3. A patient is prescribed with esomeprazole for the treatment of GERD. Upon review of
current medication use, the nurse noted that the patient is taking clopidogrel. The nurse
warned the patient that esomeprazole:
A. Have no evidence of potential interaction with clopidogrel
B. Increase the effectiveness of clopidogrel
C. Decrease the effectiveness of clopidogrel
D. Increase acid production

3. Answer: C. Decrease the effectiveness of clopidogrel

Option C: Esomeprazole inhibits CYP2C19 enzyme which serves as a pathway for



certain medication. One of which is clopidogrel, so taking it with esomeprazole will
potentially decrease the effectiveness of clopidogrel.
4. A nurse is checking the nasogastric tube position of a client receiving a long term
therapy of Omeprazole (Prilosec) by aspirating the stomach contents to check for
the PH level. The nurse proves that correct tube placement if the PH level is?

A. 5
B. 6.3
C. 6.5
D. 7

4. Answer: A. 5

Option A: Gastric placement is indicated by a pH of less than 4, but may increase to



between pH 4-6 if the patient is receiving acid-inhibiting drugs.
5. Prolonged use of Proton Pump Inhibitors will likely result with the following except:

A. Hypermagnesemia
B. Pneumonia
C. Fractures
D. Hypochlorhydria

5. Answer: A. Hypermagnesemia

 Option A: Long term use of PPIs affects intestinal magnesium absorption leading
to hypomagnesemia.

You might also like